THMMY.gr

Μαθήματα Τηλεπικοινωνιακού Κύκλου => Ψηφιακή Επεξεργασία Σήματος => Topic started by: shen on December 20, 2011, 16:00:10 pm



Title: [ΨΕΣ] Λύσεις παλιών θεμάτων
Post by: shen on December 20, 2011, 16:00:10 pm
Ανοίγω αυτό το topic για να συζυτήσουμε λύσεις παλιών θεμάτων.

Ξεκινώ εγώ με κάποιες σκέψεις πάνω στο 2ο θέμα του Φεβρουαρίου του 2009 (υπάρχει στα downloads).

Η περίπτωση που περιγράφεται είναι αντίστοιχη με τη μέθοδο επικάλυψης κράτησης (λόγω των μηκων των ακολουθιών δεν μπορεί να παραλληλιστεί με την μέθοδο επικάλυψης-πρόσθεσης).  Παρόλαυτα στη μέθοδο επικάλυψης τα μηδενικά στοιχεία είναι τα πρώτα L ενώ σε αυτό το θέμα θα είναι τα τελευταία L ψηφία. Δεν έχω βρει κάτι παρακάτω...



edit by mod: title


Title: Re: [ΨΕΣ] Λύσεις παλιών θεμάτων
Post by: shen on January 26, 2012, 16:15:11 pm
Λοιπόν, απαντώ στο 2ο θέμα του 2009, γιατί μπορεί και κάποιος άλλος να ψάχνει λύση.

Στα θέματα του 2008 ανέφερε πάνω στην εκφώνιση ότι μια ακολουθία xi[n] μήκους i, που προέκυψε από aliasing, συνδέεται με την πραγματική ακολουθία, η οποία είναι μήκους j, από τον τύπο:

xi[n] = Σ <για λ από - άπειρο έως άπειρο> xj[n-λi]

Αυτό το πήρα ως κάτι δεδομένο και το χρησιμποιήσα για το θέμα του 2009.


Στο θέμα, η γραμμική συνέλιξη έχει το πρώτο μη μηδενικό της στοιχείο στο 0+0=0 και το τελευταίο μη μηδενικό στοιχείο στο 7+19=26, δηλαδή είναι μήκους j=27 (μετράμε και το 0).

Η κυκλική συνέλιξη μας λέει ότι είναι μήκους i=20 (πάλι μετράμε και το 0).

Οι τιμές τις κυκλικής συνέλιξης που θα συμπίπτουν με αυτές της γραμμικής θα είναι εκείνες στις οποίες λ=0 επειδή για οποιαδήποτε άλλη τιμή του λ xj=0.

2 * 20 > 27 άρα το λ θα πάρει τιμές 0 και -1 γιατί για οποιαδήποτε άλλη το xj[n-λ*20]=0.

Το μέγιστο n για το οποίο το λ θα πάρει και την τιμή -1 είναι το 26-20=6. Άρα οι τιμές τις R(k) που συμπίπτουν με αυτές της Rl είναι οι R(7), R(8),...,R(19)



Title: Re: [ΨΕΣ] Λύσεις παλιών θεμάτων
Post by: Αιμιλία η φτερωτή χελώνα on January 26, 2012, 16:42:25 pm
Yeah right!Ασκηση 2.7 βιβλιο Πανα δλδ


Title: Re: [ΨΕΣ] Λύσεις παλιών θεμάτων
Post by: ILIAS on January 26, 2012, 16:53:59 pm
Δεν μπορούμε απλά να αναφέρουμε τον κανόνα ότι Ν+L-1 το μήκος της γραμμικής συνέλιξης και εφοσον είναι 26>20 όπου 20 το μήκος της συνέλιξης-20σημείων τότε τα 26-20=6 πρώτα στοιχεία της γραμμικής θα αλλοιωθούν και άρα καταλήγω σε αυτό που πες . Σελ.329 Schaum


Title: Re: [ΨΕΣ] Λύσεις παλιών θεμάτων
Post by: ILIAS on January 26, 2012, 18:49:49 pm
Βασικά αυτό γιατί να μας απασχολεί; Οι μέθοδοι επικάλυψης -πρόσθεσης και επικάλυψης-κράτησης μας δείχνουν πως να υπολογίσουμε μία συνέλιξη μεγάλου μήκους. Το αποτέλεσμα ίδιο θα είναι.


Title: Re: [ΨΕΣ] Λύσεις παλιών θεμάτων
Post by: ectoras on January 26, 2012, 18:53:57 pm
οκ απλα το ανεφερα γιατι έκανε λόγο ο shen για αυτα το πρώτο ποστ

απλά εγώ αυτό το θέμα το καταλαβαίνω με βάση την 6.15 στο θέμα του 2009


Title: Re: [ΨΕΣ] Λύσεις παλιών θεμάτων
Post by: ggpyr on January 26, 2012, 19:00:02 pm
Ρε παιδιά, από ότι καταλαβα είναι απλά να συγκρίνεις την κυκλική με τη γραμμική συνέλιξη, τι τύπους και aliasing μου λέτε  :D


Title: Re: [ΨΕΣ] Λύσεις παλιών θεμάτων
Post by: b@ki on January 26, 2012, 20:28:32 pm
ρε παιδιά σε ποιό κομμάτι της ύλης απο το βιβλίο του Schaum αντιστοιχεί το θέμα 3 του 2008??'οπως και το θέμα 4 του 2009? :o :o


Title: Re: [ΨΕΣ] Λύσεις παλιών θεμάτων
Post by: ILIAS on January 26, 2012, 22:37:10 pm
Για το δ του 2009, τώρα ή είναι τόσο απλό ή ο ποιητής άλλα θέλει να πει. Το σχήμα μας δείχνει τον μετασχηματισμό Fourier του αναλογικού σήματος. εμείς θέλουμε μια συχνότητα δειγματοληψίας τέτοια που να μην έχουμε επικάλυψη, άρα από θεώρημα Nyquist fsmin =2*f όπου f  η μέγιστη συχνότητα του φουριέ συνεχούς χρόνου. Αυτά τώρα για το άλλο δεν ξέρω ακόμη :P


Title: Re: [ΨΕΣ] Λύσεις παλιών θεμάτων
Post by: ILIAS on January 26, 2012, 22:58:17 pm
Για του 2008 ε το πρώτο φαίνεται ότι η περίδος είναι Ν=40 και στο πεδίο του χρόνου Τ=Ν*Τs όπου τη συχνότητα δίδεται.  Το διακριτό σήμα έχει αναλυθεί ήδη σε τρείς συχνότητες αν προσέξετε οι οποίες είναι 0,066*10000=660
0.172*10000=1720
0.241*10000=2410 Hz
οπότε  είναι η λέξη had ? Δεν ξέρω πείτε  :D


Title: Re: [ΨΕΣ] Λύσεις παλιών θεμάτων
Post by: shen on January 26, 2012, 23:17:44 pm
b@ki στο 3, δειγματοληψία, αν και είναι εκτός ύλης.

Στο τέταρτο 4 δείγματα δε θα πάρεις? για να έχεις 4 εξισώσεις για τους 4 αγνώστους σου?


Title: Re: [ΨΕΣ] Λύσεις παλιών θεμάτων
Post by: ILIAS on January 26, 2012, 23:32:42 pm
Αυτό να φανταστώ θέλει γνώσεις δειγματοληψίας? Γιατί αλλιώς τι εξισώσεις να πάρω  :-\ ?


Title: Re: [ΨΕΣ] Λύσεις παλιών θεμάτων
Post by: shen on January 26, 2012, 23:57:43 pm
Βασικά αν θες να βρεις 4 αγνώστους, c0, c1, c2, c3, χρειάζεσαι 4 γραμμικά ανεξάρτητες εξισώσεις της μορφής

αC0 + βC1 + ..+ .. = κάτι γνωστό, δλδ η τιμή του δείγματος σου.

Στο 3ο κεφάλαιο το μόνο που λέει και έχει μπει είναι η συχνότητα nyquist, μπορεί όμως κάποιος να θέλει να το διαβάσει ή εμένα να μου έχει ξεφύγει κάτι.


Title: Re: [ΨΕΣ] Λύσεις παλιών θεμάτων
Post by: b@ki on January 27, 2012, 01:41:31 am
Για το 2008 ο Ηλιας πρέπει να έχει δίκιο τουλάχιστον μου φαίνεται απόλυτα λογική η λύση, για το 2009 διατηρώ τις αμφιβολίες μου μήπως εννοεί κάτι πιο ψαγμενο απο απλα την συχνότητα Nyquist..


Title: Re: [ΨΕΣ] Λύσεις παλιών θεμάτων
Post by: ectoras on September 21, 2012, 19:26:50 pm
λοιπόν παιδιά για τα φετινά θέματα, έχω την εξής απορία

το τρίτο θέμα λύνεται όπως ένα παρόμοιο λυμένο πρόβλημα στο hayes σελ 228, το θέμα είναι ότι σε εμάς θα εμφανιστεί + στον παρονομαστή, τότε λέτε εμείς να το κάνουμε -(-α)?? για να μπορούμε να πάρουμε τύπο αντίστροφου μετασχηματισμού? αλλιως κάτι άλλο δεν μπορώ να σκεφτώ

όποιος γνωρίζει σίγουρα (να είναι διαβασμένος εννοω) ας απαντήσει


Title: Re: [ΨΕΣ] Λύσεις παλιών θεμάτων
Post by: Silvo the Beautiful on February 24, 2013, 20:02:18 pm
ΘΕΜΑ 3 ΣΕΠΤΕΜΒΡΗ 12

Το τριτο αμα πας εναλλακτικα που οι εξισωσεις ειναι πιο ευκολες για επιλυση πως βρισκεις τους συντελες ομως α1 α2 β0 β1?


Title: Re: [ΨΕΣ] Λύσεις παλιών θεμάτων
Post by: geozzz on February 24, 2013, 23:25:26 pm
Μαλλον θεωρουνται γνωστοι ..



Κανεις την λυση του Θεματος 4  Ιανουαριος 2012?


Title: Re: [ΨΕΣ] Λύσεις παλιών θεμάτων
Post by: Perasmus on February 08, 2014, 20:30:15 pm
Ξερει κανεις , πως λύνεται το θέμα που σου δινει Χ(z) = Log(1 -1/2 * z^(-1)  )   και σου ζητάει να βρεις το ΙΖΤ 
χρησιμοποιώντας την ιδιότητα της Παραγώγισης;
 
 Ρωτάω γιατί έχει πέσει τοσο τον Ιανουαριο 2012 , όσο και στα Θέματα Φεβρουαρίου 2013.


Title: Re: [ΨΕΣ] Λύσεις παλιών θεμάτων
Post by: teslaaaa on February 08, 2014, 20:32:21 pm
ανεβάζω τη λύση σε ολίγον,το έχει κάνει στην τάξη..
αν βγάλετε άκρη όπως τα γράφω :P


Title: Re: [ΨΕΣ] Λύσεις παλιών θεμάτων
Post by: Perasmus on February 08, 2014, 21:05:49 pm
ανεβάζω τη λύση σε ολίγον,το έχει κάνει στην τάξη..
αν βγάλετε άκρη όπως τα γράφω :P


Σε ευχαριστουμε πολυ..!!!  ;D     Αν έχεις και την λυση του α ερωτηματος απο το ιδιο θεμα του 2013 ΄ή καποια αλλη λυση παλιου θεματος
που ειπε μεσα στην ταξη,  και εχεις την διαθεση και το χρονο να το ανεβασεις ,
θα σου ημουν απιστευτα ευγνωμων..
γιατι ο χρονος ειναι λιγος και πιεζει.. :-\


Title: Re: [ΨΕΣ] Λύσεις παλιών θεμάτων
Post by: nohponex on February 08, 2014, 22:18:14 pm
Ξερει κανεις , πως λύνεται το θέμα που σου δινει Χ(z) = Log(1 -1/2 * z^(-1)  )   και σου ζητάει να βρεις το ΙΖΤ  
χρησιμοποιώντας την ιδιότητα της Παραγώγισης;
 
 Ρωτάω γιατί έχει πέσει τοσο τον Ιανουαριο 2012 , όσο και στα Θέματα Φεβρουαρίου 2013.
Μιας και έκανα τον κόπο αλλά άργησα, ανεβάζω του Ιανουαρίου 2012 κ εγώ πάλι  ::)


Title: Re: [ΨΕΣ] Λύσεις παλιών θεμάτων
Post by: status_quo on February 08, 2014, 23:07:55 pm
για το πρωτο θεμα του Σεπτεμβριου ποιο ειναι το σωστο σχημα?και γιατι?


Title: Re: [ΨΕΣ] Λύσεις παλιών θεμάτων
Post by: status_quo on February 08, 2014, 23:09:46 pm
σορρυ ξεχασα να βαλω χρονια Σεπτεμβρης του '12


Title: Re: [ΨΕΣ] Λύσεις παλιών θεμάτων
Post by: mitseli on February 08, 2014, 23:28:06 pm
για το πρωτο θεμα του Σεπτεμβριου ποιο ειναι το σωστο σχημα?και γιατι?

Θα έλεγα το 5ο σχήμα γιατί αν δεις τον τύπο 3.1.26 το x[60] θα πρέπει να είναι μονάδα.


Title: Re: [ΨΕΣ] Λύσεις παλιών θεμάτων
Post by: status_quo on February 08, 2014, 23:36:46 pm
κι εγω αυτο ελεγα αλλα με προβληματιζει αυτο το ω^2


Title: Re: [ΨΕΣ] Λύσεις παλιών θεμάτων
Post by: BluE_NoTe on February 08, 2014, 23:57:50 pm
Για το θέμα 1) Σεπτ. 2012

θα διάλεγα το 2ο σχήμα, γιατί το Η μοιάζει με γινόμενο δυο ακολουθιών εκ των οποίων το ένα στον χρόνο μεταφράζεται σαν δ(n-60). Άρα λογικό είναι στον χρόνο να περιμένουμε ένα σχήμα που να μοιάζει με συνάρτηση δ αλλά να μην είναι ακριβώς (εξού και οι κυμάτωση) και επιπλέον να είναι κεντραρισμένο στο 60! (αυτά είναι και τα δύο χαρακτηριστικά της h(n) που θα χρησιμοποιούσα στην αιτιολόγηση)
Επίσης στο δεύτερο σχήμα η ενέργεια δεν είναι συσσωρευμένη στο n=60 αλλά εκεί βρίσκεται το μεγαλύτερο μέρος. Η υπόλοιπη βρίσκεται στην κυμάτωση που προσθέτει η δεύτερη ακολουθία (ω^3).


Title: Re: [ΨΕΣ] Λύσεις παλιών θεμάτων
Post by: teslaaaa on February 09, 2014, 12:47:55 pm
μερικά λυμένα ως το 2009
δείτε τα με προσοχή γιατί μερικά τα έχω λύσει μόνη μ :P


Title: Re: [ΨΕΣ] Λύσεις παλιών θεμάτων
Post by: teslaaaa on February 09, 2014, 12:49:53 pm
Συνέχεια


Title: Re: [ΨΕΣ] Λύσεις παλιών θεμάτων
Post by: Perasmus on February 09, 2014, 13:06:37 pm
Μπορει να φαινεται λιγο χαζη ερωτηση , αλλα πραγματικα εχω κολλησει..

Εχω μια ακολουθια h1(n) = {1/2 , 1/4 , 1/2 }  .. αυτο τι ακριβως σημαινει?
τα 1/2 και 1/4 για ποιες τιμες του n ειναι;   μηπως ειναι h1(0) = 1/2  και h1(1) = 1/4 κτλ; 

Και επισης με αφορμη το θεμα 4 απο Φεβρ. 2013 ,  πως μπορω να κανω την συνελιξη του  h1(n) = {1/2 , 1/4 , 1/2 } με το h2(n) = (n+1) u(n) ;


Title: Re: [ΨΕΣ] Λύσεις παλιών θεμάτων
Post by: teslaaaa on February 09, 2014, 13:23:12 pm
Μπορει να φαινεται λιγο χαζη ερωτηση , αλλα πραγματικα εχω κολλησει..

Εχω μια ακολουθια h1(n) = {1/2 , 1/4 , 1/2 }  .. αυτο τι ακριβως σημαινει?
τα 1/2 και 1/4 για ποιες τιμες του n ειναι;   μηπως ειναι h1(0) = 1/2  και h1(1) = 1/4 κτλ; 

Και επισης με αφορμη το θεμα 4 απο Φεβρ. 2013 ,  πως μπορω να κανω την συνελιξη του  h1(n) = {1/2 , 1/4 , 1/2 } με το h2(n) = (n+1) u(n) ;
για το πρώτο που ρωτάς, λογικά είναι h1(0) = 1/2  και h1(1) = 1/4 κλπ. όπως το έγραψες
στον σάουμ συνήθως είχε μια υπογράμμιση στην τιμή που αναφερόταν στο n=0 για να προσδιορίζει χωρίς πολλά πολλά σε ποια τιμή του n αντιστοιχεί κάθε τιμή της ακολουθίας
για το θέμα δν το έχω κοιτάξει αλλά αν γράψεις την h1(n)=(1/2)*δ(n)+(1/4)*δ(n-1)+(1/2)*δ(n-2)
μετά νομίζω βγαίνει πολύ εύκολα


Title: Re: [ΨΕΣ] Λύσεις παλιών θεμάτων
Post by: TTL on February 09, 2014, 13:44:01 pm
1o θέμα Φεβρουαρίου 13 κανείς;
Αυτό που μας δίνει 5 σημεία του DFT και ζητάει τα επόμενα 3.


Title: Re: [ΨΕΣ] Λύσεις παλιών θεμάτων
Post by: teslaaaa on February 09, 2014, 13:53:40 pm
είναι X(k)=X*(-k) επειδή έχεις πραγματική ακολουθία και X*(N-k)=X*(-k) επειδή ο DFT είναι περιοδικός με περίοδο τον αριθμό δειγμάτων της αρχικής ακολουθίας


Title: Re: [ΨΕΣ] Λύσεις παλιών θεμάτων
Post by: TTL on February 09, 2014, 14:04:59 pm
thx tesla είναι το ν-οστό μάθημα που σε συμβουλεύομαι με ν --> +οο


Title: Re: [ΨΕΣ] Λύσεις παλιών θεμάτων
Post by: teslaaaa on February 09, 2014, 14:06:50 pm
thx tesla είναι το ν-οστό μάθημα που σε συμβουλεύομαι με ν --> +οο
;D ;D
γενικά μπορεί να πετάω βλακείες καμιά φορά, αλλά για το συγκεκριμένο είμαι σίγουρη :D


Title: Re: [ΨΕΣ] Λύσεις παλιών θεμάτων
Post by: Perasmus on February 09, 2014, 14:31:19 pm

Αγαπάμε teslaaaa !!!  :D  :)


Title: Re: [ΨΕΣ] Λύσεις παλιών θεμάτων
Post by: MichaelP on February 09, 2014, 14:50:44 pm
Ξέρει κανείς το πρώτο θέμα Σεπτέμβριος του 2008? Μοιάζει με την άσκηση 2.7 το Πανά αλλά ούτε εκείνη μπορώ να την λύσω.


Title: Re: [ΨΕΣ] Λύσεις παλιών θεμάτων
Post by: Perasmus on February 09, 2014, 15:01:38 pm
Δες στην προηγουμενη σελιδα,  η teslaaaa ανεβασε λυσεις μερικων παλιων θεματων , μεσα σ αυτα εχει και αυτες του Σεπτεμβριου 2008!  :)


Title: Re: [ΨΕΣ] Λύσεις παλιών θεμάτων
Post by: teslaaaa on February 09, 2014, 15:08:45 pm
δεν την έχω μέσα..
Ορίστε ;)


Title: Re: [ΨΕΣ] Λύσεις παλιών θεμάτων
Post by: MichaelP on February 09, 2014, 16:04:23 pm
Έχει κανείς καμιά ιδέα για το θέμα 4 του Ιανουαρίου του 2012? Επίσης στο θέμα 1 της ίδιας χρονιάς παίρνουμε απλά τον τύπο ορισμού του DTFT βρίσκουμε ένα "μακαρόνι" και αντικαθιστούμε όπου ω το 2πnk/N?


Title: Re: [ΨΕΣ] Λύσεις παλιών θεμάτων
Post by: teslaaaa on February 09, 2014, 16:28:34 pm
στο 1ο Ιανουάριος 2012 ναι βρίσκεις το Χ(exp(i*ω)) πρώτα και μετά αντικαθιστάς με 2πk/N (το n έχει φύγει ήδη όταν κάνεις το άθροισμα για να βρεις τον DTFT).
στο 4ο Ιανουάριος 2012καταρχήν το σχήμα που σου δίνει είναι για τον DTFT του χ(n)..οπότε πρέπει τα μπλοκ διαγράμματα να τα ανάγεις στη συχνότητα για να δεις πως λειτουργεί το φίλτρο


Title: Re: [ΨΕΣ] Λύσεις παλιών θεμάτων
Post by: makou on February 09, 2014, 16:57:10 pm
Στο 1ο θέμα του Σεπτεμβρη του 12, ειδα και το ποστ που απαντηθηκε πιο πανω αλλα δεν καταλαβαινω γιατι ειναι το 2ο και οχι το 3ο σχημα..


Title: Re: [ΨΕΣ] Λύσεις παλιών θεμάτων
Post by: nohponex on February 09, 2014, 17:08:48 pm
Στο 1ο θέμα του Σεπτεμβρη του 12, ειδα και το ποστ που απαντηθηκε πιο πανω αλλα δεν καταλαβαινω γιατι ειναι το 2ο και οχι το 3ο σχημα..

Το τρίτο δεν είναι σίγουρα επειδή ξεκινάει πριν το 60


Title: Re: [ΨΕΣ] Λύσεις παλιών θεμάτων
Post by: makou on February 09, 2014, 18:53:58 pm
α ναι σωστα δεν το ειχα προσεξει.ευχαριστω..


Title: Re: [ΨΕΣ] Λύσεις παλιών θεμάτων
Post by: soso on June 03, 2014, 18:21:20 pm
Στα θέματα Φεβρουαρίου 2013 στο 2ο εννοεί διακριτό μσ φουριερ? Τοχει λύσει κανεις? Είναι το τελευταίο μου μάθημα σημάτων και τα σιχαίνομαι. Βοηθάτε χωριανοί.


Title: Re: [ΨΕΣ] Λύσεις παλιών θεμάτων
Post by: teslaaaa on June 04, 2014, 15:33:14 pm
Στα θέματα Φεβρουαρίου 2013 στο 2ο εννοεί διακριτό μσ φουριερ? Τοχει λύσει κανεις? Είναι το τελευταίο μου μάθημα σημάτων και τα σιχαίνομαι. Βοηθάτε χωριανοί.
λογικά τον συνεχή εννοεί, γι αυτό και το συμβολίζει Χ(eiw).. άμα γράψεις το sin σαν διαφορά εκθετικών από euler και μετά πάρεις τον τύπο 4.5.1 από πανά σελ,75 λογικά θα μπορεί να εφραστεί ως μτχ  φουριερ  της σειράς αn αλλά μετατοπισμένος στο ω-ω0, ω+ω0? κάπως έτσι..αλλά δν το έχω δει αναλυτικά


Title: Re: [ΨΕΣ] Λύσεις παλιών θεμάτων
Post by: Μικρή Ηλιαχτίδα on September 21, 2014, 17:43:03 pm
Ιούνιος 2014

Καμιά ιδέα για το 3ο θέμα?


Title: Re: [ΨΕΣ] Λύσεις παλιών θεμάτων
Post by: jafou on September 21, 2014, 21:28:31 pm
Ιούνιος 2014

Καμιά ιδέα για το 3ο θέμα?

Το πρώτο είναι κινούμενος μέσος, δηλαδή φίλτρο εξομάλυνσης, οπότε χαμηλοπερατό. Το δεύτερο είναι μια μορφή παραγώγου, άρα υψιπερατό. Όσο μεγαλύτερη η διαφορά μεταξύ δύο διαδοχικών τιμών της x[n] (υψηλή συχνότητα εναλλαγής) τόσο μεγαλύτερη η έξοδος του φίλτρου.


Title: Re: [ΨΕΣ] Λύσεις παλιών θεμάτων
Post by: Μικρή Ηλιαχτίδα on September 21, 2014, 23:05:20 pm
Ιούνιος 2014

Καμιά ιδέα για το 3ο θέμα?

Το πρώτο είναι κινούμενος μέσος, δηλαδή φίλτρο εξομάλυνσης, οπότε χαμηλοπερατό. Το δεύτερο είναι μια μορφή παραγώγου, άρα υψιπερατό. Όσο μεγαλύτερη η διαφορά μεταξύ δύο διαδοχικών τιμών της x[n] (υψηλή συχνότητα εναλλαγής) τόσο μεγαλύτερη η έξοδος του φίλτρου.

Από το βιβλίο του Πανά δε βρίσκω κάτι να με βοηθάει σε αυτό. Υπάρχει κάτι στο βιβλίο του Hayes?


Title: Re: [ΨΕΣ] Λύσεις παλιών θεμάτων
Post by: jafou on September 22, 2014, 11:53:13 am
Ιούνιος 2014

Καμιά ιδέα για το 3ο θέμα?

Το πρώτο είναι κινούμενος μέσος, δηλαδή φίλτρο εξομάλυνσης, οπότε χαμηλοπερατό. Το δεύτερο είναι μια μορφή παραγώγου, άρα υψιπερατό. Όσο μεγαλύτερη η διαφορά μεταξύ δύο διαδοχικών τιμών της x[n] (υψηλή συχνότητα εναλλαγής) τόσο μεγαλύτερη η έξοδος του φίλτρου.

Από το βιβλίο του Πανά δε βρίσκω κάτι να με βοηθάει σε αυτό. Υπάρχει κάτι στο βιβλίο του Hayes?

Δεν ξέρω, εμπειρικά απάντησα


Title: Re: [ΨΕΣ] Λύσεις παλιών θεμάτων
Post by: heavy melon on February 12, 2015, 14:25:00 pm
φλεβ. 2013, θέμα 2ο

δείτε λίγο αν θέλετε αν είναι σωστή η λύση  :D


Title: Re: [ΨΕΣ] Λύσεις παλιών θεμάτων
Post by: Mr K on February 13, 2015, 19:46:50 pm
Φλεβαρης 13 1ο θεμα το εχει κανει καποιος?


Title: Re: [ΨΕΣ] Λύσεις παλιών θεμάτων
Post by: jimPster on February 13, 2015, 20:43:28 pm
Αν εχεις το βιβλιο hayes θα πας σελιδα 306 εκει που λεει συμμετρια για πραγματικο σημα ισχυει:

X(k) = ( X(N-k) )*

δηλ για Ν=8  X(0) = ( X(8 - 0) )*  => Χ(8) = ( Χ(0) )*  ...  κτλ


Title: Re: [ΨΕΣ] Λύσεις παλιών θεμάτων
Post by: jimPster on February 13, 2015, 22:45:24 pm
φλεβ. 2013, θέμα 2ο

δείτε λίγο αν θέλετε αν είναι σωστή η λύση  :D

Ειναι λαθος , υπαρχει ιδια  στο proaki manolaki

Δεν υπαρχει ο μετασχηματισμος . Αν παρεις πχ ω= π/2 το |sinp/2n| = 1 για καθε n
Eτσι αν παρεις τo απολυτο αθροισμα  απο -00 σε +00 του |x(n)| απειριζεται γιατι
για -00 εως -1  (ισχυει -1<a<1) το n γινεται αρνητικο  και αρα το a μεγαλωνει κατα απολυτη τιμη
πχ an a=1/2 τοτε θα γινεται για n=-1 , -2 , -3 => 2 4 8 ... oo

και δες σελ 95 ποτε υπαρχει DTFT hayes


Title: Re: [ΨΕΣ] Λύσεις παλιών θεμάτων
Post by: xameno kormi on February 13, 2015, 23:01:15 pm
proaki manolaki αυτο εννοεις ε ?
http://itl7.elte.hu/~zsolt/Oktatas/editable_Digital_Signal_Processing_Principles_Algorithms_and_Applications_Third_Edition.pdf
τι σελιδα ειναι αυτο το θεμα που λες ?


Title: Re: [ΨΕΣ] Λύσεις παλιών θεμάτων
Post by: jimPster on February 13, 2015, 23:11:01 pm
ασκηση 4.9 e πρεπει ν κατεβασεις το λυσαρι


Title: Re: [ΨΕΣ] Λύσεις παλιών θεμάτων
Post by: LaBomba8 on February 15, 2015, 11:58:05 am
Που υπαρχουν παλια θεματα?


Title: Re: [ΨΕΣ] Λύσεις παλιών θεμάτων
Post by: nohponex on February 15, 2015, 13:34:52 pm
Που υπαρχουν παλια θεματα?
Στα downloads
https://www.thmmy.gr/smf/index.php?action=tpmod;dl=cat99


Title: Re: [ΨΕΣ] Λύσεις παλιών θεμάτων
Post by: Kodi on February 15, 2015, 14:17:11 pm
Ιουνίος 2014 - Θέμα 3

πως μπορούμε να προσδιορίσουμε αν είναι χρονοπερατό, υψιπερατό ή ζωνοπερατό το φίλτρο;

εδιτ γιατί είναι πιο ευανάγνωστο με επικεφαλίδα :P


Title: Re: [ΨΕΣ] Λύσεις παλιών θεμάτων
Post by: jimPster on February 15, 2015, 15:21:29 pm
Αναφερει και αλλο παιδι εξηγηση πιο πανω. Αν ειχαμε την αποκριση συχνοτητας
θα λεγαμε απο πολους και μηδενικα γιατι. Τωρα,   αυτα που λεει το παιδι πιο
πανω ισχυουν.Σου  ζηταει το θεμα να αιτιολογησεις: Θα μπορουσες να πεις
επιπλεον  πχ το σημα { ......... -1,1,-1,1,-1,1 .....) = (-1)^n =  cosnp
ειναι σημα που εχει μεγιστη δυνατη συχνοτητα ( εναλλασσεται σε καθε δειγμα)
για το (x(n)+x(n-1))/2 = 0 δηλ δν το περναει το σημα γιατι ειναι υψηλης συχνοτητας

για (x(n) - x(n-1)/2 = +/- 1 δηλ το περναει το σημα που ειναι υψηλης συχνοτητας


Πηρα δηλ ενα ακραιο σημα μεγιστης συχνοτητας και εδειξα οτι το ενα το περναει ολο
(υψιπερατο ) και το αλλο τπτ (χαμηλοπερατο)

Αναλογα θα μπορουσες να παρεις (.... 1,1,1,1,,1,1,,1,1..) μηδενικης συχνοτητας
ακραιο απο την αλλη μερια και θα δεις να αναστρεφονται οι ρολοι


Title: Re: [ΨΕΣ] Λύσεις παλιών θεμάτων
Post by: Αλντεμπαράν on February 16, 2015, 01:16:38 am
Ιουνίους 2014 - Θέμα 1

Το λύσαμε στην τάξη και είπαμε ότι για n=7,5 δεν μπορώ να βρω λύση οπότε πάω στα συμμετρικά τους δλδ n=-1,-3. Γιατί δεν παίρνουμε ότι απλά εκείνοι οι συντελεστές είναι μηδέν, δηλαδή x[5]=x[7]=0 ;


Title: Re: [ΨΕΣ] Λύσεις παλιών θεμάτων
Post by: Exomag on February 16, 2015, 02:05:38 am
Ιουνίους 2014 - Θέμα 1

Το λύσαμε στην τάξη και είπαμε ότι για n=7,5 δεν μπορώ να βρω λύση οπότε πάω στα συμμετρικά τους δλδ n=-1,-3. Γιατί δεν παίρνουμε ότι απλά εκείνοι οι συντελεστές είναι μηδέν, δηλαδή x[5]=x[7]=0 ;


Παρατήρησε, απλά, πως ισχύουν τα εξής:

(http://latex.codecogs.com/gif.download?%5Cdpi%7B120%7D%20e%5E%7Bj%5Cfrac%7Bk%5Cpi%7D%7B4%7D%7D%3De%5E%7Bj%28%5Cfrac%7Bk%5Cpi%7D%7B4%7D-2%5Cpi%20k%29%7D%3De%5E%7B-j%5Cfrac%7B7k%5Cpi%7D%7B4%7D%7D)

(http://latex.codecogs.com/gif.download?%5Cdpi%7B120%7D%20e%5E%7Bj%5Cfrac%7B3k%5Cpi%7D%7B4%7D%7D%3De%5E%7Bj%28%5Cfrac%7B3k%5Cpi%7D%7B4%7D-2%5Cpi%20k%29%7D%3De%5E%7B-j%5Cfrac%7B5k%5Cpi%7D%7B4%7D%7D)


Title: Re: [ΨΕΣ] Λύσεις παλιών θεμάτων
Post by: Αλντεμπαράν on February 16, 2015, 11:35:08 am
Σεπτέμβρης 2012 - Θέμα 2

βρίσκω

(http://latex.codecogs.com/gif.latex?H%28z%29%20%3D%20%5Cfrac%7Bz%5E4%20-1%7D%7B1&plus;jz%5E-1%7D)

πώς μετά λύνω το (β) ?


Title: Re: [ΨΕΣ] Λύσεις παλιών θεμάτων
Post by: Earendil on February 16, 2015, 12:15:53 pm
Σεπτέμβρης 2012 - Θέμα 2

βρίσκω

(http://latex.codecogs.com/gif.latex?H%28z%29%20%3D%20%5Cfrac%7Bz%5E4%20-1%7D%7B1&plus;jz%5E-1%7D)

πώς μετά λύνω το (β) ?

αν αντικαταστησεις στη x[n] βλεπεις οτι x[0] = 10, x[1] = 1 -3j, ... x[4] = 10,
δηλαδη η x[n] επαναλαμβανεται με περιοδο T = 4, αρα x[n] = x[n - 4], αρα y[n] = jy[n-1]


Title: Re: [ΨΕΣ] Λύσεις παλιών θεμάτων
Post by: Kodi on February 16, 2015, 17:26:56 pm
Έχει κανεις καμια ιδέα για το 4ο θέμα του Γενάρη του 2012;;


Title: Re: [ΨΕΣ] Λύσεις παλιών θεμάτων
Post by: xameno kormi on February 16, 2015, 17:45:02 pm
Ιουνίους 2014 - Θέμα 1

Το λύσαμε στην τάξη και είπαμε ότι για n=7,5 δεν μπορώ να βρω λύση οπότε πάω στα συμμετρικά τους δλδ n=-1,-3. Γιατί δεν παίρνουμε ότι απλά εκείνοι οι συντελεστές είναι μηδέν, δηλαδή x[5]=x[7]=0 ;


μπορεις να το ανεβασεις ? η να πεις περιπου πως το λυσατε ? γραφω cos και sin σαν διαφορα εκθετικων euler και μετα τι κανω ?


Title: Re: [ΨΕΣ] Λύσεις παλιών θεμάτων
Post by: Exomag on February 16, 2015, 17:59:25 pm
Ιουνίους 2014 - Θέμα 1

Το λύσαμε στην τάξη και είπαμε ότι για n=7,5 δεν μπορώ να βρω λύση οπότε πάω στα συμμετρικά τους δλδ n=-1,-3. Γιατί δεν παίρνουμε ότι απλά εκείνοι οι συντελεστές είναι μηδέν, δηλαδή x[5]=x[7]=0 ;


μπορεις να το ανεβασεις ? η να πεις περιπου πως το λυσατε ? γραφω cos και sin σαν διαφορα εκθετικων euler και μετα τι κανω ?

Αυτό είναι σαν trick, κάπως. Ο κανονικός τρόπος είναι απλά να πάρεις την παρακάτω σχέση:

(http://latex.codecogs.com/gif.download?%5Cdpi%7B120%7D%20%5Clarge%20%5Ctilde%7Bx%7D%5Bn%5D%3D%5Cfrac%7B1%7D%7BN%7D%5Csum_%7Bk%3D0%7D%5E%7BN-1%7D%5Ctilde%7BX%7D%5Bk%5De%5E%7Bj%5Cfrac%7B2%5Cpi%20k%7D%7BN%7Dn%7D)


Title: Re: [ΨΕΣ] Λύσεις παλιών θεμάτων
Post by: et3rn1ty on February 16, 2015, 19:13:29 pm
Ιουνίους 2014 - Θέμα 1

Το λύσαμε στην τάξη και είπαμε ότι για n=7,5 δεν μπορώ να βρω λύση οπότε πάω στα συμμετρικά τους δλδ n=-1,-3. Γιατί δεν παίρνουμε ότι απλά εκείνοι οι συντελεστές είναι μηδέν, δηλαδή x[5]=x[7]=0 ;


Γιατί δεν μπορούμε να υπολογίσουμε λύση για 5 και 7? Αν αντικαταστήσουμε όπου n=5 στο άθροισμα δεν βγαίνει;

Επίσης, όταν λέει αναλυτικά δεν θέλει να του δώσουμε τύπο? Αν του δώσουμε δηλαδή την ακολουθία με αριθμούς το θεωρεί σωστό


Title: Re: [ΨΕΣ] Λύσεις παλιών θεμάτων
Post by: Exomag on February 16, 2015, 19:49:48 pm
Ιουνίους 2014 - Θέμα 1
Το λύσαμε στην τάξη και είπαμε ότι για n=7,5 δεν μπορώ να βρω λύση οπότε πάω στα συμμετρικά τους δλδ n=-1,-3. Γιατί δεν παίρνουμε ότι απλά εκείνοι οι συντελεστές είναι μηδέν, δηλαδή x[5]=x[7]=0 ;
Γιατί δεν μπορούμε να υπολογίσουμε λύση για 5 και 7? Αν αντικαταστήσουμε όπου n=5 στο άθροισμα δεν βγαίνει;

Ιουνίους 2014 - Θέμα 1

Το λύσαμε στην τάξη και είπαμε ότι για n=7,5 δεν μπορώ να βρω λύση οπότε πάω στα συμμετρικά τους δλδ n=-1,-3. Γιατί δεν παίρνουμε ότι απλά εκείνοι οι συντελεστές είναι μηδέν, δηλαδή x[5]=x[7]=0 ;


μπορεις να το ανεβασεις ? η να πεις περιπου πως το λυσατε ? γραφω cos και sin σαν διαφορα εκθετικων euler και μετα τι κανω ?

Αυτό είναι σαν trick, κάπως. Ο κανονικός τρόπος είναι απλά να πάρεις την παρακάτω σχέση:

(http://latex.codecogs.com/gif.download?%5Cdpi%7B120%7D%20%5Clarge%20%5Ctilde%7Bx%7D%5Bn%5D%3D%5Cfrac%7B1%7D%7BN%7D%5Csum_%7Bk%3D0%7D%5E%7BN-1%7D%5Ctilde%7BX%7D%5Bk%5De%5E%7Bj%5Cfrac%7B2%5Cpi%20k%7D%7BN%7Dn%7D)



Ιουνίους 2014 - Θέμα 1
Το λύσαμε στην τάξη και είπαμε ότι για n=7,5 δεν μπορώ να βρω λύση οπότε πάω στα συμμετρικά τους δλδ n=-1,-3. Γιατί δεν παίρνουμε ότι απλά εκείνοι οι συντελεστές είναι μηδέν, δηλαδή x[5]=x[7]=0 ;

Επίσης, όταν λέει αναλυτικά δεν θέλει να του δώσουμε τύπο? Αν του δώσουμε δηλαδή την ακολουθία με αριθμούς το θεωρεί σωστό
Μπορείς και αναλυτικά, αλλιώς αν έχεις βρει μεμονομένες τις τιμές μπορείς να πολλαπλασιάσεις αυτές τις τιμές με τις αντίστοιχες μετατοπισμένες


Title: Re: [ΨΕΣ] Λύσεις παλιών θεμάτων
Post by: nastia on February 16, 2015, 20:10:10 pm
Φεβρουάριος 2013, θεμα 4

Στο α ερώτημα έχω βρει την Η(z) συναρτήσει των H1,H2 κλπ...αλλά μετά έχω μπερδευτεί στο πως να κάνω τον IZT... :-\
καμιά βοήθεια?


Title: Re: [ΨΕΣ] Λύσεις παλιών θεμάτων
Post by: Dealan on February 16, 2015, 20:16:42 pm
Δεν χρειάζεται να χρησιμοποιήσεις H(z). Οι συναρτήσεις είναι απλές, απλά κάνεις συνέλιξη και βγαίνουν.

(Αν λέω μαλακία πείτε γιατί δεν το έχω με τα μπλοκ ακόμα.)


Title: Re: [ΨΕΣ] Λύσεις παλιών θεμάτων
Post by: gmtms on February 16, 2015, 20:24:17 pm
Φεβ 2013 θέμα 1 τι κάνουμε;
brute force 10x10 σύστημα και καλή τύχη;


Title: Re: [ΨΕΣ] Λύσεις παλιών θεμάτων
Post by: Exomag on February 16, 2015, 20:32:58 pm
Φεβ 2013 θέμα 1 τι κάνουμε;
brute force 10x10 σύστημα και καλή τύχη;

Χρησιμοποιείς την εξής ιδιότητα:

(http://latex.codecogs.com/gif.download?%5Cdpi%7B120%7D%20%5Clarge%20x%5Bn%5D%5Cin%20%5Cmathbb%7BR%7D%5CRightarrow%20X%5B-k%5D%3DX%5E%7B*%7D%5Bk%5D)



Title: Re: [ΨΕΣ] Λύσεις παλιών θεμάτων
Post by: nastia on February 16, 2015, 20:37:07 pm
Δεν χρειάζεται να χρησιμοποιήσεις H(z). Οι συναρτήσεις είναι απλές, απλά κάνεις συνέλιξη και βγαίνουν.

(Αν λέω μαλακία πείτε γιατί δεν το έχω με τα μπλοκ ακόμα.)

Παίρνω δηλαδή τον τύπο της συνέλιξης με το άθροισμα?
Δεν θα έχω 2 συναρτήσεις να κάνω συνέλιξη όμως,γι'αυτό έχω μπερδευτεί.


Title: Re: [ΨΕΣ] Λύσεις παλιών θεμάτων
Post by: Dealan on February 16, 2015, 20:42:49 pm
Ισχύει με βάση το σχήμα ότι h[n] = h1*h2 - h1*h3*h4, όπου * η συνέλιξη.

Επίσης ισχύει h1*h3*h4 = (h1*h3)*h4.

Άρα πράξεις.


Title: Re: [ΨΕΣ] Λύσεις παλιών θεμάτων
Post by: gmtms on February 16, 2015, 20:46:31 pm
Χρησιμοποιείς την εξής ιδιότητα:

(http://latex.codecogs.com/gif.download?%5Cdpi%7B120%7D%20%5Clarge%20x%5Bn%5D%5Cin%20%5Cmathbb%7BR%7D%5CRightarrow%20X%5B-k%5D%3DX%5E%7B*%7D%5Bk%5D)

μα εχω Χ( 1),  Χ( 2 ) ..  Χ( 5 )
πώς θα με βοηθήσει η ιδιότητα αυτή;


Title: Re: [ΨΕΣ] Λύσεις παλιών θεμάτων
Post by: Exomag on February 16, 2015, 20:48:01 pm
Χρησιμοποιείς την εξής ιδιότητα:

(http://latex.codecogs.com/gif.download?%5Cdpi%7B120%7D%20%5Clarge%20x%5Bn%5D%5Cin%20%5Cmathbb%7BR%7D%5CRightarrow%20X%5B-k%5D%3DX%5E%7B*%7D%5Bk%5D)

μα εχω Χ[1], Χ[2]..


Παρατήρησε, απλά, πως ισχύουν τα εξής:

(http://latex.codecogs.com/gif.download?%5Cdpi%7B120%7D%20e%5E%7Bj%5Cfrac%7Bk%5Cpi%7D%7B4%7D%7D%3De%5E%7Bj%28%5Cfrac%7Bk%5Cpi%7D%7B4%7D-2%5Cpi%20k%29%7D%3De%5E%7B-j%5Cfrac%7B7k%5Cpi%7D%7B4%7D%7D)

(http://latex.codecogs.com/gif.download?%5Cdpi%7B120%7D%20e%5E%7Bj%5Cfrac%7B3k%5Cpi%7D%7B4%7D%7D%3De%5E%7Bj%28%5Cfrac%7B3k%5Cpi%7D%7B4%7D-2%5Cpi%20k%29%7D%3De%5E%7B-j%5Cfrac%7B5k%5Cpi%7D%7B4%7D%7D)


Title: Re: [ΨΕΣ] Λύσεις παλιών θεμάτων
Post by: gmtms on February 16, 2015, 20:54:08 pm
δε μπορώ να ακολουθήσω
το e(-jx) είναι το συζυγές του e^jx, οκ
δεν είναι όμως ο μόνος όρος του X(k) και μου δίνει πληροφορία μόνο για τη φάση, όχι για το μέτρο
πρέπει αυτό να το εφαρμόσω ας πούμε στον κάθε όρο Χ(κ) για να απλοποιήσω το σύστημα;


Title: Re: [ΨΕΣ] Λύσεις παλιών θεμάτων
Post by: Exomag on February 16, 2015, 20:55:15 pm
δε μπορώ να ακολουθήσω
το e(-jx) είναι το συζυγές του e^jx, οκ
δεν είναι όμως ο μόνος όρος του X(k) και μου δίνει πληροφορία μόνο για τη φάση, όχι για το μέτρο
πρέπει αυτό να το εφαρμόσω ας πούμε στον κάθε όρο Χ(κ) για να απλοποιήσω το σύστημα;

Θα γράψεις τα sin/cos ως άθροισμα εκθετικών και θα εξισώσεις αυτό που θα βγει με τον τύπο ορισμού της Σειράς Fourier. Και θα χρησιμοποιήσεις και τις δύο ιδιότητες/σχέσεις που έγραψα.


Title: Re: [ΨΕΣ] Λύσεις παλιών θεμάτων
Post by: xameno kormi on February 16, 2015, 20:56:30 pm
ισχυει και το εξης ?
Χ*(Ν-k) = X*(-k)
στις πιο πισω σελιδες το ειχαν γραψει


Title: Re: [ΨΕΣ] Λύσεις παλιών θεμάτων
Post by: jimPster on February 16, 2015, 20:57:29 pm
ρε παιδια εχει απαντη8ει 3 φορες στο ιδιο thread . ελεος

και εχεις x(0), x(1), .. x(4) και οχι χ(1) .. χ(5) 

και για να γινει πιο λιανα  x(n) πραγματικη -> Χ[-κ]=(Χ[κ])*

το Χ[Κ] και το καθε Χ[κ] εχει περιοδο Ν! δηλ. Χ[κ] = Χ[Ν+κ]

Αρα Χ[-κ +Ν]= (Χ[κ+Ν])*  = (Χ[κ])*


Title: Re: [ΨΕΣ] Λύσεις παλιών θεμάτων
Post by: Exomag on February 16, 2015, 20:59:11 pm
ρε παιδια εχει απαντη8ει 3 φορες στο ιδιο thread . ελεος

και εχεις x(0), x(1), .. x(4) και οχι χ(1) .. χ(5) 

that


Title: Re: [ΨΕΣ] Λύσεις παλιών θεμάτων
Post by: LaBomba8 on February 16, 2015, 21:37:15 pm
Ianouarios 14 thema 1 to b?

p.s gia to paidi pou rwtaei prin, an blepete oti den katalabainei kapoios apla peite tin apantisi oso pio analutika boreite, min petate apla ena tipo

p.s. Μην γράφεις σε greeklish άλλαξε τα παρακαλώ.


Title: Re: [ΨΕΣ] Λύσεις παλιών θεμάτων
Post by: nastia on February 16, 2015, 21:41:49 pm
Ισχύει με βάση το σχήμα ότι h[n] = h1*h2 - h1*h3*h4, όπου * η συνέλιξη.

Επίσης ισχύει h1*h3*h4 = (h1*h3)*h4.

Άρα πράξεις.

Δε γνωρίζω τίποτα όμως για αυτές τις συναρτήσεις για να αναπτύξω κάπως το άθροισμα!
Μήπως ήθελε να γράψεις απλά αυτό και στο β να κάνεις πράξεις? :P


Title: Re: [ΨΕΣ] Λύσεις παλιών θεμάτων
Post by: Exomag on February 16, 2015, 21:49:08 pm
Ισχύει με βάση το σχήμα ότι h[n] = h1*h2 - h1*h3*h4, όπου * η συνέλιξη.

Επίσης ισχύει h1*h3*h4 = (h1*h3)*h4.

Άρα πράξεις.

Δε γνωρίζω τίποτα όμως για αυτές τις συναρτήσεις για να αναπτύξω κάπως το άθροισμα!
Μήπως ήθελε να γράψεις απλά αυτό και στο β να κάνεις πράξεις? :P

Για το (α) ήθελε απλά να γράψεις αυτό. Στο (β) είναι οι πράξεις.


Title: Re: [ΨΕΣ] Λύσεις παλιών θεμάτων
Post by: Exomag on February 16, 2015, 21:50:50 pm
Ianouarios 14 thema 1 to b?

Μήπως εννοείς άλλη χρονιά/θέμα; Γιατί δεν υπάρχουν θέματα Ιανουαρίου το 2014.


Title: Re: [ΨΕΣ] Λύσεις παλιών θεμάτων
Post by: nastia on February 16, 2015, 21:52:09 pm
Ισχύει με βάση το σχήμα ότι h[n] = h1*h2 - h1*h3*h4, όπου * η συνέλιξη.

Επίσης ισχύει h1*h3*h4 = (h1*h3)*h4.

Άρα πράξεις.

Δε γνωρίζω τίποτα όμως για αυτές τις συναρτήσεις για να αναπτύξω κάπως το άθροισμα!
Μήπως ήθελε να γράψεις απλά αυτό και στο β να κάνεις πράξεις? :P

Για το (α) ήθελε απλά να γράψεις αυτό. Στο (β) είναι οι πράξεις.

Α στο καλό και σπάω το κεφάλι μου τόση ώρα πως να κάνω τις συνελίξεις!
Ευχαριστώ!


Title: Re: [ΨΕΣ] Λύσεις παλιών θεμάτων
Post by: LaBomba8 on February 16, 2015, 21:52:47 pm
*Ιουνίου! Και β) εννοώ το x2


Title: Re: [ΨΕΣ] Λύσεις παλιών θεμάτων
Post by: jimPster on February 16, 2015, 21:54:17 pm
Έχει κανεις καμια ιδέα για το 4ο θέμα του Γενάρη του 2012;;

Υπαρχει λυμενο στο hayes sel 108

2 λυση:

επειδη δν ζηταει το h(n)  μπορεις ν το κανεις και "εμπειρικα"

(-1)^n = cosnp
x(n)cosnp -> 1/2( X(ω-p) + X(ω + p) ) δηλ. αρχικα μετατοπιζει το φασμα κατα p , (το κεντρο του τριγωνου ειναι στα  +/- p κτλ.

Εδω πρεπει να τονισω τα νεα τριγωνα δεν εχουν υψος 1/2 αλλα 1 γιατι το σημα στη συχνοτητα ειναι περιοδικο με 2p περιοδο
(αν βαλεις τιμες θα το επιβεβαιωσεις)


στη συνεχεια περναει στο  χαμηλοπερατο παιρνεις απο το σημα απο -p/4 εως  p/4

τελος παλι πολλαπλασιαζεται με cosnp αρα το νεο σημα μετατοπιζεται παλι με κεντρο +/-p

Τελικα σου χει μεινει απο 3p/4 - p και -3π/4 εως -π απο το αρχικο σημα


Αρα  συχνοτητα αποκοπης 3π/4 το y(n) ειναι το ιδιο με το x(n) αλλα παιρνεις το κομματι
που ανεφερα πιο πανω. Η λειτουργια του φιλτρου ειναι οτι ειναι υψιπερατο περναει απο 3π/4 εως
π . Η διαφορα με hayes ειναι οτι με αυτη τη μεθοδο μπορεις να περιγραψεις ολα τα σταδια του
φιλτρου.

Το πρωτο σταδιο παιρνει το περιεχομενο των υψηλων συχνοτητων και το βαζει στις χαμηλες

ετσι το χαμηλοπερατο περναει ουσιαστικα τις υψηλες και τριτο σταδιο ξαναπαει το περιεχομενο
στις σωστες συχνοτητες (υψηλες).



Title: Re: [ΨΕΣ] Λύσεις παλιών θεμάτων
Post by: xameno kormi on February 16, 2015, 21:56:56 pm
Ιουνίους 2014 - Θέμα 1
Το λύσαμε στην τάξη και είπαμε ότι για n=7,5 δεν μπορώ να βρω λύση οπότε πάω στα συμμετρικά τους δλδ n=-1,-3. Γιατί δεν παίρνουμε ότι απλά εκείνοι οι συντελεστές είναι μηδέν, δηλαδή x[5]=x[7]=0 ;

δεν μπορω να καταλαβω γιατι στα n=5,7 δεν μπορω να βρω λυση μπορει καποιος να το εξηγησει ?


Title: Re: [ΨΕΣ] Λύσεις παλιών θεμάτων
Post by: jimPster on February 16, 2015, 22:07:20 pm
Ολος ο ΦΕΒΡΟΥΑΡΙΟΣ 2013

google proakis manolakis solution manual το δευτερο link  ( 4η εκδοση)

θεμα 1 (7.1)
θεμα 2 (4.9 e)
θεμα 3 (3.6, 3.19)
θεμα 4 (2.35)


Title: Re: [ΨΕΣ] Λύσεις παλιών θεμάτων
Post by: jimPster on February 16, 2015, 22:19:33 pm
ΙΟΥΝΙΟΣ 2014 ΘΕΜΑ 1

proakis manolakis παλι

4.7 a,b


Title: Re: [ΨΕΣ] Λύσεις παλιών θεμάτων
Post by: Solaufein on February 16, 2015, 23:33:45 pm
Θέμα 4ο feb 13, (2.35 στον proakis) απο πού καταλήγει στο h2(n)-h3(n)*h4(n)= 2u(n)- δ(n)?
Δε θα έπρεπε να βγαίνει =2u(n-2)?


Title: Re: [ΨΕΣ] Λύσεις παλιών θεμάτων
Post by: LaBomba8 on February 17, 2015, 01:26:17 am
το δ(n) δεν μπορώ να καταλάβω πως το βγάζει


Title: Re: [ΨΕΣ] Λύσεις παλιών θεμάτων
Post by: chrisrn on February 17, 2015, 01:58:47 am
ianouarios tou 12 thema 2 kamia idea?


Title: Re: [ΨΕΣ] Λύσεις παλιών θεμάτων
Post by: Σαλτιμπάγκος on February 17, 2015, 02:10:35 am
Στο 3ο του Ιουν 14 :
Το 1ο ειναι υψιπερατο και το δευτερο ζωνοπερατο;


Title: Re: [ΨΕΣ] Λύσεις παλιών θεμάτων
Post by: jimPster on February 17, 2015, 02:15:57 am
Θέμα 4ο feb 13, (2.35 στον proakis) απο πού καταλήγει στο h2(n)-h3(n)*h4(n)= 2u(n)- δ(n)?
Δε θα έπρεπε να βγαίνει =2u(n-2)?

 h2(n)-h3(n)*h4(n) = (n+1)u(n) - (n-1)u(n-2)

n=0 ->   1 - 0 = 1
n=1 ->   2 - 0 = 2
n=2 ->   3 - 1 = 2
n=3 ->   4 - 2 = 2
n=4 ->   5 - 3 = 2
n=k ->   k+1 - (k-1) = 2     

Αρα  ? 


Title: Re: [ΨΕΣ] Λύσεις παλιών θεμάτων
Post by: LaBomba8 on February 17, 2015, 10:32:52 am
Άρα άψογος!


Title: Re: [ΨΕΣ] Λύσεις παλιών θεμάτων
Post by: Dealan on February 17, 2015, 11:04:26 am
Έχει κανεις καμια ιδέα για το 4ο θέμα του Γενάρη του 2012;;

Υπαρχει λυμενο στο hayes sel 108

2 λυση:

επειδη δν ζηταει το h(n)  μπορεις ν το κανεις και "εμπειρικα"

(-1)^n = cosnp
x(n)cosnp -> 1/2( X(ω-p) + X(ω + p) ) δηλ. αρχικα μετατοπιζει το φασμα κατα p , (το κεντρο του τριγωνου ειναι στα  +/- p κτλ.

Εδω πρεπει να τονισω τα νεα τριγωνα δεν εχουν υψος 1/2 αλλα 1 γιατι το σημα στη συχνοτητα ειναι περιοδικο με 2p περιοδο
(αν βαλεις τιμες θα το επιβεβαιωσεις)


στη συνεχεια περναει στο  χαμηλοπερατο παιρνεις απο το σημα απο -p/4 εως  p/4

τελος παλι πολλαπλασιαζεται με cosnp αρα το νεο σημα μετατοπιζεται παλι με κεντρο +/-p

Τελικα σου χει μεινει απο 3p/4 - p και -3π/4 εως -π απο το αρχικο σημα


Αρα  συχνοτητα αποκοπης 3π/4 το y(n) ειναι το ιδιο με το x(n) αλλα παιρνεις το κομματι
που ανεφερα πιο πανω. Η λειτουργια του φιλτρου ειναι οτι ειναι υψιπερατο περναει απο 3π/4 εως
π . Η διαφορα με hayes ειναι οτι με αυτη τη μεθοδο μπορεις να περιγραψεις ολα τα σταδια του
φιλτρου.

Το πρωτο σταδιο παιρνει το περιεχομενο των υψηλων συχνοτητων και το βαζει στις χαμηλες

ετσι το χαμηλοπερατο περναει ουσιαστικα τις υψηλες και τριτο σταδιο ξαναπαει το περιεχομενο
στις σωστες συχνοτητες (υψηλες).



Κάπως αργά ρωτάω αλλά στο bold κομμάτι γιατί δεν έχουμε από 3π/4 ως 5π/4 και -5π/4 ως -3π/4;


Title: Re: [ΨΕΣ] Λύσεις παλιών θεμάτων
Post by: Exomag on February 17, 2015, 11:09:20 am
Έχει κανεις καμια ιδέα για το 4ο θέμα του Γενάρη του 2012;;

Υπαρχει λυμενο στο hayes sel 108

2 λυση:

επειδη δν ζηταει το h(n)  μπορεις ν το κανεις και "εμπειρικα"

(-1)^n = cosnp
x(n)cosnp -> 1/2( X(ω-p) + X(ω + p) ) δηλ. αρχικα μετατοπιζει το φασμα κατα p , (το κεντρο του τριγωνου ειναι στα  +/- p κτλ.

Εδω πρεπει να τονισω τα νεα τριγωνα δεν εχουν υψος 1/2 αλλα 1 γιατι το σημα στη συχνοτητα ειναι περιοδικο με 2p περιοδο
(αν βαλεις τιμες θα το επιβεβαιωσεις)


στη συνεχεια περναει στο  χαμηλοπερατο παιρνεις απο το σημα απο -p/4 εως  p/4

τελος παλι πολλαπλασιαζεται με cosnp αρα το νεο σημα μετατοπιζεται παλι με κεντρο +/-p

Τελικα σου χει μεινει απο 3p/4 - p και -3π/4 εως -π απο το αρχικο σημα


Αρα  συχνοτητα αποκοπης 3π/4 το y(n) ειναι το ιδιο με το x(n) αλλα παιρνεις το κομματι
που ανεφερα πιο πανω. Η λειτουργια του φιλτρου ειναι οτι ειναι υψιπερατο περναει απο 3π/4 εως
π . Η διαφορα με hayes ειναι οτι με αυτη τη μεθοδο μπορεις να περιγραψεις ολα τα σταδια του
φιλτρου.

Το πρωτο σταδιο παιρνει το περιεχομενο των υψηλων συχνοτητων και το βαζει στις χαμηλες

ετσι το χαμηλοπερατο περναει ουσιαστικα τις υψηλες και τριτο σταδιο ξαναπαει το περιεχομενο
στις σωστες συχνοτητες (υψηλες).



Κάπως αργά ρωτάω αλλά στο bold κομμάτι γιατί δεν έχουμε από 3π/4 ως 5π/4 και -5π/4 ως -3π/4;

Γιατί το ιδανικό χαμηλοδιαβατό φίλτρο που έχεις στο κέντρο του συστήματος κρατάει μόνο τις συχνότητες από -π/4 έως 0 και από 0 έως π/4. Δηλαδή από τα δύο τρίγωνα (που είναι από -2π έως 0 και από 0 έως 2π), κρατάει εύρος π/4 από το καθένα.


Title: Re: [ΨΕΣ] Λύσεις παλιών θεμάτων
Post by: robaudelaire on July 04, 2015, 20:35:34 pm
Αναφερει και αλλο παιδι εξηγηση πιο πανω. Αν ειχαμε την αποκριση συχνοτητας
θα λεγαμε απο πολους και μηδενικα γιατι. Τωρα,   αυτα που λεει το παιδι πιο
πανω ισχυουν.Σου  ζηταει το θεμα να αιτιολογησεις: Θα μπορουσες να πεις
επιπλεον  πχ το σημα { ......... -1,1,-1,1,-1,1 .....) = (-1)^n =  cosnp
ειναι σημα που εχει μεγιστη δυνατη συχνοτητα ( εναλλασσεται σε καθε δειγμα)
για το (x(n)+x(n-1))/2 = 0 δηλ δν το περναει το σημα γιατι ειναι υψηλης συχνοτητας

για (x(n) - x(n-1)/2 = +/- 1 δηλ το περναει το σημα που ειναι υψηλης συχνοτητας


Πηρα δηλ ενα ακραιο σημα μεγιστης συχνοτητας και εδειξα οτι το ενα το περναει ολο
(υψιπερατο ) και το αλλο τπτ (χαμηλοπερατο)

Αναλογα θα μπορουσες να παρεις (.... 1,1,1,1,,1,1,,1,1..) μηδενικης συχνοτητας
ακραιο απο την αλλη μερια και θα δεις να αναστρεφονται οι ρολοι

εκανα ενα quote πριν αλλα ειχα λαθος . οπως τα λες ειναι , το πρωτο lowpass και το δευτερο highpass . Παρολα αυτά εγω δεν θα το απαντουσα ετσι αλλα καθως μου δινει εξισωση διαφορας βρισκω τις H(z) και απανταω με βαση τα μηδενικα καθώς δεν εχω πολους :/


Title: Re: [ΨΕΣ] Λύσεις παλιών θεμάτων
Post by: robaudelaire on July 04, 2015, 20:36:48 pm
επισης το θεμα 4 του ιουνιου 2014 το εχει λυσει καποιος ?


Title: Re: [ΨΕΣ] Λύσεις παλιών θεμάτων
Post by: jimPster on July 04, 2015, 20:54:23 pm
Αναφερει και αλλο παιδι εξηγηση πιο πανω. Αν ειχαμε την αποκριση συχνοτητας
θα λεγαμε απο πολους και μηδενικα γιατι. Τωρα,   αυτα που λεει το παιδι πιο
πανω ισχυουν.Σου  ζηταει το θεμα να αιτιολογησεις: Θα μπορουσες να πεις
επιπλεον  πχ το σημα { ......... -1,1,-1,1,-1,1 .....) = (-1)^n =  cosnp
ειναι σημα που εχει μεγιστη δυνατη συχνοτητα ( εναλλασσεται σε καθε δειγμα)
για το (x(n)+x(n-1))/2 = 0 δηλ δν το περναει το σημα γιατι ειναι υψηλης συχνοτητας

για (x(n) - x(n-1)/2 = +/- 1 δηλ το περναει το σημα που ειναι υψηλης συχνοτητας


Πηρα δηλ ενα ακραιο σημα μεγιστης συχνοτητας και εδειξα οτι το ενα το περναει ολο
(υψιπερατο ) και το αλλο τπτ (χαμηλοπερατο)

Αναλογα θα μπορουσες να παρεις (.... 1,1,1,1,,1,1,,1,1..) μηδενικης συχνοτητας
ακραιο απο την αλλη μερια και θα δεις να αναστρεφονται οι ρολοι

εκανα ενα quote πριν αλλα ειχα λαθος . οπως τα λες ειναι , το πρωτο lowpass και το δευτερο highpass . Παρολα αυτά εγω δεν θα το απαντουσα ετσι αλλα καθως μου δινει εξισωση διαφορας βρισκω τις H(z) και απανταω με βαση τα μηδενικα καθώς δεν εχω πολους :/

σου ζηταει βεβαια ρητα να μν το βρεις με συναρτησεις συστηματων


Title: Re: [ΨΕΣ] Λύσεις παλιών θεμάτων
Post by: robaudelaire on July 04, 2015, 21:06:46 pm
ουπς ναι  ::)
οποτε οπως τα πες


Title: Re: [ΨΕΣ] Λύσεις παλιών θεμάτων
Post by: unsorted on July 04, 2015, 21:37:47 pm
Αναφερει και αλλο παιδι εξηγηση πιο πανω. Αν ειχαμε την αποκριση συχνοτητας
θα λεγαμε απο πολους και μηδενικα γιατι. Τωρα,   αυτα που λεει το παιδι πιο
πανω ισχυουν.Σου  ζηταει το θεμα να αιτιολογησεις: Θα μπορουσες να πεις
επιπλεον  πχ το σημα { ......... -1,1,-1,1,-1,1 .....) = (-1)^n =  cosnp
ειναι σημα που εχει μεγιστη δυνατη συχνοτητα ( εναλλασσεται σε καθε δειγμα)
για το (x(n)+x(n-1))/2 = 0 δηλ δν το περναει το σημα γιατι ειναι υψηλης συχνοτητας

για (x(n) - x(n-1)/2 = +/- 1 δηλ το περναει το σημα που ειναι υψηλης συχνοτητας


Πηρα δηλ ενα ακραιο σημα μεγιστης συχνοτητας και εδειξα οτι το ενα το περναει ολο
(υψιπερατο ) και το αλλο τπτ (χαμηλοπερατο)

Αναλογα θα μπορουσες να παρεις (.... 1,1,1,1,,1,1,,1,1..) μηδενικης συχνοτητας
ακραιο απο την αλλη μερια και θα δεις να αναστρεφονται οι ρολοι

εκανα ενα quote πριν αλλα ειχα λαθος . οπως τα λες ειναι , το πρωτο lowpass και το δευτερο highpass . Παρολα αυτά εγω δεν θα το απαντουσα ετσι αλλα καθως μου δινει εξισωση διαφορας βρισκω τις H(z) και απανταω με βαση τα μηδενικα καθώς δεν εχω πολους :/

σου ζηταει βεβαια ρητα να μν το βρεις με συναρτησεις συστηματων

robaudelaire ακου τον jimPster που στα εξηγει ωραια....   Ειναι Αλεφαντος ο τυπος..!


Title: Re: [ΨΕΣ] Λύσεις παλιών θεμάτων
Post by: princess_of_the_dawn on July 05, 2015, 18:44:52 pm
το κέρδος DC πώς το βρίσκουμε;


Title: Re: [ΨΕΣ] Λύσεις παλιών θεμάτων
Post by: thomais on July 05, 2015, 20:25:32 pm
το κέρδος DC πώς το βρίσκουμε;
Έχω την εντύπωση ότι το βρίσκουμε αν στην συνάρτηση μεταφοράς θέσουμε το ω=0.


Title: Re: [ΨΕΣ] Λύσεις παλιών θεμάτων
Post by: robaudelaire on July 05, 2015, 20:36:49 pm
και εγω λογικα αυτο θα κανα , δηλ |Χ(ω=0)|


Title: Re: [ΨΕΣ] Λύσεις παλιών θεμάτων
Post by: princess_of_the_dawn on July 05, 2015, 20:52:32 pm
gracias


Title: Re: [ΨΕΣ] Λύσεις παλιών θεμάτων
Post by: Mous on July 06, 2015, 00:29:11 am
εχει λυσει κανεις φεβρουαριο 2015?


Title: Re: [ΨΕΣ] Λύσεις παλιών θεμάτων
Post by: princess_of_the_dawn on July 06, 2015, 00:42:18 am
εγω το 1ο και 3ο θεμα αλλα κατα 90% ειναι λαθοοοοος
αμα τα εχεις λυσει πες αποτελεσματα να συγκρινουμε


Title: Re: [ΨΕΣ] Λύσεις παλιών θεμάτων
Post by: Mous on July 06, 2015, 13:33:02 pm
εγω το 1ο και 3ο θεμα αλλα κατα 90% ειναι λαθοοοοος
αμα τα εχεις λυσει πες αποτελεσματα να συγκρινουμε
για δες θεμα 3 και πες, δεν φαινεται πολυ καλα σορρυ...


Title: Re: [ΨΕΣ] Λύσεις παλιών θεμάτων
Post by: Mous on July 06, 2015, 13:48:11 pm
εγω το 1ο και 3ο θεμα αλλα κατα 90% ειναι λαθοοοοος
αμα τα εχεις λυσει πες αποτελεσματα να συγκρινουμε
αν εχεις κανει το 1ο ανεβασε λιγο να δω την λογικη


Title: Re: [ΨΕΣ] Λύσεις παλιών θεμάτων
Post by: princess_of_the_dawn on July 06, 2015, 14:04:40 pm
εγω το 1ο και 3ο θεμα αλλα κατα 90% ειναι λαθοοοοος
αμα τα εχεις λυσει πες αποτελεσματα να συγκρινουμε
αν εχεις κανει το 1ο ανεβασε λιγο να δω την λογικη
μισο γτ νομζω εντοπισα λαθος
το 3 σωστο μου φαινεται
εγω ειχα κανει αλλο πραγμα αλλα ειμαι πραγματικα ασχετη, τ δικο οσυ φαινεται σωστοτερο  ;)


Title: Re: [ΨΕΣ] Λύσεις παλιών θεμάτων
Post by: Mous on July 06, 2015, 14:14:19 pm
ναι γιατι εγω ειμαι γνωστης πιστευεις....? ;D


Title: Re: [ΨΕΣ] Λύσεις παλιών θεμάτων
Post by: princess_of_the_dawn on July 06, 2015, 14:35:58 pm
παρακαλείται κάποιος γνωστης να σωσει την κατασταση  :D


Title: Re: [ΨΕΣ] Λύσεις παλιών θεμάτων
Post by: disconnected on July 06, 2015, 15:28:54 pm
Παραθέτω την λύση όπως την είχε κάνει μέσα στην τάξη.

Επίσης κανείς που να έχει κάνει το 2ο του Φεβ 2013?

Edit: Απλά δεν έχει Μ/Χ Fourier, έτσι?


Title: Re: [ΨΕΣ] Λύσεις παλιών θεμάτων
Post by: robaudelaire on July 06, 2015, 15:56:39 pm
για το 2ο του 2013 νομιζω ειναι αρκετα κουλ ,
αναλυεις το sin(ωο) στην γνωστη εκθετικη σχεση και απο την ιδιοτητα της διαμορφωσης λογω αυτου του ωο βγαζεις το X(ω)


Title: Re: [ΨΕΣ] Λύσεις παλιών θεμάτων
Post by: robaudelaire on July 06, 2015, 15:58:56 pm
επειδη στο δινει |α|^n μπορεις να το θεωρησεις λογικα α^n*u(n) σχεση της οποιας ο φουριε γνωστος :/


Title: Re: [ΨΕΣ] Λύσεις παλιών θεμάτων
Post by: disconnected on July 06, 2015, 16:04:07 pm
επειδη στο δινει |α|^n μπορεις να το θεωρησεις λογικα α^n*u(n) σχεση της οποιας ο φουριε γνωστος :/

Αναλύοντάς το όμως προκύπτει ένα άθροισμα N=-inf:-1 του |α|^n που άμα θέσεις -n=n γίνεται άθροισμα n=1:inf του |α|^(-n) και επειδή |α|<1, αυτό το άθροισμα δεν συγκλίνει. Σωστά?


Title: Re: [ΨΕΣ] Λύσεις παλιών θεμάτων
Post by: robaudelaire on July 06, 2015, 16:08:20 pm
αφου το κανεις α^n * u(n) λογω του u(n) το αθροισμα θα παει απο 0 εως inf και με |α|<1 οποτε συγκλινει και παιρνεις τη γνωστη μορφη
ετσι νομιζω εγω τουλαχιστον


Title: Re: [ΨΕΣ] Λύσεις παλιών θεμάτων
Post by: disconnected on July 06, 2015, 16:16:47 pm
αφου το κανεις α^n * u(n) λογω του u(n) το αθροισμα θα παει απο 0 εως inf και με |α|<1 οποτε συγκλινει και παιρνεις τη γνωστη μορφη
ετσι νομιζω εγω τουλαχιστον

Δεν νομίζω ότι μπορείς να το θεωρήσεις ότι είναι |a|^n*u(n). Και απ'ότι βλέπω είναι λυμένο στις ασκήσεις του proaki και εκεί επιβεβαιώνει ότι δεν υπάρχει ο Fourier, με μία έμμεση λύση..


Title: Re: [ΨΕΣ] Λύσεις παλιών θεμάτων
Post by: robaudelaire on July 06, 2015, 16:22:59 pm
 :???:
οκ τοτε


Title: Re: [ΨΕΣ] Λύσεις παλιών θεμάτων
Post by: princess_of_the_dawn on July 06, 2015, 22:18:20 pm
παραθέτω τη λύση για το θέμα 3 του 2015 από τον ρομπωντλέρ
όποιος θέλει ας του ρίξει μια ματιά να μας πεί για είναι σωστό


Title: Re: [ΨΕΣ] Λύσεις παλιών θεμάτων
Post by: RaiDeR on July 06, 2015, 22:38:01 pm
παραθέτω τη λύση για το θέμα 3 του 2015 από τον ρομπωντλέρ
όποιος θέλει ας του ρίξει μια ματιά να μας πεί για είναι σωστό
Για την 3 Νομιζω οτι εισαι Σωστος . Και εγω το ιδιο εκανα. Αν εχεις λυσει την 1 και καμια αλλη ανεβασε τες σε παρακαλω


Title: Re: [ΨΕΣ] Λύσεις παλιών θεμάτων
Post by: princess_of_the_dawn on July 06, 2015, 23:39:33 pm
παραθέτω τη λύση για το θέμα 3 του 2015 από τον ρομπωντλέρ
όποιος θέλει ας του ρίξει μια ματιά να μας πεί για είναι σωστό
Για την 3 Νομιζω οτι εισαι Σωστος . Και εγω το ιδιο εκανα. Αν εχεις λυσει την 1 και καμια αλλη ανεβασε τες σε παρακαλω
μπαη ρόμπο


Title: Re: [ΨΕΣ] Λύσεις παλιών θεμάτων
Post by: Professional Hater on July 07, 2015, 05:28:37 am
παραθέτω τη λύση για το θέμα 3 του 2015 από τον ρομπωντλέρ
όποιος θέλει ας του ρίξει μια ματιά να μας πεί για είναι σωστό
Κακή ώρα μάλλον αλλά εγώ διαφωνώ εκεί που σπας τα κλάσματα.. Για την ακρίβεια, αν αντικαταστήσεις τους exp όρους με z και πολλαπλασιάσεις αριθμητή και παρονομαστή με 12 (για να φανεί πιο όμορφο όχι τιποτα αλλο), καταλήγεις να έχεις το κλάσμα

( 12 z^2 - 7z +12 )  /  (  12z - 6 ) = ( 12 z^2 - 6 z ) / (12 z - 6 )   -   z / ( 12z - 6 )   +   12 / (12z - 6) .

Ο δικός μου πρώτος όρος κάνει z (που είναι λογικό να υπάρχει αφού το πολυώνυμο του αριθμητή είναι μεγαλύτερου βαθμού) και άρα μετασχηματίζεται σε δ(n+1). Κάνω λάθος?


Title: Re: [ΨΕΣ] Λύσεις παλιών θεμάτων
Post by: xameno kormi on September 19, 2015, 12:53:05 pm
εχει ασχοληθει κανεις με το θεμα 4 απο φλεβαρη '15 ?


Title: Re: [ΨΕΣ] Λύσεις παλιών θεμάτων
Post by: Niobe on September 19, 2015, 14:58:35 pm
εχει ασχοληθει κανεις με το θεμα 4 απο φλεβαρη '15 ?
Που'σαι ρε μαν, πως πηγε ο λογισμος?
On topic, παιζει να προλαβω να λυσω τον φλεβαρη του 15 και να τον σκαναρω μελενικου, μεχρι το βραδυ, να πειτε καμια γνωμη


Title: Re: [ΨΕΣ] Λύσεις παλιών θεμάτων
Post by: xameno kormi on September 19, 2015, 16:03:55 pm
ελα ρε μαν κ εδω το team θα παιξει ? :D θα δειξει νομιζα θα τα βγαζε τοσο γρηγορα οσο τα εφαρμοσμενα :P εσυ πως πηγε ??
θα προσπαθησω κ γω να λυσω τιποτα να μ πειτε ..


Title: Re: [ΨΕΣ] Λύσεις παλιών θεμάτων
Post by: Niobe on September 19, 2015, 17:30:38 pm
Ναι ρε strength in numbers..

Βασικα μια παρατηρηση: η λυση του 1ου θεματος του Σεπτεμβρη του '12 εχει λαθος νομιζω οσον αφορα το arg(H)


Title: Re: [ΨΕΣ] Λύσεις παλιών θεμάτων
Post by: inspired on September 19, 2015, 20:27:54 pm
Ναι ρε strength in numbers..

Βασικα μια παρατηρηση: η λυση του 1ου θεματος του Σεπτεμβρη του '12 εχει λαθος νομιζω οσον αφορα το arg(H)
Ναι, ειναι απλα (-60ω-25ω^2)


Title: Re: [ΨΕΣ] Λύσεις παλιών θεμάτων
Post by: xameno kormi on September 19, 2015, 20:54:31 pm
για το θεμα 4 του φλεβαρη 15
ειναι y[n] = x[n] + y[n-3] απο το διαγραμμα ?

εδιτ : το βρηκα γτ ονομασα εστω k(n) τον κομβο - αθροιστη k(n)= x(n) + k(n-3) και y(n) = 1 * k(n)


Title: Re: [ΨΕΣ] Λύσεις παλιών θεμάτων
Post by: MG9S on September 20, 2015, 19:16:51 pm
και εγω ετσι το βρήκα


Title: Re: [ΨΕΣ] Λύσεις παλιών θεμάτων
Post by: xameno kormi on September 20, 2015, 20:05:01 pm
αρα h(n) = u(n-3) ? γνκ και με μετ/μο Ζ και και μετα αντιστροφο Ζ ή λογω της ιδ. οτι αν εχω y(n)=a*y(n-1) + x(n) τοτε h(n)=a^n * u(n-1) το ιδιο δν προκυπτει ?


Title: Re: [ΨΕΣ] Λύσεις παλιών θεμάτων
Post by: Niobe on September 20, 2015, 20:25:55 pm
αρα h(n) = u(n-3) ? γνκ και με μετ/μο Ζ και και μετα αντιστροφο Ζ ή λογω της ιδ. οτι αν εχω y(n)=a*y(n-1) + x(n) τοτε h(n)=a^n * u(n-1) το ιδιο δν προκυπτει ?
confirmed!
Στο 2ο θεμα, επειδη εχει πολο στο 1/3 και μηδενικο στο 3 μπορω να γραψω οτι ειναι H(z)=(z-3)/(z-1/3) και να βρω μετα τα πραγματα?
Βασικα δε μπορω να καταλαβω αν απο το διαγραμμα προκυπτει περιοχη συγκλισης


Title: Re: [ΨΕΣ] Λύσεις παλιών θεμάτων
Post by: xameno kormi on September 20, 2015, 20:35:18 pm
βασικα ειναι νομιζω  G επι την H(z) αυτην που εγραψες ! οπου G μια σταθερα... αυτο για το κερδος εχω την εντυπωση χωρις να ειμαι σιγουρος οτι ειναι | Η(z) | = 1 για z=1 αρα ετσι βρισκεις το G ..


Title: Re: [ΨΕΣ] Λύσεις παλιών θεμάτων
Post by: Niobe on September 20, 2015, 20:41:00 pm
Το DC κερδος ειναι για ω=0? Γιατι αυτο το κολπο με τη συνελιξη?


(υ.γ. το μαθημα με εχει εκνευρισει οσο λιγα)


Title: Re: [ΨΕΣ] Λύσεις παλιών θεμάτων
Post by: MG9S on September 20, 2015, 20:43:57 pm
αφου ο πόλος είναι μέσα στο κύκλο τότε είναι αιτιατό άρα δεξιάς πλευράς και έτσι μπορείς να βρείς που συγκλίνει ! αν κατάλαβα σώστα !


Title: Re: [ΨΕΣ] Λύσεις παλιών θεμάτων
Post by: xameno kormi on September 20, 2015, 20:46:20 pm
σορυ δεν εννοουσα συνελιξη ! πολ/μο εννουσα θα το αλλαξω...ειναι G επι την H(z) που εγραψες δλδ G (z-3)/(z-1/3)  ..γιατι μπορει να εχει κ αλλους ορους(σταθερους) οι οποιοι δν επηρεαζουν και δν φαινονται στο σχημα γιατι δν ειναι πολοι η μηδενικα οποτε βαζεις την σταθερα G..


Title: Re: [ΨΕΣ] Λύσεις παλιών θεμάτων
Post by: MG9S on September 20, 2015, 21:15:05 pm
Για την άσκηση 4 του 15 μήπως είναι λάθος αυτό που είπαμε? επειδή στη σελίδα 226 του hayes εχει ενα παράδειγμα όπου λέει α^(n/10)<-ΖΤ->1/(1-αζ^(-10))


Title: Re: [ΨΕΣ] Λύσεις παλιών θεμάτων
Post by: xameno kormi on September 20, 2015, 22:41:01 pm
εχεις δικιο λαθος ειναι ..
το προχωρησα λιγο με διαφορα κυβων, μπορεις να καταληξεις στον παρανομαστη σε 3 παρενθεσεις με (z - κατι) και να κανεις μετα μεθοδο μερικων κλασματων...οποτε να τα σπασεις σε κλασματα με z^-1 μονο μετα.. αν το δοκιμασει κ κανεις αλλος ας πει.. 


Title: Re: [ΨΕΣ] Λύσεις παλιών θεμάτων
Post by: MG9S on September 21, 2015, 18:50:08 pm
μήπως έχει δοκιμάσει καποιος την 4η απο του ιουνίου 2014?


Title: Re: [ΨΕΣ] Λύσεις παλιών θεμάτων
Post by: Αλντεμπαράν on September 21, 2015, 20:54:56 pm
μήπως έχει δοκιμάσει καποιος την 4η απο του ιουνίου 2014?

Η λύση στο συνημμένο.


Title: Re: [ΨΕΣ] Λύσεις παλιών θεμάτων
Post by: vasilis94 on February 06, 2016, 23:16:32 pm
Ας ποστάρω κάτι που με ταλαιπώρησε και ακόμα δεν έχω καταλάβει πλήρως (μιας και έχει απαντηθεί λάθος σε προηγούμενη σελίδα και για να ανοίξει και η κουβέντα γενικά   :P ).

Θέμα 1ο Σεπτέμβριος 2012 (αυτό με τα σχηματάκια που πρέπει να αποφασίσουμε τη x[n] από το Fourier της)

Αυτό που είχε απαντηθεί ήταν ότι είναι το 2, γιατί:
> Το e^(-i*60*w) αντιστοιχεί σε μετατόπιση κατά 60 δεξιά στο χρόνο, άρα μένουν τα 2 και 5 (γιατί ακόμα και το 3 φαίνεται να υπάρχει λίγο πριν το 60)
> Το κυμάτισμα που μπαίνει οφείλεται στον όρο w^3 είναι αυτό που προκαλεί τα αποσβενύμμενα συνημίτονα στο 2ο διάγραμμα.

Παρόλα αυτά βάζω στο matlab να μου υπολογίσει την x[60] και μου βγάζει 0.08 περίπου... Τέλοσπαντων το κάνω για αρκετές τιμές και ιδού:
(http://s21.postimg.org/rr3c8lft3/stoxrono1.jpg[/size])
(http://s21.postimg.org/6vh1xcjlz/stoxrono2.jpg)

Όπως φαίνεται είναι το 3ο σχήμα. Ωστόσο, από το 2ο σχήμα στο ποστ φαίνεται ότι δεν έχει γίνει κάποιο λάθος και όντως υπάρχει και περιεχόμενο για n<60. Κάτι λογικό, αφού δε μας είπε κανένας ότι το σήμα με FT τον e^(-i*a*w^3) είναι αιτιατό  :P
Αλλά τότε πως δικαιολογούμε; Μάλλον το κάνουμε γαργάρα λίγο και 2 από τα χαρακτηρηστικά που ζητάει είναι:
> Έχουμε μια μετατόπιση του σήματος κατά 60 από τον τύπο (αλλά όπως είπαμε αν to αρχικό σήμα παράγει τον όρο w^3 ήταν στο -60;; )
> Οι διαφορετικές συχνότητες είναι ίσου πλάτους (μέτρο του ft σταθερο), οπότε δε περιμένουμε καμιά τρελή απόσβεση και εκφράζονται μέσω του πυκνώματος των γραμμών.


Title: Re: [ΨΕΣ] Λύσεις παλιών θεμάτων
Post by: Eilex on February 06, 2016, 23:18:49 pm
[...]

Ένας μικρός θεός!Μπράβο για την εξαιρετική δουλειά!


edit: Σου έφαγα το quote να μη φαίνεται μεγάλο συνέχεια (και thanks, απλά βαριόμουν λίγο να δω άλλα θέματα τώρα)


Title: Re: [ΨΕΣ] Λύσεις παλιών θεμάτων
Post by: Ragnar Lothbrok on February 07, 2016, 16:37:48 pm
Λυσεις σεπτεμβριου 2015 εχουμε;;;;


Title: Re: [ΨΕΣ] Λύσεις παλιών θεμάτων
Post by: vasilis94 on February 07, 2016, 20:54:13 pm
Λυσεις σεπτεμβριου 2015 εχουμε;;;;

Δεν είχε και κανένα τρομερό ενδιαφέρον, ελπίζω να μην έκανα χαζομάρες μόνο.

Στο 1ο ορισμός και πράξεις.
Στο 2ο προσοχή στο διάστημα σύγκλισης!
Στο 3ο τα πήγα όλα στο Z, μου φαίνεται το πιο απλό συνήθως σε τέτοιες περιπτώσεις και μετά το γυρνάς στην εξίσωση διαφορών όταν αποκτήσεις μια μορφή που θα έχει X(z), Υ(z). W ονόμασα το ενδιάμεσο σήμα.
Στο 4ο το υπολογίζεις βάσεις της ερμηνείας με τις αποστάσεις από μηδενικά- πόλους(δες Hayes 258-259). Προφανώς στο μηδενικό πάνω μηδενίζεται η απόκριση (έχει ξαναβάλει θέμα νομίζω και το χει αναφέρει και στο μάθημα φέτος)


Title: Re: [ΨΕΣ] Λύσεις παλιών θεμάτων
Post by: Ragnar Lothbrok on February 07, 2016, 21:25:01 pm
Λυσεις σεπτεμβριου 2015 εχουμε;;;;

Δεν είχε και κανένα τρομερό ενδιαφέρον, ελπίζω να μην έκανα χαζομάρες μόνο.

Στο 1ο ορισμός και πράξεις.
Στο 2ο προσοχή στο διάστημα σύγκλισης!
Στο 3ο τα πήγα όλα στο Z, μου φαίνεται το πιο απλό συνήθως σε τέτοιες περιπτώσεις και μετά το γυρνάς στην εξίσωση διαφορών όταν αποκτήσεις μια μορφή που θα έχει X(z), Υ(z). W ονόμασα το ενδιάμεσο σήμα.
Στο 4ο το υπολογίζεις βάσεις της ερμηνείας με τις αποστάσεις από μηδενικά- πόλους(δες Hayes 258-259). Προφανώς στο μηδενικό πάνω μηδενίζεται η απόκριση (έχει ξαναβάλει θέμα νομίζω και το χει αναφέρει και στο μάθημα φέτος)

χιλια ευχαριστω μπιλι, εισαι τεραστιος!!!  ^wav^


Title: Re: [ΨΕΣ] Λύσεις παλιών θεμάτων
Post by: Eilex on February 07, 2016, 22:27:39 pm
Λυσεις σεπτεμβριου 2015 εχουμε;;;;

Δεν είχε και κανένα τρομερό ενδιαφέρον, ελπίζω να μην έκανα χαζομάρες μόνο.

Στο 1ο ορισμός και πράξεις.
Στο 2ο προσοχή στο διάστημα σύγκλισης!
Στο 3ο τα πήγα όλα στο Z, μου φαίνεται το πιο απλό συνήθως σε τέτοιες περιπτώσεις και μετά το γυρνάς στην εξίσωση διαφορών όταν αποκτήσεις μια μορφή που θα έχει X(z), Υ(z). W ονόμασα το ενδιάμεσο σήμα.
Στο 4ο το υπολογίζεις βάσεις της ερμηνείας με τις αποστάσεις από μηδενικά- πόλους(δες Hayes 258-259). Προφανώς στο μηδενικό πάνω μηδενίζεται η απόκριση (έχει ξαναβάλει θέμα νομίζω και το χει αναφέρει και στο μάθημα φέτος)

Στο 3ο για πιο γρήγορη λύση μπορείς να θεωρήσεις ότι το u[n]-u[n-2] = δ[n]+δ[n-1].Για να γλιτώσεις κάποιες πράξεις.


Title: Re: [ΨΕΣ] Λύσεις παλιών θεμάτων
Post by: forisonex on February 07, 2016, 23:09:27 pm
Μπορεί να είναι χαζή ερώτηση αλλά το πολυώνυμο 1- 3/4 * z^(-1)+ 1/8 * z^(-2) ,πως το σπάμε σε γινόμενο 2 και να έχει εκθέτες μονο z^(-1) ?


Title: Re: [ΨΕΣ] Λύσεις παλιών θεμάτων
Post by: vasilis94 on February 07, 2016, 23:22:01 pm
Μπορεί να είναι χαζή ερώτηση αλλά το πολυώνυμο 1- 3/4 * z^(-1)+ 1/8 * z^(-2) ,πως το σπάμε σε γινόμενο 2 και να έχει εκθέτες μονο z^(-1) ?

Βρίσκεις τις ρίζες του z2-3/4*z+1/8 και κάνεις παραγοντοποίηση κατά τα γνωστά: (1-ρ1z-1)(1-ρ2z-2)

Εν προκειμένω: (1-1/2*z-1)(1-1/4*z-1)


Στο 3ο για πιο γρήγορη λύση μπορείς να θεωρήσεις ότι το u[n]-u[n-2]= δ[n]+

Ωραίος...


Title: Re: [ΨΕΣ] Λύσεις παλιών θεμάτων
Post by: forisonex on February 08, 2016, 00:31:42 am
Μπορεί να είναι χαζή ερώτηση αλλά το πολυώνυμο 1- 3/4 * z^(-1)+ 1/8 * z^(-2) ,πως το σπάμε σε γινόμενο 2 και να έχει εκθέτες μονο z^(-1) ?

Βρίσκεις τις ρίζες του z2-3/4*z+1/8 και κάνεις παραγοντοποίηση κατά τα γνωστά: (1-ρ1z-1)(1-ρ2z-2)

Εν προκειμένω: (1-1/2*z-1)(1-1/4*z-1)

Στο 3ο για πιο γρήγορη λύση μπορείς να θεωρήσεις ότι το u[n]-u[n-2]= δ[n]+

Ωραίος...

Thanks a lot!


Title: Re: [ΨΕΣ] Λύσεις παλιών θεμάτων
Post by: PanosGo on June 07, 2016, 21:48:21 pm
Παίζουν πουθενά οι λύσεις από το Φλεβάρη του 16;;


Title: Re: [ΨΕΣ] Λύσεις παλιών θεμάτων
Post by: joal on June 08, 2016, 01:26:25 am
Παίζουν πουθενά οι λύσεις από το Φλεβάρη του 16;;


Title: Re: [ΨΕΣ] Λύσεις παλιών θεμάτων
Post by: Παναγιώτης on June 08, 2016, 16:55:49 pm
2ο θέμα - Γενάρης 2012, καμιά ιδέα πως λύνεται?


Title: Re: [ΨΕΣ] Λύσεις παλιών θεμάτων
Post by: Maylo on August 21, 2016, 17:46:54 pm
2ο θέμα - Γενάρης 2012, καμιά ιδέα πως λύνεται?

Αδερφέ μου έχει ακριβώς λυμένο στο "proakis manolakis 4th edition"  παράδειγμα 3.2.8   , 164 σελίδα


Title: Re: [ΨΕΣ] Λύσεις παλιών θεμάτων
Post by: Maylo on August 21, 2016, 19:48:46 pm
Καλησπέρα , για το θέμα 4ο του Ιανουαρίου 2012 είδα πως κάπου ειπωθηκε οτι βρίσκεται λυμένη στου hayes 108 σελίδα όμως εγώ δεν το έχω . Από internet βρήκα τ (https://rockingme.files.wordpress.com/2011/09/ebook__schaum_s_outlines_-__digital_signal_processing.pdf) εδώ δεν βλέπω κάτι στην 108 . Είναι το ίδιο βιβλίο??

όπως και να χει μπορεί κάποιος να βοηθήσει για το συγκεκριμένο θέμα;


Title: Re: [ΨΕΣ] Λύσεις παλιών θεμάτων
Post by: Lampros on August 21, 2016, 20:03:33 pm
Ελπίζω να φαίνονται

εδιτ: είναι αυτό που ανέφερε ο προηγούμενος


Title: Re: [ΨΕΣ] Λύσεις παλιών θεμάτων
Post by: Maylo on August 21, 2016, 20:36:49 pm
Να σαι καλά φιλε


Title: Re: [ΨΕΣ] Λύσεις παλιών θεμάτων
Post by: lady_of_winter on August 27, 2016, 12:55:22 pm
Θεμα 2 - Φλεβαρης 2016
στο τριτο ερώτημα ρωτά αν ισχύει η σχεση Η(e^jω) = Η(z) όπου z = e^jω


τι θα πρέπει να συμβαινει για να ισχυει αυτο?
στο βιβλιο του hayes , σελ 251 λεει οτι μπορούμε να εξάγουμε τη συνάρτηση μεταφοράς από τον τύπο αυτό πάνω και γύρω στο μοναδιαίο κύκλο

αυτό σημαίνει οτι μας αρκεί να ισχύει ότι στο πεδίο σύγκλισης ανήκει κ ο μοναδιαίος κύκλος?ή κατι άλλο που δε καταλαβαινω?


Title: Re: [ΨΕΣ] Λύσεις παλιών θεμάτων
Post by: vasilis94 on August 27, 2016, 13:00:11 pm
Θεμα 2 - Φλεβαρης 2016
στο τριτο ερώτημα ρωτά αν ισχύει η σχεση Η(e^jω) = Η(z) όπου z = e^jω


τι θα πρέπει να συμβαινει για να ισχυει αυτο?
στο βιβλιο του hayes , σελ 251 λεει οτι μπορούμε να εξάγουμε τη συνάρτηση μεταφοράς από τον τύπο αυτό πάνω και γύρω στο μοναδιαίο κύκλο

αυτό σημαίνει οτι μας αρκεί να ισχύει ότι στο πεδίο σύγκλισης ανήκει κ ο μοναδιαίος κύκλος?ή κατι άλλο που δε καταλαβαινω?

Ακριβώς αυτό!


Title: Re: [ΨΕΣ] Λύσεις παλιών θεμάτων
Post by: lady_of_winter on August 27, 2016, 13:00:55 pm
ευχαριστω  :D


Title: Re: [ΨΕΣ] Λύσεις παλιών θεμάτων
Post by: Λήσταρχος Γιαγκούλας on August 30, 2016, 02:22:51 am
Θέμα 1ο Φλεβάρης 2016
Εχει βρει μήπως κάποιος ως αποτέλεσμα

x[n]=δ [ n ]  + 3δ [ n-2 ] - 2 δ [  n-3 ]

?

Το feedback και τα φώτα σας παρακαλώ συνάδελφοι!


Title: Re: [ΨΕΣ] Λύσεις παλιών θεμάτων
Post by: Maylo on August 30, 2016, 15:35:02 pm
Καλησπέρα , για θέματα όπως το 4ο του 2009 Φλεβάρη πού μπορούμε να δούμε σχετική θεωρία ;


Title: Re: [ΨΕΣ] Λύσεις παλιών θεμάτων
Post by: vasilis94 on August 30, 2016, 23:15:10 pm
Θέμα 1ο Φλεβάρης 2016
Εχει βρει μήπως κάποιος ως αποτέλεσμα

x[n]=δ [ n ]  + 3δ [ n-2 ] - 2 δ [  n-3 ]

?

Το feedback και τα φώτα σας παρακαλώ συνάδελφοι!

Νομίζω ότι είναι x[n]=δ [ n ]  - 3δ [ n-2 ] + 2 δ [  n-3 ]. Πάρε το Fourier αυτουνού που έβγαλες βασικά...
Του δ [ n-2 ] είναι exp(-j2w) και το imagine αυτού είναι -sin(2w), οπότε κάπου μάλλον έκανες κάτι λάθος με τα πρόσημα.

Καλησπέρα , για θέματα όπως το 4ο του 2009 Φλεβάρη πού μπορούμε να δούμε σχετική θεωρία ;

Τελείως χαζό θέμα. Λογικά ή στο αναλογικό στη δειγματοληψία ή στο Schaum στα εισαγωγικά του κεφαλαίου. Δε χρειάζονται και πολλά, τα βασικά της δειγματοληψίας. Δηλαδή ότι το φάσμα αντιγράφεται κάθε 1/Τ (πχ σχήμα 6,5,3 σελ 156 αναλογικό με τη διαφορά του 2π επειδή ο άξονας είναι ω).

Οπότε, στο θέμα μας στην αρχή σκέφτηκα Nyquist δηλαδή fs>2f3, αλλά αφού έχει τη μορφή αυτή με τις δ, αρκεί να είναι 4KHz το shift και δε θα συμπέφτουν ποτέ δύο κορυφές μαζί. Καλά και με 0.1ΚΗz δε θα συμπίπτουν ποτέ, αλλά λέει από 1Κ με βήμα 1Κ είναι τα διαθέσιμα.

Τώρα για τα δείγματα που ρωτάει λογικά αρκούν 4. Θα έχεις ένα 4*4 γραμμικό σύστημα (ε, δε παίζει να βγαίνει η ορίζουσα 0 για να χει άπειρες λύσεις...)


Title: Re: [ΨΕΣ] Λύσεις παλιών θεμάτων
Post by: georgopk on August 31, 2016, 13:23:15 pm
Παίδες, μια βοήθεια στο θέμα 1 Φεβρουάριος του 16;;;

έχω την x[n]. την σπάω σε xo[n] και xe[n], (περιττή και άρτια αντίστοιχα)
ο Fourier της x θα είναι ο X=Xo+Xe, (το άθροισμα των μετασχηματισμών Fourier της περιττής και της άρτιας)
λόγω συμμετρίας και επειδή η x[n] είναι πραγματική κλπ κλπ (βλέπε πινακάκι σελ 97 στον Hayes) θα έχουμε οτι
Χο περιττή και φανταστική
Χe άρτια και πραγματική
Η x[n] είναι αιτιατή, άρα x[n]=0 για n<0, το οποίο σημαίνει ότι για n<0 οι xo και xe είναι αντίθετες. Άρα για n>0 είναι ίσες. ΔΗΛΑΔΗ αρκεί να βρώ την xo[n]. (μας δίνει και την τιμή x[0]=1, άρα είμαστε καλυμμένοι!!! )

Τώρα η φτωχή μου λογική μου λέει οτι παίρνω την Χο που είναι ουσιαστικά αυτή που μας δώθηκε Χο=( 3sin(2ω)-2sin(3ω) )*i και ο αντίστροφος Fourier αυτής, είναι η xο[n].

προσωπικά πήρα τον τύπο (2.4) στη σελίδα 96 του Hayes, αλλά ή έχω ξεχάσει να λύνω ολοκλήρωμα (πολύυυυυ πιθανό) ή αυτό το πράγμα θέλει υπολογιστή, ή έχω κάνει κάτι λάθος πιο πριν (ή όλα τα παραπάνω  :P ;D)

Σας παρακαλω πείτε μου άν έχω κάποιο λάθος στη λογική γι να μην ψάχνω τζάμπα τις πράξεις μου.
και αν δεν έχω λάθος στη λογική, δώστε καμιά συμβουλή για τις πράξεις πλιιιιιζ! ^beg^


Title: Re: [ΨΕΣ] Λύσεις παλιών θεμάτων
Post by: Lampros on August 31, 2016, 18:16:03 pm
δεν ξέρω αν βοηθάει αλλά δες λίγο σελ 277 ασκ 5.12 Hayes


Title: Re: [ΨΕΣ] Λύσεις παλιών θεμάτων
Post by: Κυναίγειρος on August 31, 2016, 19:10:50 pm
Παίδες, μια βοήθεια στο θέμα 1 Φεβρουάριος του 16;;;

έχω την x[n]. την σπάω σε xo[n] και xe[n], (περιττή και άρτια αντίστοιχα)
ο Fourier της x θα είναι ο X=Xo+Xe, (το άθροισμα των μετασχηματισμών Fourier της περιττής και της άρτιας)
λόγω συμμετρίας και επειδή η x[n] είναι πραγματική κλπ κλπ (βλέπε πινακάκι σελ 97 στον Hayes) θα έχουμε οτι
Χο περιττή και φανταστική
Χe άρτια και πραγματική
Η x[n] είναι αιτιατή, άρα x[n]=0 για n<0, το οποίο σημαίνει ότι για n<0 οι xo και xe είναι αντίθετες. Άρα για n>0 είναι ίσες. ΔΗΛΑΔΗ αρκεί να βρώ την xo[n]. (μας δίνει και την τιμή x[0]=1, άρα είμαστε καλυμμένοι!!! )

Τώρα η φτωχή μου λογική μου λέει οτι παίρνω την Χο που είναι ουσιαστικά αυτή που μας δώθηκε Χο=( 3sin(2ω)-2sin(3ω) )*i και ο αντίστροφος Fourier αυτής, είναι η xο[n].

προσωπικά πήρα τον τύπο (2.4) στη σελίδα 96 του Hayes, αλλά ή έχω ξεχάσει να λύνω ολοκλήρωμα (πολύυυυυ πιθανό) ή αυτό το πράγμα θέλει υπολογιστή, ή έχω κάνει κάτι λάθος πιο πριν (ή όλα τα παραπάνω  :P ;D)

Σας παρακαλω πείτε μου άν έχω κάποιο λάθος στη λογική γι να μην ψάχνω τζάμπα τις πράξεις μου.
και αν δεν έχω λάθος στη λογική, δώστε καμιά συμβουλή για τις πράξεις πλιιιιιζ! ^beg^

Το θέμα 1/Φεβρουάριος 16 (δες και σελίδα 127 από hayes - άσκηση 2.28)


Title: Re: [ΨΕΣ] Λύσεις παλιών θεμάτων
Post by: vasilis94 on August 31, 2016, 19:12:44 pm

Ωραία η όλη σκέψη μέχρι την αντιστροφή. Εκεί σπάνια χρησιμοποιούμε τον ορισμό του IDTFT. Συνήθως, εκμεταλλευόμαστε τις ιδιότητες και τα πινακάκια.

Άρα:
- Γράφεις τα ημίτονα ως μιγαδικά εκθετικά (sinx=[exp(ix)-exp(-ix)]/2i)
- Βρίσκεις από το πινακάκι με τα ζεύγη ότι τα μιγαδικά εκθετικά αντιστοιχούν σε κάποιες μετατοπισμένες δ[n-n0].
- Ουσιαστικά έχεις κάποιες μεμονωμένες τιμές για n=2,n=3 (και n=0). Παντού αλλού μηδέν.

Δες και τη λυμένη 2,28 από Hayes που χει το αντίστοιχο με το real κομμάτι.

edit: Με πρόλαβαν...


Title: Re: [ΨΕΣ] Λύσεις παλιών θεμάτων
Post by: lady_of_winter on August 31, 2016, 19:28:34 pm
Θεμα 3 - Φλεβαρης 2016
το μηδενικο είναι στο j2, πολλαπλότητας 2 και οι πόλοι στα (sqrt(2)+14)/(2*sqrt(2)) και  (sqrt(2)-14)/(2*sqrt(2))??

λαθος, καποια στιγμη θα μαθω να κανω σωστα πραξεις :P


Title: Re: [ΨΕΣ] Λύσεις παλιών θεμάτων
Post by: vasilis94 on August 31, 2016, 19:39:58 pm
Θεμα 3 - Φλεβαρης 2016
το μηδενικο είναι στο j2, πολλαπλότητας 2 και οι πόλοι στα (sqrt(2)+14)/(2*sqrt(2)) και  (sqrt(2)-14)/(2*sqrt(2))??

Γιατί έβγαλες τέτοιους πόλους;;
1-1/4*z-1-3/8*z-2=0 -> z2-1/4z-3/8=0 -> z=-1/2 ή z=3/4

btw, είχε ο Hayes μια λυμένη που έλεγε άμεσα πως γίνεται η παραγοντοποίηση σε all pass και ελάχιστης φάσης και τέλειωνε η δουλειά σχετικά άμεσα (δεν έχω τώρα το βιβλίο για να τη βρω), η 5,29


Title: Re: [ΨΕΣ] Λύσεις παλιών θεμάτων
Post by: lady_of_winter on August 31, 2016, 20:04:55 pm
ευχαριστω πολυ, δε ξερω εγω δευτεροβαθμια ελυσα κ μου βγηκαν αυτα, και μου φανηκαν φουλ περιεργα
το βρηκα τελικα, επειτα απο 5 φορες που εκανα διακρινουσα, ειχα κανει λαθος ομωνυμα  >:( >:(

ευχαριστω πολυ! θα την τσεκαρω!

οποτε με τους πολους ειμαστε κομπλε, κ το μηδενικο μενει να αλλαξουμε για να κανουμε συναρτηση ελαχιστης φασης


Title: Re: [ΨΕΣ] Λύσεις παλιών θεμάτων
Post by: vasilis94 on August 31, 2016, 22:10:47 pm
ευχαριστω πολυ, δε ξερω εγω δευτεροβαθμια ελυσα κ μου βγηκαν αυτα, και μου φανηκαν φουλ περιεργα
το βρηκα τελικα, επειτα απο 5 φορες που εκανα διακρινουσα, ειχα κανει λαθος ομωνυμα  >:( >:(

ευχαριστω πολυ! θα την τσεκαρω!

οποτε με τους πολους ειμαστε κομπλε, κ το μηδενικο μενει να αλλαξουμε για να κανουμε συναρτηση ελαχιστης φασης


ναι, "παίρνεις" τα μηδενικά στον όρο του all pass φίλτρου και "βάζεις" στον αριθμητή του όρου της ελάχιστης φάσης (και στον παρονομαστή του all pass φίλτρου) τον όρο που έχει και ο hayes και έχει μηδενικά εντός του μοναδιαίου.


Title: Re: [ΨΕΣ] Λύσεις παλιών θεμάτων
Post by: georgopk on August 31, 2016, 23:36:01 pm
δεν ξέρω αν βοηθάει αλλά δες λίγο σελ 277 ασκ 5.12 Hayes
Το θέμα 1/Φεβρουάριος 16 (δες και σελίδα 127 από hayes - άσκηση 2.28)
Ωραία η όλη σκέψη μέχρι την αντιστροφή. Εκεί σπάνια χρησιμοποιούμε τον ορισμό του IDTFT. Συνήθως, εκμεταλλευόμαστε τις ιδιότητες και τα πινακάκια.

Άρα:
- Γράφεις τα ημίτονα ως μιγαδικά εκθετικά (sinx=[exp(ix)-exp(-ix)]/2i)
- Βρίσκεις από το πινακάκι με τα ζεύγη ότι τα μιγαδικά εκθετικά αντιστοιχούν σε κάποιες μετατοπισμένες δ[n-n0].
- Ουσιαστικά έχεις κάποιες μεμονωμένες τιμές για n=2,n=3 (και n=0). Παντού αλλού μηδέν.

Δες και τη λυμένη 2,28 από Hayes που χει το αντίστοιχο με το real κομμάτι.

edit: Με πρόλαβαν...

Ευχαριστώ πολύ παιδιά!! Να σται καλά!


Title: Re: [ΨΕΣ] Λύσεις παλιών θεμάτων
Post by: lady_of_winter on September 01, 2016, 01:03:37 am
ναι, "παίρνεις" τα μηδενικά στον όρο του all pass φίλτρου και "βάζεις" στον αριθμητή του όρου της ελάχιστης φάσης (και στον παρονομαστή του all pass φίλτρου) τον όρο που έχει και ο hayes και έχει μηδενικά εντός του μοναδιαίου.

θενκς

βασιλης για μελος του μηνα :D :D


Title: Re: [ΨΕΣ] Λύσεις παλιών θεμάτων
Post by: giwrgosbg on September 01, 2016, 20:45:25 pm
Δεν είχε και κανένα τρομερό ενδιαφέρον, ελπίζω να μην έκανα χαζομάρες μόνο.

Στο 1ο ορισμός και πράξεις.
Στο 2ο προσοχή στο διάστημα σύγκλισης!
Στο 3ο τα πήγα όλα στο Z, μου φαίνεται το πιο απλό συνήθως σε τέτοιες περιπτώσεις και μετά το γυρνάς στην εξίσωση διαφορών όταν αποκτήσεις μια μορφή που θα έχει X(z), Υ(z). W ονόμασα το ενδιάμεσο σήμα.
Στο 4ο το υπολογίζεις βάσεις της ερμηνείας με τις αποστάσεις από μηδενικά- πόλους(δες Hayes 258-259). Προφανώς στο μηδενικό πάνω μηδενίζεται η απόκριση (έχει ξαναβάλει θέμα νομίζω και το χει αναφέρει και στο μάθημα φέτος)

Γιατι στο θεμα 3ο στην πρωτη σχεση έχεις 1-ζ^(-1);; η δευτερη σχεση μου προέκυψε ακριβως η ίδια αλλα έχω σκαλώσει στην πρωτη  :-\


Title: Re: [ΨΕΣ] Λύσεις παλιών θεμάτων
Post by: elu on September 02, 2016, 01:11:05 am
για το θεμα 4 του φλεβαρη 15
ειναι y[n] = x[n] + y[n-3] απο το διαγραμμα ?

εδιτ : το βρηκα γτ ονομασα εστω k(n) τον κομβο - αθροιστη k(n)= x(n) + k(n-3) και y(n) = 1 * k(n)

Αυτό γιατί είναι έτσι και όχι y[n] = x[n] + x[n-3] ? Μπερδεύτηκα.


Title: Re: [ΨΕΣ] Λύσεις παλιών θεμάτων
Post by: fasoul on September 02, 2016, 02:51:16 am
Αν ζητάει αντίστροφο μετασχηματισμό ζ σε ακολουθία δίπλευρη ή αριστερής πλευράς πώς τον υπολογίζουμε;;
Γιατί οι πίνακες των βιβλίων είναι για την περίπτωση μονόπλευρου μετασχηματισμού


Title: Re: [ΨΕΣ] Λύσεις παλιών θεμάτων
Post by: georgopk on September 02, 2016, 12:02:23 pm
Αν ζητάει αντίστροφο μετασχηματισμό ζ σε ακολουθία δίπλευρη ή αριστερής πλευράς πώς τον υπολογίζουμε;;
Γιατί οι πίνακες των βιβλίων είναι για την περίπτωση μονόπλευρου μετασχηματισμού
+1


Title: Re: [ΨΕΣ] Λύσεις παλιών θεμάτων
Post by: Κυναίγειρος on September 02, 2016, 14:15:49 pm
Αν ζητάει αντίστροφο μετασχηματισμό ζ σε ακολουθία δίπλευρη ή αριστερής πλευράς πώς τον υπολογίζουμε;;
Γιατί οι πίνακες των βιβλίων είναι για την περίπτωση μονόπλευρου μετασχηματισμού

Για ακολουθία αριστερής πλευράς, μπορείς να χρησιμοποιήσεις την ιδιότητα x(-n) <-> Χ(z^-1), εκφράζοντας δηλαδή την ακολουθία αριστερής πλευράς ως ακολουθία δεξιάς πλευράς ανεστραμμένη στον χρόνο. Για δίπλευρη ακολουθία, υποθέτω πως ξεκινάς με το άθροισμα του ορισμού του Μ/Σ Z, το σπας κατάλληλα, και εφαρμόζεις ιδιότητες ή/και αθροίσματα σειρών σελίδας 27 βιβλίου hayes


Title: Re: [ΨΕΣ] Λύσεις παλιών θεμάτων
Post by: Ροζ συννεφάκι on February 04, 2017, 21:22:49 pm
Σεπτέμβριος 2016, θέμα 3, βγαίνει ότι με αυτό το σύστημα προκύπτει y(n)=0 για κάθε n ή κάτι κάνω λάθος;


Title: Re: [ΨΕΣ] Λύσεις παλιών θεμάτων
Post by: mfilip on February 04, 2017, 22:16:59 pm
παιδια εχει κανεις λυσεις παλαιων θεματων? θα βοηθουσε κι εμας που δε σκαμπαζουμε και πολλα


Title: Re: [ΨΕΣ] Λύσεις παλιών θεμάτων
Post by: Ancient on February 04, 2017, 22:31:55 pm
Φεβρουάριος 2016 το θέμα 4 είναι απλή ελαχιστοποίηση συνάρτησης?  :o Μάλλον κάτι μου διαφεύγει...


Title: Re: [ΨΕΣ] Λύσεις παλιών θεμάτων
Post by: Σουλης on February 05, 2017, 00:03:16 am
ναι, ειναι βασικα η μεθοδος αμεσης σχεδιασης φιλτρου που αναφερει ο πανας, φετος δεν διδαχθηκε


Title: Re: [ΨΕΣ] Λύσεις παλιών θεμάτων
Post by: Terzos on February 05, 2017, 23:06:58 pm
Στα λυμένα του Σεπτέμβρη 2012 κάνω λάθος ή το 1α) θα έπρεπε Arg(Η(e))=-60ω-25ω2?


Title: Re: [ΨΕΣ] Λύσεις παλιών θεμάτων
Post by: pansakk on February 06, 2017, 00:38:59 am
Θέμα 3 Φεβρουάριος 2015

Για να βρούμε το h2(-1) τι κάνουμε?


Title: Re: [ΨΕΣ] Λύσεις παλιών θεμάτων
Post by: Terzos on February 06, 2017, 00:47:40 am
Θέμα 3 Φεβρουάριος 2015

Για να βρούμε το h2(-1) τι κάνουμε?

Λογικά αντίστροφο μετασχηματισμό


Title: Re: [ΨΕΣ] Λύσεις παλιών θεμάτων
Post by: pansakk on February 06, 2017, 00:58:20 am
Λογικά αντίστροφο μετασχηματισμό
Για να πάρεις αντίστροφο μετασχηματισμό π.χ. του e/(e-1/2) δεν υποθέτεις αιτιατότητα? (ή λέω βλακείες?)


Title: Re: [ΨΕΣ] Λύσεις παλιών θεμάτων
Post by: Terzos on February 06, 2017, 01:06:26 am
Για να πάρεις αντίστροφο μετασχηματισμό π.χ. του e/(e-1/2) δεν υποθέτεις αιτιατότητα? (ή λέω βλακείες?)
Εννοούσα αντίστροφο μετασχηματισμό Ζ. Δεν το έλυσα, αλλά νομίζω θα βγει κάτι του στυλ z^1 που σημαίνει δ(n+1). Δεν βάζω κ το χέρι μου στη φωτιά.

Σεπτέμβριος του 2015 Θέμα 1ο βγάζω Y(k)=DFT{X(k)}. Το έλυσε μήπως και κανάς άλλος να συγκρίνουμε;


Title: Re: [ΨΕΣ] Λύσεις παλιών θεμάτων
Post by: Terzos on February 06, 2017, 01:31:56 am
Θέμα 4β) Σεπτέμβριος 2015 κανείς;


Title: Re: [ΨΕΣ] Λύσεις παλιών θεμάτων
Post by: pansakk on February 06, 2017, 01:36:57 am
Σεπτέμβριος του 2015 Θέμα 1ο βγάζω Y(k)=DFT{X(k)}. Το έλυσε μήπως και κανάς άλλος να συγκρίνουμε;
Νομίζω βγαίνει Y(k)=X(k)+X(k)*e-jkπ.(Δεν το έλυσα αναλυτικά)


Title: Re: [ΨΕΣ] Λύσεις παλιών θεμάτων
Post by: Terzos on February 06, 2017, 05:14:56 am
Νομίζω βγαίνει Y(k)=X(k)+X(k)*e-jkπ.(Δεν το έλυσα αναλυτικά)

Εγώ αναλυτικά το έλυσα και το έβγαλα έτσι. (Το έσπασα σε 2 αθροίσματα και έκανα αλλαγή μεταβλητής στο 2ο.)
Θέμα 1ο Σεπτέμβριος 2016 βγάζω X(k)=0 για κάθε k. Είναι λογικό;


Title: Re: [ΨΕΣ] Λύσεις παλιών θεμάτων
Post by: Vlassis on August 31, 2017, 13:18:24 pm
Αμα λύσει κανεις τα θεματα του Ρεκανου απο τον Ιανουαριο, ας τα ποσταρει


Title: Re: [ΨΕΣ] Λύσεις παλιών θεμάτων
Post by: SportBillyPap on August 31, 2017, 13:23:30 pm
Αμα λύσει κανεις τα θεματα του Ρεκανου απο τον Ιανουαριο, ας τα ποσταρει
Υπαρχουν καποια λυμένα στις ασκήσεις του μαθήματος με μικρά λάθη .
https://www.thmmy.gr/smf/index.php?topic=67316.msg1193630#msg1193630
Το θέμα 1 έχει σίγουρα λάθος γιατί έχει ξεχάσει ένα z^-1 στο δεύτερο κομμάτι.


Title: Re: [ΨΕΣ] Λύσεις παλιών θεμάτων
Post by: georgkonst on August 31, 2017, 20:10:22 pm
Μία πρόταση για το 4ο Θέμα του Ιανουαρίου (courtesy of konkerama)



Title: Re: [ΨΕΣ] Λύσεις παλιών θεμάτων
Post by: SportBillyPap on August 31, 2017, 20:26:48 pm
Μία πρόταση για το 4ο Θέμα του Ιανουαρίου (courtesy of konkerama)


Καλή λύση θα έπρεπε να γράψεις και ότι ROC{|z|<1} αλλα λεπτομέρεια .Επίσης νομίζω ότι έκανες πολύ δουλειά για μια μονάδα :P να γράψεις απλά λ*a^n*u(-n-1) για τους 3 πόλους αρκούσε .


Title: Re: [ΨΕΣ] Λύσεις παλιών θεμάτων
Post by: georgkonst on August 31, 2017, 20:31:01 pm
Καλή λύση θα έπρεπε να γράψεις και ότι ROC{|z|<1} αλλα λεπτομέρεια .Επίσης νομίζω ότι έκανες πολύ δουλειά για μια μονάδα :P να γράψεις απλά λ*a^n*u(-n-1) για τους 3 πόλους αρκούσε .

Νομίζω ότι πρέπει να το δείξεις ότι καταλήγει σε πραγματικό, δεν είναι ξεκάθαρο ότι απαλοίφονται τα μιγαδικά. Επίσης άκουσα ότι κόβει αβέρτα οπότε just in case, αν και δεν πιστεύω ότι είναι εφικτό να βγει αυτό στην ώρα της εξέτασης.

Σωστός για το roc, ty


Title: Re: [ΨΕΣ] Λύσεις παλιών θεμάτων
Post by: Ap.Mor. on January 30, 2018, 13:49:13 pm
Μια λύση για το 1o θέμα του Σεπτεμβρίου του '17.
Κάποια πιο "γρήγορη" λύση ίσως ήταν καλύτερη...αν έχει κάποιος άλλος καμιά ιδέα :P


Title: Re: [ΨΕΣ] Λύσεις παλιών θεμάτων
Post by: abadasa13 on February 01, 2018, 21:25:52 pm
Μια λύση για το 1o θέμα του Σεπτεμβρίου του '17.
Κάποια πιο "γρήγορη" λύση ίσως ήταν καλύτερη...αν έχει κάποιος άλλος καμιά ιδέα :P

Αν δεις σελίδα 73 του βιβλίου του Hayes έχει παρόμοιο παράδειγμα που το λύνει διαφορετικά (χωρίς μετασχηματισμό z).
Προσπαθώ αυτή τη στιγμή να το λύσω έτσι, αν το καταφέρω θα το ανεβάσω.


Title: Re: [ΨΕΣ] Λύσεις παλιών θεμάτων
Post by: kekonstan on February 02, 2018, 23:21:43 pm
Μια λύση για το 4ο θέμα Γενάρη του 17 με ολοκληρωτικά υπόλοιπα (πολύ απλή, αλλά πιάνει και μια μονάδα  :D)


Title: Re: [ΨΕΣ] Λύσεις παλιών θεμάτων
Post by: κύριος Φασόλης on February 03, 2018, 13:15:31 pm
Μια λύση για το 4ο θέμα Γενάρη του 17 με ολοκληρωτικά υπόλοιπα (πολύ απλή, αλλά πιάνει και μια μονάδα  :D)


μπορει να πω βλακεια, αλλα μηπως επειδη σου λεει οτι ειναι αντιαιτιατο πρεπει να παρεις και πολο στο 0 επειδη το n σου θα ειναι <0 ?


Title: Re: [ΨΕΣ] Λύσεις παλιών θεμάτων
Post by: valid on February 03, 2018, 20:50:00 pm
για να παρεις πολο στο 0 πρεπει στη Χ(z)*z^(n-1) να υπαρχει ορος 1/z
ειναι λαθος ομως η λυση γιατι το ROC ειναι |z|<1 οποτε δεν υπαρχουν πολοι εντος του ROC


Title: Re: [ΨΕΣ] Λύσεις παλιών θεμάτων
Post by: princess_of_the_dawn on February 03, 2018, 23:06:51 pm
Υπαρχουν καποια λυμένα στις ασκήσεις του μαθήματος με μικρά λάθη .
https://www.thmmy.gr/smf/index.php?topic=67316.msg1193630#msg1193630
Το θέμα 1 έχει σίγουρα λάθος γιατί έχει ξεχάσει ένα z^-1 στο δεύτερο κομμάτι.
Έχει λύσει κανείς άλλος το 1ο θέμα; Μήπως έχει ξεχάσει ένα 1/4 στις λύσεις;


Title: Re: [ΨΕΣ] Λύσεις παλιών θεμάτων
Post by: ablaoublas on February 04, 2018, 00:49:40 am
Για το 1ο Θέμα του Φεβρουαρίου 2017:

Στο τελευταίο ερώτημα,

Έστω ότι βρίσκω κανονικά την Y(z) εφαρμόζοντας μονόπλευρο Μ/Τ  Ζ για να λάβω υπόψη μου τις αρχικές συνθήκες στην αρχική εξίσωση διαφορών...

Πως γίνεται να βρω την y(n) χωρίς να ξέρω την περιοχή σύγκλισης της Y(z) ;


EDIT: Είναι γνωστή τελικά η περιοχή σύγκλισης: RoCx τομή RoCy


Title: Re: [ΨΕΣ] Λύσεις παλιών θεμάτων
Post by: κύριος Φασόλης on February 04, 2018, 13:53:06 pm
Για το 1ο Θέμα του Φεβρουαρίου 2017:
...
Πως γίνεται να βρω την y(n) χωρίς να ξέρω την περιοχή σύγκλισης της Y(z) ;


EDIT: Είναι γνωστή τελικά η περιοχή σύγκλισης: RoCx τομή RoCy

Μηπως μπορεις να μου εξηγησεις πως καταλαβαινεις οτι ειναι γνωστη?


Επισης, εχει κανεις καμια ιδεα για θεμα 3ο απο σεπτεμβρη 17 ? Ξερω οτι ειναι στη σελιδα 159 του Πανα αλλα ακομα κι ετσι δε νιωθω τι πρεπει να κανω...
εδιτ: υπαρχει λυμενο στις φετινες σημειωσεις σελ. 46-47 του 2ου παρτ.


Title: Re: [ΨΕΣ] Λύσεις παλιών θεμάτων
Post by: ablaoublas on February 04, 2018, 16:57:14 pm
Μηπως μπορεις να μου εξηγησεις πως καταλαβαινεις οτι ειναι γνωστη?


Επισης, εχει κανεις καμια ιδεα για θεμα 3ο απο σεπτεμβρη 17 ? Ξερω οτι ειναι στη σελιδα 159 του Πανα αλλα ακομα κι ετσι δε νιωθω τι πρεπει να κανω...
εδιτ: υπαρχει λυμενο στις φετινες σημειωσεις σελ. 46-47 του 2ου παρτ.

Από την μορφή της x(n) προκύπτει ότι RoCx: |z|>1/8 από το προηγούμενο ερώτημα ξέρεις  RoCh: |z|>1/2 άρα RoCy=RoCx τομή RoCh :
|z| > 1/2


Για το RoCh: |z|>1/2 βασικά δεν είμαι και τόσο σίγουρος, μπορεί να είναι <1/2.. Ισως πρέπει να πάρεις περιπτώσεις. Ξέρει κανείς τι παίζει τελικά ; Μήπως συμπλήρωσε την εκφώνηση μέσα στην τάξη; Π.χ. το σύστημα είναι αιτιατό;


Title: Re: [ΨΕΣ] Λύσεις παλιών θεμάτων
Post by: persephonee on February 04, 2018, 18:16:05 pm
Για το RoCh: |z|>1/2 βασικά δεν είμαι και τόσο σίγουρος, μπορεί να είναι <1/2.. Ισως πρέπει να πάρεις περιπτώσεις. Ξέρει κανείς τι παίζει τελικά ; Μήπως συμπλήρωσε την εκφώνηση μέσα στην τάξη; Π.χ. το σύστημα είναι αιτιατό;
αφού η y(n) εξαρτάται μόνο από την τρέχουσα είσοδο και προγενέστερες τιμές της εξόδου ( y(n-1) και y(n-2) )  το σύστημα είναι αιτιατό.


Title: Re: [ΨΕΣ] Λύσεις παλιών θεμάτων
Post by: ablaoublas on February 04, 2018, 18:32:06 pm
αφού η y(n) εξαρτάται μόνο από την τρέχουσα είσοδο και προγενέστερες τιμές της εξόδου ( y(n-1) και y(n-2) )  το σύστημα είναι αιτιατό.

Οκ ήταν προφανές τελικά, δεν χρειαζόταν να δοθεί κάτι στην εκφώνηση :P


Title: Re: [ΨΕΣ] Λύσεις παλιών θεμάτων
Post by: SaltyMeatBalls on February 04, 2018, 19:02:51 pm
θέμα 4ο σεπτέμβριος 2017 καμια ιδεα;


Title: Re: [ΨΕΣ] Λύσεις παλιών θεμάτων
Post by: Argirios on February 04, 2018, 19:04:37 pm
Μια λύση για το 4ο θέμα Γενάρη του 17 με ολοκληρωτικά υπόλοιπα (πολύ απλή, αλλά πιάνει και μια μονάδα  :D)

οι ρίζες του (z^2-2z+4) δεν είναι πόλοι? γιατί μόνο στο z=1?


Title: Re: [ΨΕΣ] Λύσεις παλιών θεμάτων
Post by: gal on February 04, 2018, 19:17:34 pm
θέμα 4ο σεπτέμβριος 2017 καμια ιδεα;

60 και 240 εβγαλα εγω


Title: Re: [ΨΕΣ] Λύσεις παλιών θεμάτων
Post by: gal on February 04, 2018, 19:20:33 pm
οι ρίζες του (z^2-2z+4) δεν είναι πόλοι? γιατί μόνο στο z=1?

συμφωνω ! και εγω με ολοκληρωτικα το εκανα αλλα πηρα και τους δυο πολλους που λες


Title: Re: [ΨΕΣ] Λύσεις παλιών θεμάτων
Post by: kokkinos drakos ths zhxal on February 04, 2018, 20:38:06 pm
φεβρουαριος 17 θεμα 4
θελει αντιταιτιατο, αυτο σημαινει οτι θελιες abs(z)<1
για ν>=0 δεν εχεις πολους
για ν<0 εχεις πολο πολλαπλοτητας ν+1
θετεις ζ=1/ρ και κανεις res
(οσον αφορα το αντιαιτιατο και το abs(z)<1 τα εχεις στις περσινες σημειωσεις)


Title: Re: [ΨΕΣ] Λύσεις παλιών θεμάτων
Post by: κύριος Φασόλης on February 04, 2018, 21:27:30 pm
60 και 240 εβγαλα εγω

μηπως μπορεις να ανεβασεις τη λυση σου?


Title: Re: [ΨΕΣ] Λύσεις παλιών θεμάτων
Post by: Μουργόλυκος on February 04, 2018, 22:01:52 pm
μηπως μπορεις να ανεβασεις τη λυση σου?

Σεπτέμβριος 2017, Θέμα 4ο

Εγώ έβγαλα 30 και 120. Επισυνάπτω τη λύση μου. Βασίζεται στην περιοδικότητα της ej2πn/N. Δηλαδή e-j2πkn/N=ej2π(N-k)n/N και έτσι συσχετίζω τα περιεχόμενα για k=200, 2200 και k=800, 1600


Title: Re: [ΨΕΣ] Λύσεις παλιών θεμάτων
Post by: Μουργόλυκος on February 04, 2018, 22:04:21 pm
Σεπτέμβριος 2017, Θέμα 4ο

Εγώ έβγαλα 30 και 120. Επισυνάπτω τη λύση μου. Βασίζεται στην περιοδικότητα της ej2πn/N. Δηλαδή e-j2πkn/N=ej2π(N-k)n/N και έτσι συσχετίζω τα περιεχόμενα για k=200, 2200 και k=800, 1600

Αν ζητούσε τις υψηλότερες δυνατές συχνότητες θα εξέφραζα τον όρο 200 με βάση τον 2200 και τον 800 με βάση τον 1600.


Title: Re: [ΨΕΣ] Λύσεις παλιών θεμάτων
Post by: christina_ on February 04, 2018, 22:12:29 pm
Και γω το ίδιο εβγαλα αλλά δεν ξέρω κατά πόσο είναι σωστός ο τρόπος ή απλά έτυχε.


Title: Re: [ΨΕΣ] Λύσεις παλιών θεμάτων
Post by: christina_ on February 04, 2018, 22:15:46 pm
Το πήγα με βάση το pdf


Title: Re: [ΨΕΣ] Λύσεις παλιών θεμάτων
Post by: ablaoublas on February 04, 2018, 22:34:48 pm
Congratz και στους δύο πάντως... Λογικά ο Ρέκανος θα ήθελε με βάση το pdf του.


Title: Re: [ΨΕΣ] Λύσεις παλιών θεμάτων
Post by: ablaoublas on February 04, 2018, 22:53:11 pm
Και γω το ίδιο εβγαλα αλλά δεν ξέρω κατά πόσο είναι σωστός ο τρόπος ή απλά έτυχε.

Το ελάχιστες πιθανές συχνότητες f1,f2 στην εκφώνηση, χρησιμοποιείται ώστε να κάνουμε τις επιλογές:

f1= 30 ή 330 Hz (συμμετρικές ως προς Fs) άρα για ελάχιστη f1 τότε f1=30 Hz

f2= 120 ή 240 Hz (συμμετρικές ως προς Fs) άρα για ελάχιστη f2 τότε f2=120 Hz

Εκεί χρησιμεύει αυτό το δεδομένο;


Title: Re: [ΨΕΣ] Λύσεις παλιών θεμάτων
Post by: christina_ on February 04, 2018, 23:07:31 pm
Για να ισχύει ο Nyquist πρέπει Fs>2fmax (νομίζω αυτό εννοείς)


Title: Re: [ΨΕΣ] Λύσεις παλιών θεμάτων
Post by: ablaoublas on February 05, 2018, 00:08:39 am
Για να ισχύει ο Nyquist πρέπει Fs>2fmax (νομίζω αυτό εννοείς)

Α ναι σωστά, εφόσον είναι "καθαρό" το φάσμα, σημαίνει ότι Fs>2f1,2f2 άρα απορρίπτονται οι συχνότητες 330,240Hz. Άρα τελικά το δεδομένο της εκφώνησης μικρότερες πιθανές τιμές των f1,f2 που χρησιμεύει στην επίλυση της άσκησης;


Title: Re: [ΨΕΣ] Λύσεις παλιών θεμάτων
Post by: SaltyMeatBalls on February 05, 2018, 00:10:23 am
Α ναι σωστά, εφόσον είναι "καθαρό" το φάσμα, σημαίνει ότι Fs>2f1,2f2 άρα απορρίπτονται οι συχνότητες 330,240Hz. Άρα τελικά το δεδομένο της εκφώνησης μικρότερες πιθανές τιμές των f1,f2 που χρησιμεύει στην επίλυση της άσκησης;

ε ουσιαστικα σε βοηθαει, σε περιπτωση που δεν σκεφτεις απο μόνος σου οτι ικανοποιειται το κριτηριο Nyquist


Title: Re: [ΨΕΣ] Λύσεις παλιών θεμάτων
Post by: christina_ on February 05, 2018, 00:18:13 am
Θα μας δώσει τυπολόγιο για τους τύπους των φίλτρων Butterworth ανάλογα με την τάξη τους;


Title: Re: [ΨΕΣ] Λύσεις παλιών θεμάτων
Post by: SaltyMeatBalls on February 05, 2018, 00:23:28 am
Θα μας δώσει τυπολόγιο για τους τύπους των φίλτρων Butterworth ανάλογα με την τάξη τους;

το πολυωνυμο χρειαζεσαι μονο και εχει στα βιβλια πινακα με τους συντελεστες αναλογα με την ταξη


Title: Re: [ΨΕΣ] Λύσεις παλιών θεμάτων
Post by: thanospr on June 22, 2018, 12:28:47 pm
Καποιος που εχει λυσει τα θεματα Φεβρουαριου του 18;


Title: Re: [ΨΕΣ] Λύσεις παλιών θεμάτων
Post by: electric67 on June 24, 2018, 00:23:50 am
Καποιος που εχει λυσει τα θεματα Φεβρουαριου του 18;
+1


Title: Re: [ΨΕΣ] Λύσεις παλιών θεμάτων
Post by: K on June 24, 2018, 12:52:37 pm
Καποιος που εχει λυσει τα θεματα Φεβρουαριου του 18;

+1


Title: Re: [ΨΕΣ] Λύσεις παλιών θεμάτων
Post by: gkyriazt on June 24, 2018, 14:38:23 pm
Αυτή είναι μια δικιά μου προσπάθεια.. Λείπει το Θέμα 3. Γενικά σίγουρα σωστό είναι μόνο το 2 και μάλλον το 4.
οποιος έχει καμιά ιδέα για το 3 ας πει.

 https://www.dropbox.com/sh/ixm8haa98rnrswg/AADgvwYxwE9YiAN08UrBp-IYa?dl=0 (https://www.dropbox.com/sh/ixm8haa98rnrswg/AADgvwYxwE9YiAN08UrBp-IYa?dl=0)


Title: Re: [ΨΕΣ] Λύσεις παλιών θεμάτων
Post by: Chandler on June 24, 2018, 15:51:57 pm
Για το θέμα 3 λες ότι επειδή είναι το σήμα x[n] πραγματικό και αιτιατό τότε x[n]=2*xe[n] και το xe[n] είναι ολοκλήρωμα 1/2*π από -π μέχρι π του Χreal(e^iω)*e^j*n*ω dω


Title: Re: [ΨΕΣ] Λύσεις παλιών θεμάτων
Post by: gkyriazt on June 24, 2018, 17:36:41 pm
Για το θέμα 3 λες ότι επειδή είναι το σήμα x[n] πραγματικό και αιτιατό τότε x[n]=2*xe[n] και το xe[n] είναι ολοκλήρωμα 1/2*π από -π μέχρι π του Χreal(e^iω)*e^j*n*ω dω

Ευχαριστώ ούτε μεθαύριο δεν θα το έβρισκα αυτές τις ιδιότητες του Fourier τις εχω ξεχάσει για τα καλά.


Title: Re: [ΨΕΣ] Λύσεις παλιών θεμάτων
Post by: Chandler on June 24, 2018, 18:00:54 pm
Και το θέμα 4 του 2018


Title: Re: [ΨΕΣ] Λύσεις παλιών θεμάτων
Post by: Ζak Μckracken on June 24, 2018, 18:05:57 pm
Και το θέμα 4 του 2018

Σωραιος!!!


Title: Re: [ΨΕΣ] Λύσεις παλιών θεμάτων
Post by: MrRobot on June 24, 2018, 18:11:12 pm
Σωραιος!!!

Σωραία *


Title: Re: [ΨΕΣ] Λύσεις παλιών θεμάτων
Post by: Chandler on June 24, 2018, 18:12:29 pm
ΝΑ ΣΑΙ ΚΑΛΑ !! ;D ;D ;D ;D ;D


Title: Re: [ΨΕΣ] Λύσεις παλιών θεμάτων
Post by: Argirios on June 24, 2018, 18:13:21 pm
Και το θέμα 4 του 2018
nice, γιατι κανεις επι u(n-1) στο τελος?


Title: Re: [ΨΕΣ] Λύσεις παλιών θεμάτων
Post by: Chandler on June 24, 2018, 18:15:38 pm
Γιατί θέλω y(0)=0  8))


Title: Re: [ΨΕΣ] Λύσεις παλιών θεμάτων
Post by: Argirios on June 24, 2018, 18:17:56 pm
Γιατί θέλω y(0)=0  8))
Ααα μαλιστα, θενξ!  :)


Title: Re: [ΨΕΣ] Λύσεις παλιών θεμάτων
Post by: K on June 24, 2018, 19:03:36 pm
Αυτή είναι μια δικιά μου προσπάθεια.. Λείπει το Θέμα 3. Γενικά σίγουρα σωστό είναι μόνο το 2 και μάλλον το 4.
οποιος έχει καμιά ιδέα για το 3 ας πει.

https://www.dropbox.com/s/76qikm91lkbm4ev/Pses_2018.rar?dl=0 (https://www.dropbox.com/s/76qikm91lkbm4ev/Pses_2018.rar?dl=0)

Στη σχεδίαση του φίλτρου, δε μπορούμε να πάρουμε μια τιμή για το ΔΤ που να μας βολεύει;
Σύμφωνα με τον Hayes σελ.493 το ΔΤ ή Τs όπως το ονομάζει, δε συμμετέχει στη σχεδίαση.

Επίσης στον Hayes σελίδα 73, παράδειγμα 1.38 καταλαβαίνει κανείς πώς βρίσκει την κρουστική απόκριση;


Title: Re: [ΨΕΣ] Λύσεις παλιών θεμάτων
Post by: Ζak Μckracken on June 24, 2018, 22:41:44 pm
ΝΑ ΣΑΙ ΚΑΛΑ !! ;D ;D ;D ;D ;D

Και τόσο καλογραμμένο ... από κοπέλα θα ήταν


Title: Re: [ΨΕΣ] Λύσεις παλιών θεμάτων
Post by: Ζak Μckracken on June 24, 2018, 23:07:01 pm
Και το θέμα 4 του 2018

ρε συ τυπος: y(n)=(1/2)^(n)*n*(n-1)  είναι σωστός;;; δεν βγάζει τα y[]=.. που υπολογισες


Title: Re: [ΨΕΣ] Λύσεις παλιών θεμάτων
Post by: Argirios on June 24, 2018, 23:13:01 pm
ρε συ τυπος: y(n)=(1/2)^(n)*n*(n-1)  είναι σωστός;;; δεν βγάζει τα y[]=.. που υπολογισες

γράφει y(n)=(1/2)^(n-1)*u(n-1)  αν δε κάνω λάθος


Title: Re: [ΨΕΣ] Λύσεις παλιών θεμάτων
Post by: Ζak Μckracken on June 24, 2018, 23:26:09 pm
γράφει y(n)=(1/2)^(n-1)*u(n-1)  αν δε κάνω λάθος

δικιο εχεις!!


Title: Re: [ΨΕΣ] Λύσεις παλιών θεμάτων
Post by: Argirios on June 24, 2018, 23:52:27 pm
όταν ζητάει να βρούμε εξίσωση διαφοράς από μπλόκ διάγραμμα όπως Σεπ 2015 3.α τί κάνουμε? υπάρχει κάποιο παράδειγμα στο βιβλίο?


Title: Re: [ΨΕΣ] Λύσεις παλιών θεμάτων
Post by: electric67 on June 24, 2018, 23:52:35 pm
Στο θέμα 3 φεβρουαριος 2018 βγαίνει για αποτέλεσμα     x(n) = 2 / (n * π)   , n>0  ;


Title: Re: [ΨΕΣ] Λύσεις παλιών θεμάτων
Post by: Chandler on June 25, 2018, 01:59:12 am
x[n]=sin (n*π/2)/(n*π/2) για n>0 βγάζει αν θυμάμαι καλά. Θα το δώ πάλι αύριο να το ανεβάσω.


Title: Re: [ΨΕΣ] Λύσεις παλιών θεμάτων
Post by: Chandler on June 25, 2018, 12:47:17 pm
Το θέμα 3


Title: Re: [ΨΕΣ] Λύσεις παλιών θεμάτων
Post by: Ζak Μckracken on June 25, 2018, 13:07:53 pm
Το θέμα 3

Είσαι Θεά!! δώσε τροφή στο λαό σου!


Title: Re: [ΨΕΣ] Λύσεις παλιών θεμάτων
Post by: Ragnar Lothbrok on June 25, 2018, 13:35:39 pm
Ρε παιδια μπορει καποιος να ανεβασει λύση απο Θέμα 3 Σεπτέβριο 2017; εχει πεσει 2 φορες.


Title: Re: [ΨΕΣ] Λύσεις παλιών θεμάτων
Post by: Chandler on June 25, 2018, 14:36:05 pm
το έχει λύσει ο ρέκανος στην τάξη . τσέκαρε το zip αρχείο στα downloads : dps_shmeiwseis_2017 το part 2 σελ 46


Title: Re: [ΨΕΣ] Λύσεις παλιών θεμάτων
Post by: Ζak Μckracken on June 25, 2018, 16:03:39 pm
παίζει λύση για θέμα 1ο ΣΕΠΤ 17 και για θεμα 1ο ΦΕΒ 18;;;;


Title: Re: [ΨΕΣ] Λύσεις παλιών θεμάτων
Post by: Ragnar Lothbrok on June 25, 2018, 20:47:00 pm
παίζει λύση για θέμα 1ο ΣΕΠΤ 17 και για θεμα 1ο ΦΕΒ 18;;;;

+1


Title: Re: [ΨΕΣ] Λύσεις παλιών θεμάτων
Post by: Ζak Μckracken on June 25, 2018, 22:22:16 pm
παίζει λύση για θέμα 1ο ΣΕΠΤ 17 και για θεμα 1ο ΦΕΒ 18;;;;

η λύσεις είναι η άσκηση σελ 270 του hayes αν δεν κανω λαθος;;;


Title: Re: [ΨΕΣ] Λύσεις παλιών θεμάτων
Post by: electric67 on June 26, 2018, 13:05:32 pm
Το θέμα 3

σ ευχαριστούμε πολύ :)


Για το θέμα 1 φεβ. 2018, το έλυσα με βάση τη μέθοδο στη σελίδα 73 του hayes και βρήκα h(n) = 1.618n - 0.618n . συμφωνείτε;


Title: Re: [ΨΕΣ] Λύσεις παλιών θεμάτων
Post by: Ζak Μckracken on June 26, 2018, 19:41:29 pm
σ ευχαριστούμε πολύ :)


Για το θέμα 1 φεβ. 2018, το έλυσα με βάση τη μέθοδο στη σελίδα 73 του hayes και βρήκα h(n) = 1.618n - 0.618n . συμφωνείτε;

νομίζω ειναι από το 5ο κεφάλαιο αυτες οι ασκησεις


Title: Re: [ΨΕΣ] Λύσεις παλιών θεμάτων
Post by: Ζak Μckracken on June 26, 2018, 19:49:54 pm
αν μπορεί κάποιος λύση για θέμα 1ο ΣΕΠΤ 17 και για θεμα 1ο ΦΕΒ 18;;;;


Title: Re: [ΨΕΣ] Λύσεις παλιών θεμάτων
Post by: Argirios on June 26, 2018, 20:19:22 pm
Το πρώτο Σεπτεμβρίου 2017, με δική σας ευθύνη.  :P
(αυτό που ζητάει την έξοδο δε το τελείωσα είχε πολλές πράξεις και βαριέμαι, αλλά είναι ίδια λογική με την κρουστική απόκριση απλά πιο δύσκολο, ίσως να υπάρχει και άλλος τρόπος.)


Title: Re: [ΨΕΣ] Λύσεις παλιών θεμάτων
Post by: Chandler on June 26, 2018, 23:45:21 pm
Έχεις ξεχάσει ότι έχεις αρχικές συνθήκες μη μηδενικές που πρέπει να μπουν στον μετασχηματισμό z


Title: Re: [ΨΕΣ] Λύσεις παλιών θεμάτων
Post by: Argirios on June 26, 2018, 23:52:39 pm
Έχεις ξεχάσει ότι έχεις αρχικές συνθήκες μη μηδενικές που πρέπει να μπουν στον μετασχηματισμό z
δε τις θεωρουμε μηδενικες οταν ψαχνουμε κρουστικη αποκριση?


Title: Re: [ΨΕΣ] Λύσεις παλιών θεμάτων
Post by: AB_Kingbird on June 27, 2018, 00:30:28 am
σ ευχαριστούμε πολύ :)


Για το θέμα 1 φεβ. 2018, το έλυσα με βάση τη μέθοδο στη σελίδα 73 του hayes και βρήκα h(n) = 1.618n - 0.618n . συμφωνείτε;

Εγώ αυτό έκανα, αλλά δεν είμαι και σίγουρος, ακόμα μαθαίνω! Επίσης το ότι είναι ευσταθές, σε τι μας βοηθάει;

π.σ. η Η(z) βρίσκεται κάπου εκεί


Title: Re: [ΨΕΣ] Λύσεις παλιών θεμάτων
Post by: Chandler on June 27, 2018, 00:31:37 am
Απλά είχες ξεχάσει και τον όρο -0.75y(-1) και νόμιζα δε τις έβαλες καθόλου  ::) Ίσως θα ήταν καλύτερα Μόλις βρείτε την κρουστικη αποκριση να κάνετε αντίστροφο μετασχηματισμό z με τις αρχικές συνθήκες αυτή τη φορά στην εξίσωση διαφορών . Τον όρο που οφείλεται στις μη μηδενικές αρχικές συνθήκες τον μετατρέπεται κανονικά από z σε n αλλά τον όρο με το h(n) Κάντε y(n)=x(n)συνελιξη h(n) . Θα βγει με αθροίσματα εκθετικων σειρών . Στο τέλος τα αθροιζεται όλα μαζί .

σαν συνταγή μαγειρικής


Title: Re: [ΨΕΣ] Λύσεις παλιών θεμάτων
Post by: ilesgidi on June 27, 2018, 00:43:29 am
Η ευσταθεια λεει οτι στην ROC Θα πρεπει να περιεχεται ο μοναδιαιος κυκλος , πιο απλα αμα δεις τον παρανομαστη της H(z) θα πρεπει να εχει (z-a) με a <1{ROC : IZI<1} δηλαδη (στην λυση σου εχεις Izi>1.61 και Izi>0,615 οποτε Roc IzI>1.61 αρα και δεν ειναι ευσταθες). Τουλαχιστον αυτο καταλαβα .. :P


Title: Re: [ΨΕΣ] Λύσεις παλιών θεμάτων
Post by: Argirios on June 27, 2018, 00:52:43 am
Απλά είχες ξεχάσει και τον όρο -0.75y(-1) και νόμιζα δε τις έβαλες καθόλου  ::) Ίσως θα ήταν καλύτερα Μόλις βρείτε την κρουστικη αποκριση να κάνετε αντίστροφο μετασχηματισμό z με τις αρχικές συνθήκες αυτή τη φορά στην εξίσωση διαφορών . Τον όρο που οφείλεται στις μη μηδενικές αρχικές συνθήκες τον μετατρέπεται κανονικά από z σε n αλλά τον όρο με το h(n) Κάντε y(n)=x(n)συνελιξη h(n) . Θα βγει με αθροίσματα εκθετικων σειρών . Στο τέλος τα αθροιζεται όλα μαζί .

σαν συνταγή μαγειρικής
ωχ ναι τώρα το είδα  ;)


Title: Re: [ΨΕΣ] Λύσεις παλιών θεμάτων
Post by: Chandler on June 27, 2018, 01:08:30 am
Στο θέμα 1 Φεβρουαρίου του 2018 πρέπει στο δεύτερο υποερωτημα να τροποποιήσεις την h(n) ώστε στην περιοχή σύγκλισης να ανήκει ο μοναδιαίος κύκλος (η περιοχή σύγκλισης είναι ανάμεσα στους δύο πόλους) . Έπειτα βρισκεις την H(z) με αντίστροφο μετασχηματισμό z.


Title: Re: [ΨΕΣ] Λύσεις παλιών θεμάτων
Post by: ilesgidi on June 27, 2018, 01:16:07 am
Στο θέμα 1 Φεβρουαρίου του 2018 πρέπει στο δεύτερο υποερωτημα να τροποποιήσεις την h(n) ώστε στην περιοχή σύγκλισης να ανήκει ο μοναδιαίος κύκλος (η περιοχή σύγκλισης είναι ανάμεσα στους δύο πόλους) . Έπειτα βρισκεις την H(z) με αντίστροφο μετασχηματισμό z.
Τι εννοεις εδω ποιητρια??? :P :P πως κανω τροποποιηση στην h(n)?


Title: Re: [ΨΕΣ] Λύσεις παλιών θεμάτων
Post by: Chandler on June 27, 2018, 01:29:40 am
Κάτι τέτοιο πρέπει να είναι με κάθε επιφύλαξη όμως!!


Title: Re: [ΨΕΣ] Λύσεις παλιών θεμάτων
Post by: ilesgidi on June 27, 2018, 01:51:42 am
Να σε ρωτησω : η βηματικη σου στο u(-n-1) ειναι σιγουρα σωστη?  γιατι μου φαινεται οτι θα επρεπε να ηταν u(n-1)
Εκτως απο αυτο γιατι μου φαινεται οτι παλι ROC εχω για IZI>-1.61?  T.T



Σαν να βρηκα αυτο που εκανες στις σημειωσεις ρεκανου 2016-2017(σελ 33-35)  και εχει μια αποδειξη αυτου που χρησιμοποιησες με διαφορετικη φορα ολοκληρωσης του επικαμπυλιου (στην ουσια ειμουν αρκετα λαθος , λες θετω z=1/ρ και αυτο οδηγει σε εναν πολο για ρ=1/b οπου b ειναι αυτος με το μεγαλυτερο μετρο).


Title: Re: [ΨΕΣ] Λύσεις παλιών θεμάτων
Post by: ilesgidi on June 27, 2018, 12:03:53 pm
Λοιπόν εχεις δικιο και μαλλον για ολα τα πιθανα σεναρια λύνεται κάπως έτσι : αλλα μετα η μετατροπη σε H(z) ποια θα ειναι?


Title: Re: [ΨΕΣ] Λύσεις παλιών θεμάτων
Post by: kakashi on June 27, 2018, 20:26:20 pm
Λοιπόν εχεις δικιο και μαλλον για ολα τα πιθανα σεναρια λύνεται κάπως έτσι : αλλα μετα η μετατροπη σε H(z) ποια θα ειναι?

Νομίζω ότι έχεις κάποια λάθη στα πρόσημα εκεί στο 0.45 στην αρχή (κάπως τρως το -). Επίσης νομίζω ότι είναι -(-0.61)n u(n) αλλά δεν είμαι και σίγουρος.
Τέλος νομίζω ότι η H(Z) που ζητάει είναι ίδια ακριβώς με αυτή του πρώτου ερωτήματος.Απλά αλλάζει η ROC της.Έχει πει στο μάθημα ότι μπορεί μια Η(Ζ) να έχει διαφορετικές h(n) με διαφορετική ROC.


Title: Re: [ΨΕΣ] Λύσεις παλιών θεμάτων
Post by: ilesgidi on June 27, 2018, 22:07:57 pm
οντως εχω αρκετα λαθη αλλα ειμουν μεσα στην απογνωση μπας και μου πει κανενας χριστιανος τι να κανω  :D :D
Υπαρχει κανενας να ελυσε το θεμα 4 του φεβρουαριου 2017 ?


Title: Re: [ΨΕΣ] Λύσεις παλιών θεμάτων
Post by: Kthulu on June 27, 2018, 22:24:43 pm
Τελικά, π.χ. στο 1ο θέμα Σεπτ '16, όταν έχουμε ΓΕΔΣΣ και ζητάει κρουστική απόκριση, συμπεριλαμβάνουμε τις αρχικές τιμές ή όχι;


Title: Re: [ΨΕΣ] Λύσεις παλιών θεμάτων
Post by: ilesgidi on June 27, 2018, 22:33:44 pm
δεν σε επιασα πρωτο θεμα του 2016 σεπτ?


Title: Re: [ΨΕΣ] Λύσεις παλιών θεμάτων
Post by: Kthulu on June 28, 2018, 04:02:49 am
Συγγνώμη, εννοούσα Σεπτ '17


Title: Re: [ΨΕΣ] Λύσεις παλιών θεμάτων
Post by: ilesgidi on June 28, 2018, 11:01:53 am
Εγώ πήρα από τα παιδιά λύσεις και έγραψα αυτο


Title: Re: [ΨΕΣ] Λύσεις παλιών θεμάτων
Post by: ilesgidi on June 28, 2018, 11:52:13 am
Πάντως για να μην σε πάρω και στον λαιμό μου η λογική είναι να το λύσεις αρχικά χωρίς τιμές στο y(-1) , y(-2) και χωρίς την συνάρτηση x(n) υπόψη σου και να βρεις την h(n) και μετά να το ξανακανροεις πάλι με τις τιμές των παραπάνω .Για να είσαι καλυμμένος θα μπορούσες να γράψεις κάπου στο βιβλίο για να μην ψαχνεσαι για τον μετασχηματισμό που έχει πολλούς IzI>1 και για διπλό πόλο . :P :P


Title: Re: [ΨΕΣ] Λύσεις παλιών θεμάτων
Post by: kakashi on June 28, 2018, 12:46:33 pm
Πάντως για να μην σε πάρω και στον λαιμό μου η λογική είναι να το λύσεις αρχικά χωρίς τιμές στο y(-1) , y(-2) και χωρίς την συνάρτηση x(n) υπόψη σου και να βρεις την h(n) και μετά να το ξανακανροεις πάλι με τις τιμές των παραπάνω .Για να είσαι καλυμμένος θα μπορούσες να γράψεις κάπου στο βιβλίο για να μην ψαχνεσαι για τον μετασχηματισμό που έχει πολλούς IzI>1 και για διπλό πόλο . :P :P

Γενικά σωστή λογική.Τα μόνα λάθη που βρήκα είναι στους υπολογισμούς του B(z) και Γ(z).
Για το Β αρκεί το z^2 στον αριθμητή να το κάνεις z*z και το ένα να το πας στο πρώτο μέλος.Να κάνεις μερικά κλάσματα και αφού κάνεις την μετατροπή στην h(n) να θέσεις όπου n=n+1 λόγω του ότι όταν πας το z ξανά πίσω στο 2ο μέλος εμφανίζεται ως "επιτάχυνση"(το αντίστροφο της καθυστέρησης  :D)
Για το Γ, διαλέγεις να πας το z στο πρώτο μέλος (σωστός) αλλά μετά το σπάσιμο σε κλάσματα πρέπει να το επιστρέψεις ξανά στο 2ο μέλος για να φτιάξεις σωστά τα κλάσματα και μετά να πάρεις την h(n).Αν το κάνεις έτσι θα δεις ότι δεν υπάρχει κάποια καθυστέρηση τύπου z^-1.

Ελπίζω και εγώ να μην είμαι λάθος χαχα.


Title: Re: [ΨΕΣ] Λύσεις παλιών θεμάτων
Post by: kakashi on June 28, 2018, 12:55:22 pm
Ουσιαστικά (έτσι όπως το έχω καταλάβει εγώ) αν στο Γ(z) δεν πήγαινες στο πρώτο μέλος της εξίσωσης το z πριν το σπάσιμο σε μερικά κλάσματα τότε αυτό θα εξαφανιζόταν κατά την διαδικασία του σπασίματος. Επομένως μετά θα έπρεπε να το βάλεις με το ζόρι για να βολέψει η μετατροπή, και άρα τότε θα έμπαινε και ένα z^-1 για καθυστέρηση. Απλά σε αυτή την περίπτωση θα ήταν και διαφορετικά τα κλάσματα, δηλαδή με άλλους συντελεστές.

Οπότε κάνουμε ότι μας βολεύει πιο πολύ κάθε φορά. Και οι δύο τρόποι υποθέτω ότι θα βγάζουν ίδιο αποτέλεσμα αν και δεν το δοκίμασα :P

edit: επειδή δεν είναι ίδιο το αποτέλεσμα τλκ (λογικό αφού έχουν αλλάξει οι συντελεστές) και δεν ξέρω αν ο δεύτερος τρόπος είναι σωστός, καλύτερα κάνε τον πρώτο που μεταφέρεις το z στο πρώτο μέλος πριν το σπάσιμο σε μερικά κλάσματα.


Title: Re: [ΨΕΣ] Λύσεις παλιών θεμάτων
Post by: ilesgidi on June 28, 2018, 13:30:49 pm
Μπορεις να το στειλεις σε φοτο ?? ξερω το λεω τελευταια στιγμη! :D :D


Title: Re: [ΨΕΣ] Λύσεις παλιών θεμάτων
Post by: ilesgidi on June 28, 2018, 13:40:11 pm
Στον υπολογισμό τους ειμουν λάθος αλά εν τέλη τα έβαλα σωστά εγώ αυτά βρίσκω και πες μου άμα εχω πέσει έξω :P
btw αυτο που λες(επιταχυνση γινεται για διπλο πολο (p.x.    (z-a)^2    ) οποτε νομιζω δεν βρισκεις εδω κατι τετοιο (και παλι δεν ειμαι 100% σιγουρος )   


Title: Re: [ΨΕΣ] Λύσεις παλιών θεμάτων
Post by: kakashi on June 28, 2018, 13:48:17 pm
Μπορεις να το στειλεις σε φοτο ?? ξερω το λεω τελευταια στιγμη! :D :D

Πολύ πρόχειρα.

Στην δεύτερη έχω δοκιμάσει να κρατήσω το z στο δεύτερο μέλος για να το τσεκάρω.


Title: Re: [ΨΕΣ] Λύσεις παλιών θεμάτων
Post by: ilesgidi on June 28, 2018, 14:00:18 pm
Πολύ πρόχειρα.

Στην δεύτερη έχω δοκιμάσει να κρατήσω το z στο δεύτερο μέλος για να το τσεκάρω.

Άρα στο B(z) βγάζουμε το ίδιο τώρα που διωρθωθηκε αλλά στο Γ(z) έχει διαφορετικές λύσεις ?? Κοίτα επειδή νομίζω είναι και οι δύο σωστές ίσως να ήταν καλύτερα να μην υπάρχει n-1 για να τα αρθροισουμε στο τέλος.


Title: Re: [ΨΕΣ] Λύσεις παλιών θεμάτων
Post by: Pest on September 09, 2018, 19:21:49 pm
Παιδια μπορει καποιος να ανεβασει τη λυση απ' το θεμα 2 Φεβρ. 18? ( εχει απαντηθει σε ενα  dropbox link αλλα δε λειτουργει πλεον)


Title: Re: [ΨΕΣ] Λύσεις παλιών θεμάτων
Post by: Wu on September 11, 2018, 17:40:33 pm
επίσης μπορεί κάποιος να ανεβάσει λύσεις για Θ3 Σεπτεμβρη 17 και Θ5 φεβρουάριος 17

και για Σ.17 Θ4

ή που μπορω να τα βρω στο βιβλίο του hayes


Title: Re: [ΨΕΣ] Λύσεις παλιών θεμάτων
Post by: Pest on September 11, 2018, 17:57:29 pm
επίσης μπορεί κάποιος να ανεβάσει λύσεις για Θ3 Σεπτεμβρη 17 και Θ5 φεβρουάριος 17

και για Σ.17 Θ4

ή που μπορω να τα βρω στο βιβλίο του hayes
Για τα forward differences τσεκαρε τις σημειωσεις dsp_17_part2 σελ. 46. Ειναι τα 2 πρωτα θεματα που ρωτας.


Title: Re: [ΨΕΣ] Λύσεις παλιών θεμάτων
Post by: chasiotis on December 21, 2018, 13:43:17 pm
Ξέρει κάποιος κανένα καλό τυπολόγιο ;


Title: Re: [ΨΕΣ] Λύσεις παλιών θεμάτων
Post by: gdims on January 18, 2019, 23:39:58 pm
Η λύση μου στο θέμα 1 του Σεπτέμβρη 2018.
Την έχω επαληθεύσει για n > 0 και δυστυχώς αποτυγχάνει στο n=0.
Αν κάποιος ξέρει γιατί, ας μου πει!

EDIT: λογικά χρειάζεται u(n) στον πρώτο όρο και u(n-1) στον δεύτερο όρο του αθροίσματος της λύσης...


Title: Re: [ΨΕΣ] Λύσεις παλιών θεμάτων
Post by: kons on January 24, 2019, 22:49:17 pm
Η λύση μου στο θέμα 1 του Σεπτέμβρη 2018.
Την έχω επαληθεύσει για n > 0 και δυστυχώς αποτυγχάνει στο n=0.

Έχεις κάνει ουσιαστικά την περίπτωση για n>=0

Στην περίπτωση n<0 ο αριθμητής z^n γίνεται πόλος πολλαπλότητας |n|, οπότε πρέπει να αθροίσεις και το ολοκληρωτικό υπόλοιπο γι' αυτόν.

(και μάλλον θα αναιρείται με το άλλο ολ. υπόλοιπο οπότε θα βγαίνει 0 για n<0)


Title: Re: [ΨΕΣ] Λύσεις παλιών θεμάτων
Post by: gdims on January 26, 2019, 21:46:35 pm
Στο θέμα 2 του Σεπτέμβρη 2018 έδωσα την λύση που επισυνάπτω.
Δεν είμαι καθόλου σίγουρος για το αν είναι σωστή. Μπορεί κάποιος να βοηθήσει?


Title: Re: [ΨΕΣ] Λύσεις παλιών θεμάτων
Post by: empargio on January 31, 2019, 14:57:50 pm
Για τα butterworth θα μας δίνει τυπολογιο;


Title: Re: [ΨΕΣ] Λύσεις παλιών θεμάτων
Post by: MrRobot on January 31, 2019, 15:07:40 pm
Γραφουμε ετσι κιαλλιως με ανοιχτα βιβλια, που έχουν μέσα τους τύπους


Title: Re: [ΨΕΣ] Λύσεις παλιών θεμάτων
Post by: gdims on January 31, 2019, 15:28:48 pm
Έχετε βρει τους τύπους στο Hayes?


Title: Re: [ΨΕΣ] Λύσεις παλιών θεμάτων
Post by: Mr Xaxas on February 01, 2019, 02:31:06 am
εχει κανεις λυση για το θεμα 4 του Ιανουριου του 2017 ???


Title: Re: [ΨΕΣ] Λύσεις παλιών θεμάτων
Post by: empargio on February 01, 2019, 18:07:56 pm
Νομίζω θέλει να πεις ότι επειδή είναι αντιαιτιατο |z,|<1  άρα ο μόνος πόλος που σου μένει να κάνει ολοκληρωτικό υπόλοιπο είναι ο 0 και επειδή έχεις πολλαπλότητα n αντικαθιστας z=1/ρ και βρίσκεις x(n)


Title: Re: [ΨΕΣ] Λύσεις παλιών θεμάτων
Post by: empargio on February 01, 2019, 18:10:06 pm
Εγώ θέλω να ρωτήσω για το θέμα 1 του Σεπτέμβρη. 2017..μπορούμε να χρησιμοποιήσουμε ολοκληρωτικά υπόλοιπα μιας και έχουμε ζ^2 στον αριθμητή;; Για να αποφύγουμε πόλλες τις πράξεις; Γενικά τα ολοκληρωτικά υπόλοιπα μπορώ να τα χρησιμοποιώ αντί να υπολογίζω A και Β?


Title: Re: [ΨΕΣ] Λύσεις παλιών θεμάτων
Post by: λευκός θόρυβος on February 01, 2019, 18:11:58 pm
Εγώ θέλω να ρωτήσω για το θέμα 1 του Σεπτέμβρη. 2017..μπορούμε να χρησιμοποιήσουμε ολοκληρωτικά υπόλοιπα μιας και έχουμε ζ^2 στον αριθμητή;; Για να αποφύγουμε πόλλες τις πράξεις; Γενικά τα ολοκληρωτικά υπόλοιπα μπορώ να τα χρησιμοποιώ αντί να υπολογίζω A και Β?


Φανταζομαι πως δεν υπαρχει προβλημα.Το δοκιμασα και με τους 2 τροπους και καταληγω στο ιδιο αποτελεσμα


Title: Re: [ΨΕΣ] Λύσεις παλιών θεμάτων
Post by: empargio on February 01, 2019, 18:26:56 pm
Πόσο σου βγήκε; Εμένα Μ βγήκε αυτό...


Title: Re: [ΨΕΣ] Λύσεις παλιών θεμάτων
Post by: Mr Xaxas on February 01, 2019, 18:35:47 pm
Νομίζω θέλει να πεις ότι επειδή είναι αντιαιτιατο |z,|<1  άρα ο μόνος πόλος που σου μένει να κάνει ολοκληρωτικό υπόλοιπο είναι ο 0 και επειδή έχεις πολλαπλότητα n αντικαθιστας z=1/ρ και βρίσκεις x(n)

αφου κανεις την αντιστροφη .ROC προκυπτει |p|>1 και παιρνεις res και για τους 3 πολους ετσι ???


Title: Re: [ΨΕΣ] Λύσεις παλιών θεμάτων
Post by: empargio on February 01, 2019, 18:40:19 pm
Τι πόλους βρήκες; Ναι έτσι ειναι αλλά εμένα οι δύο πόλοι Μ βγήκαν μικρότεροι της μοναδας


Title: Re: [ΨΕΣ] Λύσεις παλιών θεμάτων
Post by: empargio on February 01, 2019, 18:41:25 pm
Τα μέτρα τους δηλαδη


Title: Re: [ΨΕΣ] Λύσεις παλιών θεμάτων
Post by: Mr Xaxas on February 01, 2019, 19:07:07 pm
οι πολοι του z εχουν μετρα 1 , και 2 συζηγεις μιγαδικους με μετρο 2 ...Οποτε μετα την αντιστροφη εχουν μετρο 1 και 1/2 οι αλλοι δυο .Θεωρω ομως οτι αφου τωρα ROC |p|>1 στα RES θα πρεπει να παρεις και τους 3 πολους !!!!


Title: Re: [ΨΕΣ] Λύσεις παλιών θεμάτων
Post by: empargio on February 01, 2019, 19:21:52 pm
Ναι κ μένα έτσι Μ βγήκαν...εφόσον |p|>1 δε πρέπει αν αποκλειστούν αυτοί ο έχουν μέτρα 1/2? Η όχι; Νομίζω ότι πλέον έχεις έναν νέο κύκλο στον οποίο ανήκει πλέον μόνο ο ένας...αλλ δεν είμαι κ σιγουρη


Title: Re: [ΨΕΣ] Λύσεις παλιών θεμάτων
Post by: L on February 01, 2019, 19:24:24 pm
Ναι κ μένα έτσι Μ βγήκαν...εφόσον |p|>1 δε πρέπει αν αποκλειστούν αυτοί ο έχουν μέτρα 1/2? Η όχι;


αφού περικλείονται...
κάνε σχέδιο με κλειστή καμπύλη C εντός της ROC που να περικλείει την αρχή των αξόνων (με ωρολογιακή φορά εδώ)


Title: Re: [ΨΕΣ] Λύσεις παλιών θεμάτων
Post by: λευκός θόρυβος on February 02, 2019, 01:43:06 am
Θέμα 5 Σεπτέμβρη 17. Χρησιμοποιώ 2 τρόπους για να βρω την h(n) μια με τη μέθοδο των ολοκληρωτικων υπολοίπων και μια με τη μέθοδο των κλασματων και βγάζω διαφορετικό αποτέλεσμα. Γνωρίζει κανείς γιατι;


Title: Re: [ΨΕΣ] Λύσεις παλιών θεμάτων
Post by: chrimili on February 02, 2019, 02:31:29 am
έχω την εντύπωση πως είπε ότι ειναι δυνατο να βγουν διαφορετικα αποτελεσματα αν το πας με διαφορετικο τροπο..αλλα αν δουμε καλυτερα τα δυο ειναι τελικα ιδια.κατι τετοιο


Title: Re: [ΨΕΣ] Λύσεις παλιών θεμάτων
Post by: L on February 02, 2019, 13:26:48 pm
Θέμα 5 Σεπτέμβρη 17. Χρησιμοποιώ 2 τρόπους για να βρω την h(n) μια με τη μέθοδο των ολοκληρωτικων υπολοίπων και μια με τη μέθοδο των κλασματων και βγάζω διαφορετικό αποτέλεσμα. Γνωρίζει κανείς γιατι;

στη μέθοδο με τα κλάσματα αντί να θέσεις n←n-1 θέσε n←(-n-1) και κάνε μερικές πράξεις


Title: Re: [ΨΕΣ] Λύσεις παλιών θεμάτων
Post by: λευκός θόρυβος on February 02, 2019, 13:35:16 pm
στη μέθοδο με τα κλάσματα αντί να θέσεις n←n-1 θέσε n←(-n-1) και κάνε μερικές πράξεις

Ουσιαστικα μου λες οτι προκυπτει το ιδιο; γιατι να θεσω (-n-1); Αφου στο H(z) εχω z^(-1)


Title: Re: [ΨΕΣ] Λύσεις παλιών θεμάτων
Post by: L on February 02, 2019, 13:52:59 pm
Ουσιαστικα μου λες οτι προκυπτει το ιδιο; γιατι να θεσω (-n-1); Αφου στο H(z) εχω z^(-1)

ναι προφανώς σε εκείνο το σημείο σωστά το θέτεις (ήθελα απλά να δείξω ότι βγαίνει το ίδιο*)
υποθέτω πως οι δύο όροι έχουν κάποια συμμετρία και συμβαίνει αυτό που συμβαίνει (θα είδες υποθέτω ως τώρα ότι δίνοντάς τιμές και στις δύο περιπτώσεις παίρνεις τα ίδια αποτελέσματα)
σκέψου επίσης ότι το αρχικό κλάσμα θα μπορούσες να το σπάσεις σε (2/3)* z/(z-2)  -  (2/3)* z/(z-1/2) και θα σου βγει το ίδιο με τα υπόλοιπα

*edit: Άμα στο αποτέλεσμα των κλασμάτων θέσεις όπου n το -n θα σου προκύψει η συνάρτηση που βρήκες στα υπόλοιπα. Μετά είναι θέμα παρατηρητικότητας ότι και για τα δύο αποτελέσματα ισχύει h[n]=h[-n]


Title: Re: [ΨΕΣ] Λύσεις παλιών θεμάτων
Post by: λευκός θόρυβος on February 02, 2019, 14:03:26 pm
ναι προφανώς σε εκείνο το σημείο σωστά το θέτεις (ήθελα απλά να δείξω ότι βγαίνει το ίδιο)
υποθέτω πως οι δύο όροι έχουν κάποια συμμετρία και συμβαίνει αυτό που συμβαίνει (θα είδες υποθέτω ως τώρα ότι δίνοντάς τιμές και στις δύο περιπτώσεις παίρνεις τα ίδια αποτελέσματα)
σκέψου επίσης ότι το αρχικό κλάσμα θα μπορούσες να το σπάσεις σε (2/3)* z/(z-2)  -  (2/3)* z/(z-1/2) και θα σου βγει το ίδιο με τα υπόλοιπα

Εχεις δικιο.Οποτε δεν υπαρχει καποιος περιορισμος ως προς τον ποιο τροπο χρησιμοποιουμε; ( ολοκληρωτικα υπολοιπα ή κλασματα )


Title: Re: [ΨΕΣ] Λύσεις παλιών θεμάτων
Post by: L on February 02, 2019, 14:15:14 pm
Εχεις δικιο.Οποτε δεν υπαρχει καποιος περιορισμος ως προς τον ποιο τροπο χρησιμοποιουμε; ( ολοκληρωτικα υπολοιπα ή κλασματα )
όλα ισχύουν (εφόσον ξέρεις τι κάνεις  :P)
τα υπόλοιπα είναι imba σε δύσκολες περιπτώσεις, αλλά θέλουν προσοχή, π.χ. εδώ

Η λύση μου στο θέμα 1 του Σεπτέμβρη 2018.
Την έχω επαληθεύσει για n > 0 και δυστυχώς αποτυγχάνει στο n=0.
άμα δεν πάρεις την περίπτωση στα υπόλοιπα για n=0 (extra πόλος) είναι να βαράς το κεφάλι σου  :D

απορία: άμα κάνουμε υπόλοιπα το δέχεται να το αφήσουμε σαν παράγωγο ή πρέπει πάντα να κάνουμε αυτό με την αντιστροφή;




Title: Re: [ΨΕΣ] Λύσεις παλιών θεμάτων
Post by: neniak on February 02, 2019, 21:19:33 pm
Στο θέμα 1 του Σεπτεμβρίου 2018,προσπαθώντας να βρω την h(n) για να εφαρμόσω το y(n) = x(n) * h(n),μετασχηματίζω την σχέση που μας δίνεται κατά Z και βρίσκω το H(Z).Το θέμα μου είναι ότι στο παρονομαστή το τριώνυμο βγάζει αρνητική διακρίνουσα και δεν μπορώ να το παραγοντοποιήσω για να εφαρμόσω IZT και να βρω το h(n) κατά τα γνωστά. Γνωρίζει κανείς τι κάνω σε αυτές τις περιπτώσεις;


Title: Re: [ΨΕΣ] Λύσεις παλιών θεμάτων
Post by: λευκός θόρυβος on February 02, 2019, 21:26:14 pm
Στο θέμα 1 του Σεπτεμβρίου 2018,προσπαθώντας να βρω την h(n) για να εφαρμόσω το y(n) = x(n) * h(n),μετασχηματίζω την σχέση που μας δίνεται κατά Z και βρίσκω το H(Z).Το θέμα μου είναι ότι στο παρονομαστή το τριώνυμο βγάζει αρνητική διακρίνουσα και δεν μπορώ να το παραγοντοποιήσω για να εφαρμόσω IZT και να βρω το h(n) κατά τα γνωστά. Γνωρίζει κανείς τι κάνω σε αυτές τις περιπτώσεις;

Μπορεις να το παραγοντοποιησεις.απλως οι λυσεις του πολυωνυμου θα βγουν συζυγεις μιγαδικες ριζες .


Title: Re: [ΨΕΣ] Λύσεις παλιών θεμάτων
Post by: TheoProt on February 03, 2019, 01:36:52 am
Τα θέματα με τις εξισώσεις διαφορών μπορούμε να τα λύσουμε σαν να είχαμε διαφορικές εξισώσεις ; Όπως η λύση που έχω στην εικόνα ;

https://www.dropbox.com/s/nu4dntm8a0gq01y/Photo%2003-02-2019%2C%2000%2029%2041.jpg?dl=0

Και η λύση στις κάτω φωτογραφίες για το θέμα 3 Φεβρουάριος 2018 είναι οκ ;


Title: Re: [ΨΕΣ] Λύσεις παλιών θεμάτων
Post by: λευκός θόρυβος on February 03, 2019, 03:53:02 am
Eχει λυσει καποιος το θεμα 4 του Φεβρουαριου 2017;


Title: Re: [ΨΕΣ] Λύσεις παλιών θεμάτων
Post by: TheoProt on February 03, 2019, 03:57:26 am
Eχει λυσει καποιος το θεμα 4 του Φεβρουαριου 2017;
Έχω κάνει αυτό αλλά μετά δεν ξέρω πώς να συνεχίσω.


Title: Re: [ΨΕΣ] Λύσεις παλιών θεμάτων
Post by: empargio on February 03, 2019, 10:29:01 am
Σε μια άσκηση που έλυσε φέτος ο ρεκανος φενεται να αλλάζουν και η περιοχή σύγκλισης και η C όταν κάνει την αντιστροφη ζ=1/ρ και μετά του μένει μόνο ένας πόλος αντί για δύο...όταν λες ωρολογιακη φορά τι θα αλλάξει ως προς του πόλους; Αφού η επριοχη σύγκλισης θα είναι p>1?
αφού περικλείονται...
κάνε σχέδιο με κλειστή καμπύλη C εντός της ROC που να περικλείει την αρχή των αξόνων (με ωρολογιακή φορά εδώ)


Title: Re: [ΨΕΣ] Λύσεις παλιών θεμάτων
Post by: empargio on February 03, 2019, 10:32:15 am
Η καμπύλη C είναι από το |Ζ| και μέσα;


Title: Re: [ΨΕΣ] Λύσεις παλιών θεμάτων
Post by: empargio on February 03, 2019, 10:55:44 am
Τα θέματα με τις εξισώσεις διαφορών μπορούμε να τα λύσουμε σαν να είχαμε διαφορικές εξισώσεις ; Όπως η λύση που έχω στην εικόνα ;

https://www.dropbox.com/s/nu4dntm8a0gq01y/Photo%2003-02-2019%2C%2000%2029%2041.jpg?dl=0

Και η λύση στις κάτω φωτογραφίες για το θέμα 3 Φεβρουάριος 2018 είναι οκ ;



Νομίζω στη λύση σου στο ντρομποξ έχεις ένα λάθος στις διάφορες των δ...εxεις x(n) =δ(n-1) -2δ(n-1) άρα το y(0) βγαίνει 0 και ούτω καθεξής...και για να συμπεριλάβεις το 0 στη λύση σου κάνεις n->n-1...το θέμα είναι ότι βγαίνει αρκετά διαφορετικό και μένα έτσι Μ βγήκε...


Title: Re: [ΨΕΣ] Λύσεις παλιών θεμάτων
Post by: Mr Xaxas on February 03, 2019, 11:25:42 am
Έχω κάνει αυτό αλλά μετά δεν ξέρω πώς να συνεχίσω.
Το δευτερο κλασμα εχει τριωνυμο που βγαζει μιγαδικες ριζες.Δεν σε ενοχλει εσενα οποτε θα πρεπει να σπασεις και το δευτερο κλασμα σε απλα και μετα βγαινει οπως ολα .


Title: Re: [ΨΕΣ] Λύσεις παλιών θεμάτων
Post by: Spiral on February 03, 2019, 15:25:08 pm
Πως πάμε απο το s plane sto z plane με γραφική παράσταση?


Title: Re: [ΨΕΣ] Λύσεις παλιών θεμάτων
Post by: TheoProt on February 03, 2019, 15:47:57 pm


Νομίζω στη λύση σου στο ντρομποξ έχεις ένα λάθος στις διάφορες των δ...εxεις x(n) =δ(n-1) -2δ(n-1) άρα το y(0) βγαίνει 0 και ούτω καθεξής...και για να συμπεριλάβεις το 0 στη λύση σου κάνεις n->n-1...το θέμα είναι ότι βγαίνει αρκετά διαφορετικό και μένα έτσι Μ βγήκε...

Για να συμπεριλάβω το 0, θέτω στη λύση που βρήκα n = n-1 και λέω ότι η τελική λύση είναι αυτή με n-1 ; Και νομίζω ότι είναι πιθανό να βγαίνουν διαφορετικές παραστάσεις αλλά να προκύπτουν ίδια αποτελέσματα απο αυτές!


Title: Re: [ΨΕΣ] Λύσεις παλιών θεμάτων
Post by: empargio on February 03, 2019, 16:53:59 pm
Για να συμπεριλάβω το 0, θέτω στη λύση που βρήκα n = n-1 και λέω ότι η τελική λύση είναι αυτή με n-1 ; Και νομίζω ότι είναι πιθανό να βγαίνουν διαφορετικές παραστάσεις αλλά να προκύπτουν ίδια αποτελέσματα απο αυτές!

ναι η τελικη λυση ειναι αυτη με το n-1 ετσι ωστε οταν θεσεις n=0 το σημα να σου δωσει y(0)=0...


Title: Re: [ΨΕΣ] Λύσεις παλιών θεμάτων
Post by: λευκός θόρυβος on February 03, 2019, 17:18:53 pm
ναι η τελικη λυση ειναι αυτη με το n-1 ετσι ωστε οταν θεσεις n=0 το σημα να σου δωσει y(0)=0...

Σε ολες τις περιπτωσεις πρεπει να θεσω το n->n-1?


Title: Re: [ΨΕΣ] Λύσεις παλιών θεμάτων
Post by: vag178 on February 03, 2019, 20:29:13 pm
Στο θέμα 2 Σεπτέμβρης του 2018 παίρνουμε συχνότητα αποκοπής αυτή που προκύπτει από τα 15 KHz ή πρέπει να βρούμε άλλη για τα -3 db; Αν ναι ποίο τύπο παίρνουμε;


Title: Re: [ΨΕΣ] Λύσεις παλιών θεμάτων
Post by: chrimili on February 03, 2019, 21:29:42 pm
Γενικά νομίζω το φίλτρο μπορείς να το σχεδιάσεις με βάση τις συχνότητες που σου δίνει απλά μετά όταν πας από το αναλογικό στο ψηφιακό θα πρέπει να ξέρεις την συχνότητα αποκοπής για να την βάλεις στον τύπο του φίλτρου..βάζω την λύση στο συνημμένο χωρίς να ξέρω αν είναι σωστη


Title: Re: [ΨΕΣ] Λύσεις παλιών θεμάτων
Post by: Singularity on February 03, 2019, 21:32:22 pm
Στο 2ο θέμα Σεπτέμβριος 2018 πως χειριζόμαστε την απροσδιοριστία του tan(π/2) κατα τον υπολογισμό των νέου Ω_2' ;


Title: Re: [ΨΕΣ] Λύσεις παλιών θεμάτων
Post by: laserscout on February 03, 2019, 22:02:43 pm
Στο 2ο θέμα Σεπτέμβριος 2018 πως χειριζόμαστε την απροσδιοριστία του tan(π/2) κατα τον υπολογισμό των νέου Ω_2' ;

Η προδιαγραφή λέει απόσβεση για Ω>30KHz.
Θέτεις Ω_2 = 29KHz, οπότε είσαι εντός των προδιαγραφών και φεύγει η απροσδιοριστία.


Title: Re: [ΨΕΣ] Λύσεις παλιών θεμάτων
Post by: vag178 on February 03, 2019, 22:46:43 pm
Έλυσε κανένας το θέμα 3 από Σεπτέμβρη 2018; 


Title: Re: [ΨΕΣ] Λύσεις παλιών θεμάτων
Post by: chrimili on February 03, 2019, 22:55:06 pm
Νομίζω έτσι ειναι


Title: Re: [ΨΕΣ] Λύσεις παλιών θεμάτων
Post by: L on February 04, 2019, 01:57:25 am
Σε μια άσκηση που έλυσε φέτος ο ρεκανος φενεται να αλλάζουν και η περιοχή σύγκλισης και η C όταν κάνει την αντιστροφη ζ=1/ρ και μετά του μένει μόνο ένας πόλος αντί για δύο...όταν λες ωρολογιακη φορά τι θα αλλάξει ως προς του πόλους; Αφού η επριοχη σύγκλισης θα είναι p>1?
οι νέοι πόλοι έχουν μέτρο <=1
η νεα ROC είναι για > 1
άρα οποιαδήποτε C εντός της ROC (ενν. κλειστή κ περικλείει το 0) περικλείει όλους τους νέους πόλους

στις ασκήσεις με τον Διγραμμικό ο Ρέκανος στο pdf του παίρνει Ωc'=Ω1, ενώ σε μια λύση παραπίσω γίνεται χρήση κάποιων τύπων... μπορεί κανείς να πει τι παίζει;

nvm το βρηκα


Title: Re: [ΨΕΣ] Λύσεις παλιών θεμάτων
Post by: empargio on February 04, 2019, 11:02:23 am
οι νέοι πόλοι έχουν μέτρο <=1
η νεα ROC είναι για > 1
άρα οποιαδήποτε C εντός της ROC (ενν. κλειστή κ περικλείει το 0) περικλείει όλους τους νέους πόλους

στις ασκήσεις με τον Διγραμμικό ο Ρέκανος στο pdf του παίρνει Ωc'=Ω1, ενώ σε μια λύση παραπίσω γίνεται χρήση κάποιων τύπων... μπορεί κανείς να πει τι παίζει;

nvm το βρηκα

Αα οκκ σε ευχαριστώ πολύ!!


Title: Re: [ΨΕΣ] Λύσεις παλιών θεμάτων
Post by: empargio on February 04, 2019, 11:22:13 am
Σε ολες τις περιπτωσεις πρεπει να θεσω το n->n-1?

Αν χρειάζεσαι το 0 σα λυση ναι...αλλά μπορεί κάπου να μη σου βγαίνει το y(0) να είναι 0 ...ανάλογα τι εχεις


Title: Re: [ΨΕΣ] Λύσεις παλιών θεμάτων
Post by: TheoProt on February 04, 2019, 11:40:09 am
Αντιαιτιατό σήμα σημαίνει : x(n)=0 για n>=0 ;


Title: Re: [ΨΕΣ] Λύσεις παλιών θεμάτων
Post by: vag178 on February 04, 2019, 11:44:37 am
Αντιαιτιατό σήμα σημαίνει : x(n)=0 για n>=0 ;

ναι


Title: Re: [ΨΕΣ] Λύσεις παλιών θεμάτων
Post by: katerinap99 on June 16, 2019, 19:17:03 pm
Θεμα 5 Φεβ 19 υπαρχει καποια λυση?


Title: Re: [ΨΕΣ] Λύσεις παλιών θεμάτων
Post by: clementine on June 19, 2019, 16:25:41 pm
Επειδή δεν τα βρίσκω στα downloads , είναι εύκολο να με καθοδηγήσει κάποιος για το πού βρίσκονται τα παλιά θέματα ;


Title: Re: [ΨΕΣ] Λύσεις παλιών θεμάτων
Post by: Niri on June 19, 2019, 16:55:13 pm
Επειδή δεν τα βρίσκω στα downloads , είναι εύκολο να με καθοδηγήσει κάποιος για το πού βρίσκονται τα παλιά θέματα ;

Αν εννοεις οτι δεν βρισκεις τα downloads του μαθηματος τοτε ειναι εδω:
https://www.thmmy.gr/smf/index.php?action=tpmod;dl=cat99
Για αλλα θεματα εκτος απο τα downloads δεν ξερω να σου πω.


Title: Re: [ΨΕΣ] Λύσεις παλιών θεμάτων
Post by: blackmirror on June 20, 2019, 15:58:41 pm
Θεμα 5 Φεβ 19 υπαρχει καποια λυση?
αυτή πρεπει να είναι η λύση για την εξίσωση διαφορών. καμια ιδεα για το δεύτερο κομμάτι με την είσοδο;


Title: Re: [ΨΕΣ] Λύσεις παλιών θεμάτων
Post by: MarPap on June 22, 2019, 20:46:02 pm
Για το θέμα 2 του Φλεβάρη 19΄, έχετε κάποια λύση?


Title: Re: [ΨΕΣ] Λύσεις παλιών θεμάτων
Post by: varvarip on June 22, 2019, 21:33:56 pm
Για το 2ο Θέμα Σεπτέμβρη του '18 και γενικά για τα φίλτρα που η εφαπτομένη απειρίζεται ποια είναι η σωστότερη προσέγγιση;; Επισυνάπτω μία, αλλά έχω διαβάσει και ότι απλά επιλέγουμε μία ελαφρώς μικρότερη συχνότητα από αυτήν που απειρίζεται για να απαλείψουμε τον απειρισμό και συνεχίζουμε κανονικά.


Title: Re: [ΨΕΣ] Λύσεις παλιών θεμάτων
Post by: ροζ θορυβος on June 22, 2019, 21:43:14 pm
Για το 2ο Θέμα Σεπτέμβρη του '18 και γενικά για τα φίλτρα που η εφαπτομένη απειρίζεται ποια είναι η σωστότερη προσέγγιση;; Επισυνάπτω μία, αλλά έχω διαβάσει και ότι απλά επιλέγουμε μία ελαφρώς μικρότερη συχνότητα από αυτήν που απειρίζεται για να απαλείψουμε τον απειρισμό και συνεχίζουμε κανονικά.
νομιζω το χειμώνα που έδωσα απλά επέλεξα συχνότητα και μου το πήρε οκ.


Title: Re: [ΨΕΣ] Λύσεις παλιών θεμάτων
Post by: unfo on June 23, 2019, 09:45:43 am
νομιζω το χειμώνα που έδωσα απλά επέλεξα συχνότητα και μου το πήρε οκ.

Αφού πάρουμε ελαφρώς μικρότερη συχνότητα και απαλείψουμε τον απειρισμό μετά χρειάζεται να βρούμε ακριβώς το Ωc από τον τύπο
Ω1 / (10^(-Κ1/10) - 1)^ (1/2n)    <=    Ωc    <=   Ω2 / (10^(-Κ2/10) - 1)^ (1/2n)  ? Επίσης ο προηγούμενος τύπος θέλει τα αρχικά Ω ή τα αποστρεβλωμένα Ω' ?

Ρωτάω γιατί στο pdf του Ρέκανου απλά λέει Ω1' = Ωc' και απλά συνεχίζει χωρίς να κάνει τα παραπάνω.


Title: Re: [ΨΕΣ] Λύσεις παλιών θεμάτων
Post by: BIC on June 23, 2019, 17:25:55 pm
Για το 2ο Θέμα Σεπτέμβρη του '18 και γενικά για τα φίλτρα που η εφαπτομένη απειρίζεται ποια είναι η σωστότερη προσέγγιση;; Επισυνάπτω μία, αλλά έχω διαβάσει και ότι απλά επιλέγουμε μία ελαφρώς μικρότερη συχνότητα από αυτήν που απειρίζεται για να απαλείψουμε τον απειρισμό και συνεχίζουμε κανονικά.

Το Ωc το καινούργιο γιατί το υπολογίζεις με τις αρχικές αναλογικές συχνότητες και όχι τις αποστρεβλωμένες?


Title: Re: [ΨΕΣ] Λύσεις παλιών θεμάτων
Post by: varvarip on June 23, 2019, 18:20:34 pm
Το Ωc το καινούργιο γιατί το υπολογίζεις με τις αρχικές αναλογικές συχνότητες και όχι τις αποστρεβλωμένες?

Το είχα κάνει σαν λύση για τον απειρισμό. Αλλά τελικά το σωστό είναι να επιλέξεις μία μικρότερη συχνότητα.


Title: Re: [ΨΕΣ] Λύσεις παλιών θεμάτων
Post by: BIC on June 23, 2019, 18:35:01 pm
Το είχα κάνει σαν λύση για τον απειρισμό. Αλλά τελικά το σωστό είναι να επιλέξεις μία μικρότερη συχνότητα.

Που βρήκες ότι η λύση είναι να πάρεις απλά μία μικρότερη συχνότητα, είναι κάπου στις σημειώσεις ή στα φυλλάδια?


Title: Re: [ΨΕΣ] Λύσεις παλιών θεμάτων
Post by: varvarip on June 23, 2019, 18:45:20 pm
Που βρήκες ότι η λύση είναι να πάρεις απλά μία μικρότερη συχνότητα, είναι κάπου στις σημειώσεις ή στα φυλλάδια?

Αν θέλεις τέτοια πηγή δεν έχω. Απλά μίλησα με συμφοιτητές και τους το πήρε σωστό.


Title: Re: [ΨΕΣ] Λύσεις παλιών θεμάτων
Post by: BIC on June 23, 2019, 18:52:30 pm
Ευχαριστώ πολύ! Αν τύχει και βρω κάτι άλλο θα σας πω


Title: Re: [ΨΕΣ] Λύσεις παλιών θεμάτων
Post by: macvegie on June 24, 2019, 12:30:01 pm
Ξέρει κανείς για το 3ο Θέμα Φερβρουάριος 19' τι κάνουμε; Παίρνω αυτό που έχει στις σημειώσεις T<(-2A)/B  όπου A=max{Re{pi}} , B=max{|pi|^2} ή καμία σχέση;


Title: Re: [ΨΕΣ] Λύσεις παλιών θεμάτων
Post by: A Caster on June 24, 2019, 12:37:07 pm
Ξέρει κανείς για το 3ο Θέμα Φερβρουάριος 19' τι κάνουμε; Παίρνω αυτό που έχει στις σημειώσεις T<(-2A)/B  όπου A=max{Re{pi}} , B=max{|pi|^2} ή καμία σχέση;

Ναι, το μονο που με προβληματιζει εμενα ειναι αν θελει να κανουμε αυτο με τα Μαξ επιτοπου, η αν κανουμε ξεχωριστη αναλυση για καθε πολο και βρισκουμε τα διαφορα περιθώρια του T και βγαζουμε την κοινη περιοχη...κανεις καμια ιδεα?


Title: Re: [ΨΕΣ] Λύσεις παλιών θεμάτων
Post by: 00128419 on June 24, 2019, 13:00:17 pm
Όταν λέει max{Re{pi}}, εννοεί κατά απόλυτη τιμή ή όχι. Δηλαδή στο θέμα που έπεσε φεβρουάριο 19 θα επιλέγαμε το -2 ή το -3?


Title: Re: [ΨΕΣ] Λύσεις παλιών θεμάτων
Post by: A Caster on June 24, 2019, 14:31:38 pm
Όταν λέει max{Re{pi}}, εννοεί κατά απόλυτη τιμή ή όχι. Δηλαδή στο θέμα που έπεσε φεβρουάριο 19 θα επιλέγαμε το -2 ή το -3?

απο τη σχεση T < -2a / (a^2 + b^2) κρίνω οτι θελει χωρις απολυτη τιμη, δηλαδη το -2, αφου το περιοριζει περισσοτερο. Και επιπλεον βγαζει και νοημα επειδη οσο πιο κοντα στο 0, τοσο λιγοτερο ευσταθες το συστημα , αρα τοσο μικροτερη πρεπει να ειναι η Τ δειγματοληψιας...


Title: Re: [ΨΕΣ] Λύσεις παλιών θεμάτων
Post by: A Caster on June 24, 2019, 15:09:01 pm
αυτή πρεπει να είναι η λύση για την εξίσωση διαφορών. καμια ιδεα για το δεύτερο κομμάτι με την είσοδο;

Καπως ετσι το εβγαλα το 2ο μερος, αλλα μου φαινεται λιγο περιεργο. Μπορει καποιος να διπλοτσεκαρει ?


Title: Re: [ΨΕΣ] Λύσεις παλιών θεμάτων
Post by: windwaker on June 24, 2019, 15:31:55 pm
Πιστεύω είναι σωστή λογική, αλλά καλύτερα να έκανες τη συνελιξη γιατί μπλέκονται μιγαδικοί...


Title: Re: [ΨΕΣ] Λύσεις παλιών θεμάτων
Post by: unfo on June 24, 2019, 16:53:35 pm
απο τη σχεση T < -2a / (a^2 + b^2) κρίνω οτι θελει χωρις απολυτη τιμη, δηλαδη το -2, αφου το περιοριζει περισσοτερο. Και επιπλεον βγαζει και νοημα επειδη οσο πιο κοντα στο 0, τοσο λιγοτερο ευσταθες το συστημα , αρα τοσο μικροτερη πρεπει να ειναι η Τ δειγματοληψιας...


Εγώ πιστεύω ότι το -3 είναι το σωστό.
Γιατί αν πάρουμε τον κάθε πόλο ξεχωριστά (τύπος Τ < -2α / (α^2 + β^2)  έχουμε : T1 = 4/4 = 1     T2 = 4/4 = 1     T3 = 6/9 = 2/3
Άρα και από τον τύπο για πολλούς πόλους (Τ < -2Α / Β) θα πρέπει να έχουμε ίδιο αποτέλεσμα και επομένως Τ < 2/3
Αυτό που δεν καταλαβαίνω είναι για την αστάθεια και την ευστάθεια. Μέχρι στιγμής είχα καταλάβει από τις σημειώσεις ότι αν είναι Τ < αριθμού είναι ευσταθές. Άρα για Τ > αριθμού είναι ασταθές ??? Ίσως λέω χοντρές βλακείες !!!!!  :-[


Title: Re: [ΨΕΣ] Λύσεις παλιών θεμάτων
Post by: Pest on June 24, 2019, 16:57:30 pm
Εγώ πιστεύω ότι το -3 είναι το σωστό.
Γιατί αν πάρουμε τον κάθε πόλο ξεχωριστά (τύπος Τ < -2α / (α^2 + β^2)  έχουμε : T1 = 4/4 = 1     T2 = 4/4 = 1     T3 = 6/9 = 2/3
Άρα και από τον τύπο για πολλούς πόλους (Τ < -2Α / Β) θα πρέπει να έχουμε ίδιο αποτέλεσμα και επομένως Τ < 2/3
Αυτό που δεν καταλαβαίνω είναι για την αστάθεια και την ευστάθεια. Μέχρι στιγμής είχα καταλάβει από τις σημειώσεις ότι αν είναι Τ < αριθμού είναι ευσταθές. Άρα για Τ > αριθμού είναι ασταθές ??? Ίσως λέω χοντρές βλακείες !!!!!  :-[

Από τις σημειώσεις του μαθήματος αναφέρει πως το Si είναι αρνητικό, οπότε το σωστό στη συγκεκριμένη περίπτωση είναι το -2.


Title: Re: [ΨΕΣ] Λύσεις παλιών θεμάτων
Post by: unfo on June 24, 2019, 16:58:45 pm
ναι το είδα αυτό, όμως δεν θα πρέπε να έχουμε ίδια αποτελέσματα ?


Title: Re: [ΨΕΣ] Λύσεις παλιών θεμάτων
Post by: Pest on June 24, 2019, 17:04:39 pm
ναι το είδα αυτό, όμως δεν θα πρέπε να έχουμε ίδια αποτελέσματα ?
Μα ίδιο βγαίνει ρε συ. Επίσης το Τ1 είναι 1/2.


Title: Re: [ΨΕΣ] Λύσεις παλιών θεμάτων
Post by: unfo on June 24, 2019, 17:12:40 pm
Μα ίδιο βγαίνει ρε συ. Επίσης το Τ1 είναι 1/2.

ναι ναι έχεις δίκιο για το Τ1, αλλά το Τ3 βγαίνει 6/9 με τον 1ο τρόπο και 4/9 με το 2ο.
Από τον 1ο έχεις 1/2 1/2 6/9, αρα επιλέγεις 1/2 σαν μικρότερο
Από τον 2ο έχεις 4/9.
Δεν θα πρεπε να είναι ίδια ?


Title: Re: [ΨΕΣ] Λύσεις παλιών θεμάτων
Post by: Pest on June 24, 2019, 17:32:50 pm
ναι ναι έχεις δίκιο για το Τ1, αλλά το Τ3 βγαίνει 6/9 με τον 1ο τρόπο και 4/9 με το 2ο.
Από τον 1ο έχεις 1/2 1/2 6/9, αρα επιλέγεις 1/2 σαν μικρότερο
Από τον 2ο έχεις 4/9.
Δεν θα πρεπε να είναι ίδια ?
Ααα τωρα καταλαβα τι εννοείς. Οχι δε θα πρεπε να ναι ιδια απαραιτητα. Γιατι αλλιως δε θα ειχε νοημα να λεγεται πιο "αυστηρη" συνθηκη. Επισης είχε πει στην αιθουσα ο Ρεκανος αμα θελουμε να δωσουμε ολες τις διαφορετικες συνθηκες (οποτε και διαφορετικα αποτελεσματα).


Title: Re: [ΨΕΣ] Λύσεις παλιών θεμάτων
Post by: A Caster on June 24, 2019, 17:33:43 pm
ναι ναι έχεις δίκιο για το Τ1, αλλά το Τ3 βγαίνει 6/9 με τον 1ο τρόπο και 4/9 με το 2ο.
Από τον 1ο έχεις 1/2 1/2 6/9, αρα επιλέγεις 1/2 σαν μικρότερο
Από τον 2ο έχεις 4/9.
Δεν θα πρεπε να είναι ίδια ?


το μονο που με προβληματιζει εμενα ειναι αν θελει να κανουμε αυτο με τα Μαξ επιτοπου, η αν κανουμε ξεχωριστη αναλυση για καθε πολο και βρισκουμε τα διαφορα περιθώρια του T και βγαζουμε την κοινη περιοχη...κανεις καμια ιδεα?

Οκ για να απαντήσω και σε σένα αλλα και σε μενα, νομιζω έχω μια ιδέα:

Ο τρόπος με τα max που δινει T<-2A/B είναι η πιο αυστηρή συνθήκη. Δηλαδή σου δίνει ξερω γω Τ<4/9, αλλά υπάρχει και λιγο περιθωριο για μεγαλυτερες τιμες του 4/9 που ακόμα είναι ευσταθές το σύστημα. (π.χ. 4/9 = 0.444, αλλά για  0.444<Τ=0.45 < 0.5, είναι ακόμα ευσταθές)

Η παγίδα στην συγκεκριμένη άσκηση είναι η εξής:

Ζητάει το περιθώριο του Τ για το οποίο το σύστημα είναι ΑΣΤΑΘΕΣ. Άρα αν πεις Τ < 4/9 για ΕΥΣΤΑΘΕΙΑ, άρα Τ >= 4/9 για ΑΣΤΑΘΕΙΑ είναι λάθος.
Πιο συγκεκριμένα: Για Τ < 4/9 ΥΠΑΡΧΕΙ ΕΥΣΤΑΘΕΙΑ, αλλά δεν είναι ΜΟΝΟ αυτή η περιοχή...

Ισχύει ότι υπάρχει αστάθεια για Τ >= 1/2 που βγαίνει αν κάνεις ξεχωριστά κάθε πόλο με τον τύπο (-2α / α^2 + β^2).

Διορθώστε με αν κάνω λάθος


Title: Re: [ΨΕΣ] Λύσεις παλιών θεμάτων
Post by: unfo on June 24, 2019, 17:36:08 pm
Ααα τωρα καταλαβα τι εννοείς. Οχι δε θα πρεπε να ναι ιδια απαραιτητα. Γιατι αλλιως δε θα ειχε νοημα να λεγεται πιο "αυστηρη" συνθηκη. Επισης είχε πει στην αιθουσα ο Ρεκανος αμα θελουμε να δωσουμε ολες τις διαφορετικες συνθηκες (οποτε και διαφορετικα αποτελεσματα).

Ναι ΟΚ κατάλαβα τι εννοείς, και εμένα το -2 μου φαινόταν πιο λογικό απλά νόμιζα ότι έπρεπε να είναι ίδια.
Επομένως, έχουμε Τ < 4/9 και  για αστάθεια έχουμε Τ > 4/9, σωστά ?


Title: Re: [ΨΕΣ] Λύσεις παλιών θεμάτων
Post by: unfo on June 24, 2019, 18:07:54 pm
Επίσης   @Enter+Prnt+R πως σου βγήκε αυτή η H(z) ? Είσαι σίγουρος ότι είναι έτσι ?

Στο 2ο μέρος συμφωνώ μαζί σου


Title: Re: [ΨΕΣ] Λύσεις παλιών θεμάτων
Post by: andreasroko on June 24, 2019, 18:46:51 pm
Καπως ετσι το εβγαλα το 2ο μερος, αλλα μου φαινεται λιγο περιεργο. Μπορει καποιος να διπλοτσεκαρει ?


Αφου ζητάει την τιμή στην μόνιμη κατάσταση δεν θέλει ουσιαστικά το lim(y(n)) για n που τείνει στο άπειρο?Και μετα αυτό είναι ίσο με lim((1-z^-1)*Y(z)) για z που τείνει στο 1(απο σελ 22 sto pdf zt των σημειώσεων).


Title: Re: [ΨΕΣ] Λύσεις παλιών θεμάτων
Post by: A Caster on June 24, 2019, 18:56:56 pm
Επίσης   @Enter+Prnt+R πως σου βγήκε αυτή η H(z) ? Είσαι σίγουρος ότι είναι έτσι ?

Στο 2ο μέρος συμφωνώ μαζί σου

την H(Z) μπορεις να την βρεις και στην λυση του blackmirror. Αυτό που εκανα για να βολεψει ειναι διαιρεση πολυωνύμων, οπότε βγηκε το 2 απεξω και εμεινε το υπολοιπο.


Εντωμεταξυ εκανα καποια πραματα και για τα θεματα 1 εως 4 του φεβ19, αν θελετε τσεκαρετε...


Title: Re: [ΨΕΣ] Λύσεις παλιών θεμάτων
Post by: ilektraem on June 24, 2019, 19:10:16 pm
Υπάρχει καμία ιδέα για το 2ο ερώτημα του 4ου θέματος που θέλει Ωc στα -dB?


Title: Re: [ΨΕΣ] Λύσεις παλιών θεμάτων
Post by: A Caster on June 24, 2019, 19:17:57 pm
Αφου ζητάει την τιμή στην μόνιμη κατάσταση δεν θέλει ουσιαστικά το lim(y(n)) για n που τείνει στο άπειρο?Και μετα αυτό είναι ίσο με lim((1-z^-1)*Y(z)) για z που τείνει στο 1(απο σελ 22 sto pdf zt των σημειώσεων).

Καλή σκέψη, προσπάθησα να το δοκιμάσω αλλά βγαίνει λίγο περίεργο. Νομίζω το θέμα είναι ότι επειδή η είσοδος είναι περιοδική, η έξοδος δεν είναι σταθερή, δηλαδή δεν τείνει σε κάποια σταθερή τιμή. Αν δοκιμάσεις να βγάλεις το όριο και στην y(n) που έβγαλα, υπάρχουν 4 διαφορετικές τιμές λόγω του ( i^n + (-i)^n )... μόνο κάποιος που το έκανε σωστά μπορεί να μας πει τι ζητούσε εν τέλει :Ρ


Υπάρχει καμία ιδέα για το 2ο ερώτημα του 4ου θέματος που θέλει Ωc στα -dB?

Στις λύσεις π ανεβασα πριν 20 λεπτακια, επέλεξα Ω_c = 4000 r/s. Νομίζω αυτό είναι σχεδιαστικό ζήτημα και μπορείς να το επιλέξεις για να βγει μετά η H(z). Δηλαδή -3dB θα έχω για ακριβως Ω = Ω_c


Title: Re: [ΨΕΣ] Λύσεις παλιών θεμάτων
Post by: unfo on June 24, 2019, 19:51:16 pm
Στις λύσεις π ανεβασα πριν 20 λεπτακια, επέλεξα Ω_c = 4000 r/s. Νομίζω αυτό είναι σχεδιαστικό ζήτημα και μπορείς να το επιλέξεις για να βγει μετά η H(z). Δηλαδή -3dB θα έχω για ακριβως Ω = Ω_c

Εγώ πιστεύω ότι αυτό πρέπει να το βρούμε από τον τύπο με το log που βρίσκεται στο but_dif στη σελίδα 3. Δηλαδή να βάλουμε το Ω2΄ και σαν db να βάλουμε το -3. Μετά από υπολογισμούς μου βγήκε Ωc' = 8057,8 rad/s


Title: Re: [ΨΕΣ] Λύσεις παλιών θεμάτων
Post by: A Caster on June 24, 2019, 20:08:34 pm
Εγώ πιστεύω ότι αυτό πρέπει να το βρούμε από τον τύπο με το log που βρίσκεται στο but_dif στη σελίδα 3. Δηλαδή να βάλουμε το Ω2΄ και σαν db να βάλουμε το -3. Μετά από υπολογισμούς μου βγήκε Ωc' = 8057,8 rad/s

Δεν εχεις αδικο. Το 8050 ειναι εντος του ευρους τιμων που βγαζω και εγω. Ομως οταν βαζεις Ω_2', αυτο σημαινει οτι το αποτελεσμα εξαρταται αναλογα με το Ω_2 σου (δηλαδη αν επελεξες 999, 990, 950, 900 ... κλπ για να βγει το tan στην αρχη). Και επιπλεον οταν χρησιμοποιείς το Ω_2', αυτό σημαίνει οτι η αποσβεση ειναι ΑΚΡΙΒΩΣ -20dB για Ω = Ω_2'. Αν επιλέξεις μικρότερο Ω_c από αυτό που βγάζεις (π.χ. 7000) , τότε για Ω = Ω_2' θα έχεις ακομα μεγαλυτερη αποσβεση, πχ -22dB.

Εν κατακλέιδα: Αυτό π κανεις ειναι σωστό. Απλά θεωρώ κάνεις τζαμπα πολλούς υπολογισμούς ενώ υπάρχει έτοιμος τύπος (σελ 2 του but_dif.pdf)  που νομίζω είναι εξίσου σωστός.

Επιπλέον ένα επίσης μειονέκτημα: Οταν παιρνεις Ω_c = 8057,xx.. , μετά οταν κανεις την αντικατασταση για να φτασεις στο τελικο H(z) θεωρω οτι σου βγαίνει κιαλλο η πιστη με πράξεις. Σε μενα γινεται απαλοιφή.


Title: Re: [ΨΕΣ] Λύσεις παλιών θεμάτων
Post by: unfo on June 24, 2019, 20:13:20 pm
Δεν εχεις αδικο. Το 8050 ειναι εντος του ευρους τιμων που βγαζω και εγω. Ομως οταν βαζεις Ω_2', αυτο σημαινει οτι το αποτελεσμα εξαρταται αναλογα με το Ω_2 σου (δηλαδη αν επελεξες 999, 990, 950, 900 ... κλπ για να βγει το tan στην αρχη). Και επιπλεον οταν χρησιμοποιείς το Ω_2', αυτό σημαίνει οτι η αποσβεση ειναι ΑΚΡΙΒΩΣ -20dB για Ω = Ω_2'. Αν επιλέξεις μικρότερο Ω_c από αυτό που βγάζεις (π.χ. 7000) , τότε για Ω = Ω_2' θα έχεις ακομα μεγαλυτερη αποσβεση, πχ -22dB.

Εν κατακλέιδα: Αυτό π κανεις ειναι σωστό. Απλά θεωρώ κάνεις τζαμπα πολλούς υπολογισμούς ενώ υπάρχει έτοιμος τύπος (σελ 2 του but_dif.pdf)  που νομίζω είναι εξίσου σωστός.

Επιπλέον ένα επίσης μειονέκτημα: Οταν παιρνεις Ω_c = 8057,xx.. , μετά οταν κανεις την αντικατασταση για να φτασεις στο τελικο H(z) θεωρω οτι σου βγαίνει κιαλλο η πιστη με πράξεις. Σε μενα γινεται απαλοιφή.


Από σένα δεν καταλαβαίνω από που συμπεραίνεις ότι με 4000 έχεις -3db, στο περίπου το καταλαβαίνεις ?

***EDIT: Δεν χρειάζεται στις πράξεις αυτό, ουσιαστικά αυτό με το -3db νομίζω είναι σαν 2ο ερώτημα


Title: Re: [ΨΕΣ] Λύσεις παλιών θεμάτων
Post by: 00128419 on June 24, 2019, 21:37:34 pm
Τελικά στα θέματα με διγραμμικό Μ/Σ που έχουμε απροσδιοριστία με την Ω2' πως εργαζόμαστε αφού επίλεξουμε αυθαίρετα μια καινούργια Ω2. Χρησιμοποιύμε την αρχική Ωc που μας δίνει για να βρούμε την τάξη του φίλτρου και να το σχεδιάσουμε ή πρέπει να υπολογίσουμε νεα Ωc απ'τον τύπο που έχει στην 2η σελίδα του buf_dif?


Title: Re: [ΨΕΣ] Λύσεις παλιών θεμάτων
Post by: coraface on September 12, 2019, 21:27:05 pm
Μπορεί κάποιος να εξηγήσει γιατί στο 1ο θεμα σεπτ 17 στις λύσεις απο downloads δεν λαμβανουμε υπόψιν αυτο το z^2 ?


Title: Re: [ΨΕΣ] Λύσεις παλιών θεμάτων
Post by: Singularity on September 12, 2019, 21:59:00 pm
Μπορεί κάποιος να εξηγήσει γιατί στο 1ο θεμα σεπτ 17 στις λύσεις απο downloads δεν λαμβανουμε υπόψιν αυτο το z^2 ?

δεν ξερω κατα πόσο είναι σωστό γιατί θα μπορούσε απλά να διαιρέσει με z μόνο, μετά να έκανε την ανάλυση σε απλά κλάσματα της Η(z)/z και έβρισκε την h[n] χωρίς να έχει εκείνα τα n-1.


Title: Re: [ΨΕΣ] Λύσεις παλιών θεμάτων
Post by: coraface on September 12, 2019, 22:12:51 pm
Οκ ευχαριστω!


Title: Re: [ΨΕΣ] Λύσεις παλιών θεμάτων
Post by: jaime on September 13, 2019, 13:12:33 pm
 Αν έχω 2 πόλους που είναι συζυγείς μιγαδικές ρίζες τότε έχω 3 ROC ή μόνο μία? Μία δεν θα πρεπε να έχω εφόσον το μέτρο τους είναι ίσο?


Title: Re: [ΨΕΣ] Λύσεις παλιών θεμάτων
Post by: Singularity on September 13, 2019, 13:22:00 pm
Αν έχω 2 πόλους που είναι συζυγείς μιγαδικές ρίζες τότε έχω 3 ROC ή μόνο μία? Μία δεν θα πρεπε να έχω εφόσον το μέτρο τους είναι ίσο?

λογικά 1 roc με |z|>μέτρο των μιγαδικών ριζών.


Title: Re: [ΨΕΣ] Λύσεις παλιών θεμάτων
Post by: jaime on September 13, 2019, 14:28:04 pm
 Θένκς, γιατί σε ένα pdf με λυμένα ο συνάδελφος έχει πάρει 3 περιοχές σύγκλισης και δεν μου έβγαζε νόημα.


Title: Re: [ΨΕΣ] Λύσεις παλιών θεμάτων
Post by: coraface on September 14, 2019, 15:04:36 pm
Μας γ****ε τα όνειρα χθες...


Title: Re: [ΨΕΣ] Λύσεις παλιών θεμάτων
Post by: MrRobot on September 14, 2019, 15:21:41 pm
Νομίζω ότι όλα τα θέματα που έβαλε τα είχε ξαναβάλει, δεν ρώτησε κάτι καινούργιο


Title: Re: [ΨΕΣ] Λύσεις παλιών θεμάτων
Post by: coraface on September 14, 2019, 15:33:34 pm
Το πρώτο ναι. Από το δεύτερο το πρώτο ερώτημα ναι, το δεύτερο άμα δεν είχες ασχοληθεί πολύ όπως εγώ δύσκολα το έβγαζες. Το τρίτο θέμα δεν το είχε ξαναβάλει, είχε βάλει ένα παρόμοιο ναι, αλλα εκείνο έβγαινε εύκολα. Αυτό το συγκεκριμένο νομίζω είχε μια παγαποντια την οποία δεν την έβρισκα και η y(n) μου έβγαινε ίση και με μια ακόμα συνάρτηση πέρα της x(n).


Title: Re: [ΨΕΣ] Λύσεις παλιών θεμάτων
Post by: malappapas on January 31, 2020, 17:15:03 pm
Στο Θέμα 1 Σεπτέμβρης 17 δεν θα έπρεπε να είναι 2*(1/2)^n u(n+1) + (1/4)^n u(n+1) αντί του  2*(1/2)^n u(n) + (1/4)^n u(n) μιας και στο ολοκληρωτικό υπόλοιπο το z πάει στο παρονομαστή για n< -1;


Title: Re: [ΨΕΣ] Λύσεις παλιών θεμάτων
Post by: coraface on January 31, 2020, 19:38:49 pm
Έχει λύσει κανείς το 2β και το 3α σεπτεμβρίου του 19?


Title: Re: [ΨΕΣ] Λύσεις παλιών θεμάτων
Post by: arg_13 on January 31, 2020, 22:17:58 pm
Στο Θέμα 1 Σεπτέμβρης 17 δεν θα έπρεπε να είναι 2*(1/2)^n u(n+1) + (1/4)^n u(n+1) αντί του  2*(1/2)^n u(n) + (1/4)^n u(n) μιας και στο ολοκληρωτικό υπόλοιπο το z πάει στο παρονομαστή για n< -1;
Εγώ έτσι το έλυσα , μου βγήκε u(n+1) .
Βγάζω το z απ 'έξω και μετά εφαρμόζω συνέλιξη. Δεν είμαι σίγουρος πόσο σωστό είναι
Αντίστοιχα και το ερώτημα που έχεις είσοδο . Βγάζω το z^2 απο έξω και κάνω μετά συνέλιξη.
Χωρίς να χρησιμοποιώ ολοκληρωτικά υπόλοιπα.


Title: Re: [ΨΕΣ] Λύσεις παλιών θεμάτων
Post by: HarryDresden on February 01, 2020, 16:15:50 pm
Παιδια εχω την εξης απορια για το πρωτο θεμα Φεβρουαριου 19. Αμα παρω το μετασχηματισμο Ζ το y(-1) απο το πρωτο ορο y[n-1]
φευγει γιατι για να βρω το H(z) δε λαμβανω τις αρχηκες συνθηκες ωστοσο για τον ορο y[n+1] επειτα απο πραξεις μου μενει ενα y(0) το οποιο δε μπορω να το διωξω αφου δεν ειναι αρχικη συνθηκη. Παιζει καμια ιδεα για το τι μπορουμε να κανουμε;;;


Title: Re: [ΨΕΣ] Λύσεις παλιών θεμάτων
Post by: kirios_imouna on February 01, 2020, 23:06:29 pm
Παιδια εχω την εξης απορια για το πρωτο θεμα Φεβρουαριου 19. Αμα παρω το μετασχηματισμο Ζ το y(-1) απο το πρωτο ορο y[n-1]
φευγει γιατι για να βρω το H(z) δε λαμβανω τις αρχηκες συνθηκες ωστοσο για τον ορο y[n+1] επειτα απο πραξεις μου μενει ενα y(0) το οποιο δε μπορω να το διωξω αφου δεν ειναι αρχικη συνθηκη. Παιζει καμια ιδεα για το τι μπορουμε να κανουμε;;;

Νομίζω εφόσον δεν υπάρχει αιτιατότητα δεν μπορείς να θεωρήσεις αρχικές συνθήκες 0...αλλά είχα και εγώ πρόβλημα με το συγκεκριμένο θέμα, δεν ξέρω πως λύνεται 


Title: Re: [ΨΕΣ] Λύσεις παλιών θεμάτων
Post by: HarryDresden on February 02, 2020, 00:40:09 am
Νομίζω εφόσον δεν υπάρχει αιτιατότητα δεν μπορείς να θεωρήσεις αρχικές συνθήκες 0...αλλά είχα και εγώ πρόβλημα με το συγκεκριμένο θέμα, δεν ξέρω πως λύνεται 

εγινε μυστη μου αμα μαθεις κατι μη με ξεχασεις


Title: Re: [ΨΕΣ] Λύσεις παλιών θεμάτων
Post by: syrgianm on February 02, 2020, 12:43:49 pm
Παιδια εχω την εξης απορια για το πρωτο θεμα Φεβρουαριου 19. Αμα παρω το μετασχηματισμο Ζ το y(-1) απο το πρωτο ορο y[n-1]
φευγει γιατι για να βρω το H(z) δε λαμβανω τις αρχηκες συνθηκες ωστοσο για τον ορο y[n+1] επειτα απο πραξεις μου μενει ενα y(0) το οποιο δε μπορω να το διωξω αφου δεν ειναι αρχικη συνθηκη. Παιζει καμια ιδεα για το τι μπορουμε να κανουμε;;;
Δεν λεει για κατι για αιτιοτητα,ειναι διπλευρος ο μετασχηματισμος οποτε εαν το πας με τον ορισμο το n τρεχει απο -οο εως +οο οποτε θα διαπιστωσεις με μια αλλαγη μεταβλητης στο αθροισμα πως δεν προκυπτουν αρχικες συνθηκες εκτος απο την ιδιοτητα της μετατοπισης  8)) 8))


Title: Re: [ΨΕΣ] Λύσεις παλιών θεμάτων
Post by: sterpapi on February 02, 2020, 22:00:16 pm
Έχει κανείς λύσεις Σεπτεμβρίου 2019 να ανεβάσει;


Title: Re: [ΨΕΣ] Λύσεις παλιών θεμάτων
Post by: asteridp on February 03, 2020, 03:46:23 am
To h(n) της y(n)=h(n)*x(n) αν το κανω μτσχηματισμο Ζ βρισκω Η(z)?
Αν ξερω Η(z) πως βρισκω h(n) ?


Title: Re: [ΨΕΣ] Λύσεις παλιών θεμάτων
Post by: Van Gogh on July 12, 2020, 11:28:54 am
Έχει κανείς ιδέα για λύση του θέματος 2, Φεβρουάριος 2020;


Title: Re: [ΨΕΣ] Λύσεις παλιών θεμάτων
Post by: ilektrik on July 13, 2020, 16:51:01 pm
Έχει κανείς ιδέα για λύση του θέματος 2, Φεβρουάριος 2020;

Βρήκα αυτές τις λύσεις για τον Φλεβάρη του '20 (εκτός από το ερώτημα γ του θέματος 3). Δεν εγγυώμαι ότι είναι σωστές, αλλά ελπίζω να βοηθήσουν!  8))



Title: Re: [ΨΕΣ] Λύσεις παλιών θεμάτων
Post by: Van Gogh on July 13, 2020, 18:55:22 pm
Βρήκα αυτές τις λύσεις για τον Φλεβάρη του '20 (εκτός από το ερώτημα γ του θέματος 3). Δεν εγγυώμαι ότι είναι σωστές, αλλά ελπίζω να βοηθήσουν!  8))



Thank you!!!  ;D


Title: Re: [ΨΕΣ] Λύσεις παλιών θεμάτων
Post by: Μαύρη Μάμπα on July 13, 2020, 19:02:02 pm
Βρήκα αυτές τις λύσεις για τον Φλεβάρη του '20 (εκτός από το ερώτημα γ του θέματος 3). Δεν εγγυώμαι ότι είναι σωστές, αλλά ελπίζω να βοηθήσουν!  8))



 ^super^


Title: Re: [ΨΕΣ] Λύσεις παλιών θεμάτων
Post by: koumanas on July 13, 2020, 19:39:11 pm
Αφου ζητάει την τιμή στην μόνιμη κατάσταση δεν θέλει ουσιαστικά το lim(y(n)) για n που τείνει στο άπειρο?Και μετα αυτό είναι ίσο με lim((1-z^-1)*Y(z)) για z που τείνει στο 1(απο σελ 22 sto pdf zt των σημειώσεων).

+1

Δεν εχεις αδικο. Το 8050 ειναι εντος του ευρους τιμων που βγαζω και εγω. Ομως οταν βαζεις Ω_2', αυτο σημαινει οτι το αποτελεσμα εξαρταται αναλογα με το Ω_2 σου (δηλαδη αν επελεξες 999, 990, 950, 900 ... κλπ για να βγει το tan στην αρχη). Και επιπλεον οταν χρησιμοποιείς το Ω_2', αυτό σημαίνει οτι η αποσβεση ειναι ΑΚΡΙΒΩΣ -20dB για Ω = Ω_2'. Αν επιλέξεις μικρότερο Ω_c από αυτό που βγάζεις (π.χ. 7000) , τότε για Ω = Ω_2' θα έχεις ακομα μεγαλυτερη αποσβεση, πχ -22dB.

Εν κατακλέιδα: Αυτό π κανεις ειναι σωστό. Απλά θεωρώ κάνεις τζαμπα πολλούς υπολογισμούς ενώ υπάρχει έτοιμος τύπος (σελ 2 του but_dif.pdf)  που νομίζω είναι εξίσου σωστός.

Επιπλέον ένα επίσης μειονέκτημα: Οταν παιρνεις Ω_c = 8057,xx.. , μετά οταν κανεις την αντικατασταση για να φτασεις στο τελικο H(z) θεωρω οτι σου βγαίνει κιαλλο η πιστη με πράξεις. Σε μενα γινεται απαλοιφή.

+1


Title: Re: [ΨΕΣ] Λύσεις παλιών θεμάτων
Post by: ilektrik on July 13, 2020, 23:20:23 pm
Βρήκα αυτές τις λύσεις για τον Φλεβάρη του '20 (εκτός από το ερώτημα γ του θέματος 3). Δεν εγγυώμαι ότι είναι σωστές, αλλά ελπίζω να βοηθήσουν!  8))

Στο θέμα 4 υπάρχει ένα μικρό λάθος. Ο μετασχηματισμός Z του w[n+1] είναι W[z]*z (και όχι z^(-1))!

Ανεβάζω και λύσεις των 2, 3, 4 από Σεπτέμβρη '16 και 3 από Φλεβάρη '16. Κάποια ίσως έχουν ήδη λυθεί παλαιότερα σε αυτό το thread.


Title: Re: [ΨΕΣ] Λύσεις παλιών θεμάτων
Post by: jim.jt on February 04, 2021, 23:00:23 pm
Έχει κανείς καμία ιδέα για το Θέμα 3γ του Φεβρουαρίου 2020;


Title: Re: [ΨΕΣ] Λύσεις παλιών θεμάτων
Post by: mermaid on February 10, 2021, 15:08:01 pm
Έχει κανείς καμία ιδέα για το Θέμα 3γ του Φεβρουαρίου 2020;

Στις λύσεις 2020 που έχουν ανέβει πιο πάνω, αν και είναι μια χαρά λυμένο το 3β), δεν επιτρέπει να παρατηρήσουμε κάτι σχετικά με το γ). Ένας άλλος τρόπος για το β) είναι να πούμε ότι x[n] = (-1)n = (e)n, δηλαδή το σήμα εισόδου είναι εκθετικό και άρα παραμένει αναλλοίωτο (η έξοδος θα είναι η είσοδος πολλαπλασιασμένη με ένα βάρος). Το βάρος αυτό είναι το μέτρο της απόκρισης συχνότητας για ω = π που βγαίνει 0 και άρα καταλήγει στο ίδιο αποτέλεσμα. (Αυτή η λογική είχε γίνει σε άσκηση του Χατζηλεοντιάδη σελ. 74-76 από σημειώσεις sugar99)

Πλέον ξέρουμε σε ποια συχνότητα αναφέρεται το β), η οποία είναι και η μέγιστη αφού η πληροφορία βρίσκεται σε μια περίοδο (Τ = 2π) και μετά υπάρχει συμμετρία, έστω από -π μέχρι π. Άρα το β) λέει ότι στη συχνότητα π δεν περνάει κάτι, δηλαδή το σύστημα δεν μπορεί να χρησιμοποιηθεί σαν υψιπερατό.

Αν δεν κάνω λάθος είναι στην ουσία LowPass και αυτό φαίνεται και από την ΕΔ και από το διάγραμμα της H που προκύπτει στο α) ως εξής.

Για την ΕΔ

y[n] = x[n] + 2x[n-1] + 2x[n-2] + x[n-3]
       = x[n] + x[n-1] + x[n-1] + x[n-2] + x[n-2] + x[n-3]
       = x[n-1] + x[n] + x[n-2] + x[n-1] + x[n-3] + x[n-2]

Τα χρωματισμένα είναι τοπικά ολοκληρώματα, δηλαδή Low Pass άρα το σύστημα είναι Low Pass.

Για το διάγραμμα πόλων-μηδενικών και μέτρο της απόκρισης συχνότητας |H(e)|

Από το διάγραμμα πόλων-μηδενικών που αναφέρεται στο α) μπορούμε προσεγγιστικά να φτιάξουμε το διάγραμμα του |H(e)| με βάση την απόσταση των μηδενικών και τον πόλων από διαφορετικά σημεία του μοναδιαίου κύκλου (ei) (σελ. 78-79 από σημειώσεις sugar99). Αρκεί να παρατηρήσουμε το διάγραμμα από 0 έως π και μετά θα είναι συμμετρικό. Άρα ξεκινώντας από το ω = 0 στο pole-zero plot και προχωρώντας προς π, έχουμε την απόσταση των πόλων να παραμένει σταθερή και ίση με τη μονάδα ενώ την απόσταση των μηδενικών να μειώνεται ώσπου πέφτουμε πάνω στο μηδενικό -1 και μηδενίζεται η απόσταση τους, άρα και ο αριθμητής του μέτρου της απόκρισης συχνότητας άρα και το ίδιο το μέτρο, για αυτό και το μηδενικό αποτέλεσμα στο β) (λίγο πιο ποιοτικά).
Δηλαδή τελικά το διάγραμμα της H θα ξεκινάει από μια τιμή και σιγά σιγά θα μειώνεται ώσπου θα μηδενιστεί για ω = π -> μορφή LowPass.

Αυτά μετά από μια συζήτηση, σόρρυ για το κατεβατό αλλά είπα να τα έχουμε όλα μαζί. :P Αν κάπου το έχω χάσει πείτε μου να το δούμε.


Title: Re: [ΨΕΣ] Λύσεις παλιών θεμάτων
Post by: Numb3rs on February 12, 2021, 19:57:10 pm
Ανεβαζω τις δικες μου απαντησεις απο Ιουνιο 2020 (πολλαπλης)
  • 1 a
  • 2 a
  • 3 b
  • 4 b
  • 5 Λάθος
  • 6 b
  • 7 Σωστό
  • 8 a
  • 9 b
  • 10 Σωστό
  • 11 c (ισως και a αν το ROC περιέχει το 1 και εννοείται)
  • 12 c
  • 13 b
  • 14 Σωστό
  • 15 c
  • 16 a
  • 17 d
  • 18 Λάθος
  • 19 c
  • 20 a
  • 21 c
  • 22 Λάθος
  • 23 a
  • 24 d
  • 25 d
  • 26 a
  • 27 b
  • 28 d
  • 29 b
  • 30 c
Αν κάποιος τα έλυσε ας μου πει τα συγκρίνουμε λύσεις γιατί τα είδα λίγο βιαστικά

Edit: διορθώθηκαν τα λάθη


Title: Re: [ΨΕΣ] Λύσεις παλιών θεμάτων
Post by: achariso on February 13, 2021, 01:05:33 am
Στο 10 εγώ θα έβαζα Σωστό, γτ ο DFT έχει πολυπλοκότητα O(n^2) και άρα για να υπολογίσει τα πρώτα log2(N) θα χρειαστεί log2(N) * log2(N) ενώ ο FFT επειδή το λύνει αναδρομικά θα πρέπει να υπολογίσει όλες τις τιμές και να πάρει τις πρώτες log2(N), το οποίο σημαίνει log2(N)*N. Και επειδή log2(N) < N θα έβαζα Σωστό.

Με επιφύλαξη πάντα :)

Edit: Επίσης στο 13 θα έβαζα το b.
Edit 2: Στο 25 το έβγαλα να θέλει ±✓(1-α) και όχι ±(1-α). Θες να μου πεις πως το βρήκες;

sent from mTHMMY (https://play.google.com/store/apps/details?id=gr.thmmy.mthmmy)  


Title: Re: [ΨΕΣ] Λύσεις παλιών θεμάτων
Post by: Numb3rs on February 13, 2021, 02:49:18 am
Στο 10 εγώ θα έβαζα Σωστό, γτ ο DFT έχει πολυπλοκότητα O(n^2) και άρα για να υπολογίσει τα πρώτα log2(N) θα χρειαστεί log2(N) * log2(N) ενώ ο FFT επειδή το λύνει αναδρομικά θα πρέπει να υπολογίσει όλες τις τιμές και να πάρει τις πρώτες log2(N), το οποίο σημαίνει log2(N)*N. Και επειδή log2(N) < N θα έβαζα Σωστό.

Με επιφύλαξη πάντα :)

Edit: Επίσης στο 13 θα έβαζα το b.
Edit 2: Στο 25 το έβγαλα να θέλει ±✓(1-α) και όχι ±(1-α). Θες να μου πεις πως το βρήκες;

sent from mTHMMY (https://play.google.com/store/apps/details?id=gr.thmmy.mthmmy)  

Το 10 μαλλον έχεις δίκιο γιατί ο fft θελει δύναμη του 2 για να δουλεψει τελεια, ισως το χανω και απο αλλου  :P

Στο 13 καταρχάς δεν πρεπει να έχει συντελεστή μπροστά αν δεν κάνω λάθος, και ναι το β είναι πιο κοντα https://en.wikipedia.org/wiki/Discrete_Fourier_series

Στο 25 βρήκα το μετρο του H(e^jw) που ειναι b/[1-a*e^jw] = ... =b/sqrt(1+a^2-2*a*cosw) που παίνρει μέγιστη τιμή για cosw=1 αρα b/(sqrt(a^2-2a+1)) = b/(sqrt(a-1)^2)) = b/(a-1) με +- λόγω ρίζας και τέλος εξισώνω αριθμητή με παρονομαστή γιατί θέλω μέτρο μονάδα

Edit: Στο βιβλιο του Shaum εχει παρομοια ασκηση 2.16


Title: Re: [ΨΕΣ] Λύσεις παλιών θεμάτων
Post by: No_one on February 13, 2021, 14:45:21 pm
το 18 μπορει να μου το εξηγησει καποιος?


Title: Re: [ΨΕΣ] Λύσεις παλιών θεμάτων
Post by: Numb3rs on February 13, 2021, 14:55:04 pm
το 18 μπορει να μου το εξηγησει καποιος?
H διατυπωση ειναι περιεργη αλλα ο DFT οριζεται για πεπερασμενη ακουλουθια μηκους N απο 0,1,..,Ν-1


Title: Re: [ΨΕΣ] Λύσεις παλιών θεμάτων
Post by: No_one on February 13, 2021, 16:08:45 pm
Αυτο φανταστηκα και εγω


Title: Re: [ΨΕΣ] Λύσεις παλιών θεμάτων
Post by: Patatompataria on February 13, 2021, 16:54:53 pm
Στο 13 καταρχάς δεν πρεπει να έχει συντελεστή μπροστά αν δεν κάνω λάθος, και ναι το β είναι πιο κοντα https://en.wikipedia.org/wiki/Discrete_Fourier_series

Για το 13 κι εμένα καμία επιλογή δεν μου φαίνεται σωστή τώρα. Θα έπρεπε ή να μην έχει συντελεστή μπροστά, ή αν εννοεί για το αντίστροφο dfs αντί για x[n] να είχε τα c_n


Title: Re: [ΨΕΣ] Λύσεις παλιών θεμάτων
Post by: πανωλεθρία on February 13, 2021, 17:12:39 pm
Για το 13 κι εμένα καμία επιλογή δεν μου φαίνεται σωστή τώρα. Θα έπρεπε ή να μην έχει συντελεστή μπροστά, ή αν εννοεί για το αντίστροφο dfs αντί για x[n] να είχε τα c_n

https://elearning.auth.gr/mod/resource/view.php?id=267092 (https://elearning.auth.gr/mod/resource/view.php?id=267092)

πρωτη σειρα στο DTFS νομιζω ειναι η φαση  8))


Title: Re: [ΨΕΣ] Λύσεις παλιών θεμάτων
Post by: Patatompataria on February 13, 2021, 18:40:36 pm
όντως με βάση αυτό το φυλλάδιο έτσι είναι..

Μου φαίνεται ότι υπάρχουν 2 τρόποι ορισμού, επειδή και στο μάθημα και στη Wikipedia και στο βιβλίο του Schaum έχει το 1/Ν στην αντίστροφη DFS. Σε κάποια άλλα μέρη όμως (που μπορεί να αναφέρεται ως dtfs) έχει το 1/Ν στον ευθύ.
τελοσπαντων...


Title: Re: [ΨΕΣ] Λύσεις παλιών θεμάτων
Post by: snek on February 13, 2021, 21:13:04 pm
ερωτηση 3 γτ το δ ?


Title: Re: [ΨΕΣ] Λύσεις παλιών θεμάτων
Post by: msideridou on February 13, 2021, 21:28:14 pm
για την ερωτηση 26 που καμια απο τις απαντησεις δεν είναι σωστες κάνουμε χρήση τους τύπους για τον υπολογισμο της εξόδου βάση του ότι η είσοδος είναι αθροισμα  αιώνιων cos και sin;Θα μπορουσε κάποιος να ανεβάσει την λύση;



Title: Re: [ΨΕΣ] Λύσεις παλιών θεμάτων
Post by: Patatompataria on February 13, 2021, 22:14:41 pm
ερωτηση 3 γτ το δ ?
Νομίζω ολοδιαβατο εγώ, αφού το |Η(ω)| είναι 1 για κάθε ω


για την ερωτηση 26 που καμια απο τις απαντησεις δεν είναι σωστες κάνουμε χρήση τους τύπους για τον υπολογισμο της εξόδου βάση του ότι η είσοδος είναι αθροισμα  αιώνιων cos και sin;Θα μπορουσε κάποιος να ανεβάσει την λύση;

Νομίζω η 26 είναι γεμάτη τυπογραφικά λάθη. Για παράδειγμα, το /2 μπορεί να εννοούσε ότι είναι μέσα στο sin, στις απαντήσεις το n δε γίνεται να είναι στον παρονομαστή, και τα 26,6 μάλλον είναι μοίρες όχι ακτίνια. Έτσι έβγαλα το γ.
Το σκεπτικό είναι ότι για κάθε ημίτονο πολλαπλασιάζεις με ένα μέτρο και προσθέτεις μια φάση, κάθε φορά αυτά που δίνουν το Η(ω) για το ω του κάθε ημιτόνου


Title: Re: [ΨΕΣ] Λύσεις παλιών θεμάτων
Post by: πανωλεθρία on February 13, 2021, 22:22:59 pm
ερωτηση 3 γτ το δ ?

Νομίζω ολοδιαβατο εγώ, αφού το |Η(ω)| είναι 1 για κάθε ω

Κι εγω βγαζω οτι το φιλτρο ειναι all-pass (δηλαδη οτι το σωστο ειναι το b) με βαση σελ. 78-79 που εχουν ανεβει κι εδω στα downloads.

Ουσιαστικα η συναρτηση H(z) που μας δινεται ειναι 1/z^k, άρα έχει πολο/πολους στο 0. Αν κανεις το zero-pole plot και βαλεις τον/τους πολους σου στο μηδεν, θα δεις πως εχεις μονιμα αποσταση 1 απο τον μοναδιαιο κυκλο, από το 0 εως το π (μετα εχουμε συμμετρικοτητα). Αρα το μετρο της συναρτησης ειναι σταθερο και ετσι αποδεχεται ολες τις συχνοτητες (δεν εχει καμπυλη, οπως το BP φιλτρο των σημειωσεων).


Title: Re: [ΨΕΣ] Λύσεις παλιών θεμάτων
Post by: kanou_tom on February 13, 2021, 22:44:03 pm
Κάποια εξήγηση για την ερώτηση 14;


Title: Re: [ΨΕΣ] Λύσεις παλιών θεμάτων
Post by: snek on February 13, 2021, 22:46:19 pm
Κι εγω βγαζω οτι το φιλτρο ειναι all-pass (δηλαδη οτι το σωστο ειναι το b) με βαση σελ. 78-79 που εχουν ανεβει κι εδω στα downloads.

Ουσιαστικα η συναρτηση H(z) που μας δινεται ειναι 1/z^k, άρα έχει πολο/πολους στο 0. Αν κανεις το zero-pole plot και βαλεις τον/τους πολους σου στο μηδεν, θα δεις πως εχεις μονιμα αποσταση 1 απο τον μοναδιαιο κυκλο, από το 0 εως το π (μετα εχουμε συμμετρικοτητα). Αρα το μετρο της συναρτησης ειναι σταθερο και ετσι αποδεχεται ολες τις συχνοτητες (δεν εχει καμπυλη, οπως το BP φιλτρο των σημειωσεων).

Συμφώνω, μήπως ξέρεις ερώτηση 6 αν υπάρχει μεθοδολογία απο τις σημειώσεις για την λύση του ?


Title: Re: [ΨΕΣ] Λύσεις παλιών θεμάτων
Post by: jim.jt on February 13, 2021, 23:01:09 pm
Συμφώνω, μήπως ξέρεις ερώτηση 6 αν υπάρχει μεθοδολογία απο τις σημειώσεις για την λύση του ?

Δες στις σημειώσεις στα Downloads σελίδα 79 αυτά που δείχνει με τους πόλους και τα μηδενικά. Ουσιαστικά, άμα έχεις ένα χαμηλοπερατό (που εδώ έχει έναν πόλο στο αριστερό ημιεπίπεδο) και πάρεις τον πόλο στο αντισυμμετρικό του, τότε θα έχεις την αντίστροφη απόκριση συχνότητας, άρα θα είναι υψιπερατό.


Title: Re: [ΨΕΣ] Λύσεις παλιών θεμάτων
Post by: jim.jt on February 13, 2021, 23:09:29 pm
Κάποια εξήγηση για την ερώτηση 14;

Στον Μ/Σ Fourier βάλε όπου ω το -ω. Μετά βάλε όπου x[n], το x[-n] (αφού η x είναι άρτια αυτά τα δύο είναι ίσα) και άλλαξε μεταβλητή στο άθροισμα σε u=-n. Τότε βγαίνει η ίδια ακριβώς σειρά με το X(ω), άρα Χ(ω)=Χ(-ω), οπότε είναι σωστό.


Title: Re: [ΨΕΣ] Λύσεις παλιών θεμάτων
Post by: snek on February 13, 2021, 23:22:37 pm
ερώτηση 9 έβγαλα το β


Title: Re: [ΨΕΣ] Λύσεις παλιών θεμάτων
Post by: kanou_tom on February 13, 2021, 23:36:55 pm
Επίσης το 18 εμένα σωστό μου φαίνεται. Συμφωνεί κάποιος άλλος; Ουσιαστικά λέει πως η ακολουθία x είναι περιοδική με περίοδο Ν κάτι το οποίο από τις σημειώσεις προκύπτει για τον  DFS.


Title: Re: [ΨΕΣ] Λύσεις παλιών θεμάτων
Post by: Numb3rs on February 14, 2021, 00:02:55 am
ερώτηση 9 έβγαλα το β
πως το εβγαλες; Εγω πηρα το αθροισμα, το εσπασα απο 0 εως απειρο και -απειρο έως 1, προσθαφαιρασα την μοναδα για να τα φερω στην μορφη απο 0 εως απειρο του b^n που ειναι γνωστο κλπ αλλα δεν εβγαλα κατι παρομοιο


Title: Re: [ΨΕΣ] Λύσεις παλιών θεμάτων
Post by: snek on February 14, 2021, 00:08:21 am
Στο 10 εγώ θα έβαζα Σωστό, γτ ο DFT έχει πολυπλοκότητα O(n^2) και άρα για να υπολογίσει τα πρώτα log2(N) θα χρειαστεί log2(N) * log2(N) ενώ ο FFT επειδή το λύνει αναδρομικά θα πρέπει να υπολογίσει όλες τις τιμές και να πάρει τις πρώτες log2(N), το οποίο σημαίνει log2(N)*N. Και επειδή log2(N) < N θα έβαζα Σωστό.

Με επιφύλαξη πάντα :)

Edit: Επίσης στο 13 θα έβαζα το b.
Edit 2: Στο 25 το έβγαλα να θέλει ±✓(1-α) και όχι ±(1-α). Θες να μου πεις πως το βρήκες;

sent from mTHMMY (https://play.google.com/store/apps/details?id=gr.thmmy.mthmmy)  
Γιατι να τις υπολογισει ολες και να παρει τις πρώτες log2(N)? Αφού λεει ότι απλά ο επιθυμητος αριθμός τιμών είναι log2(N) , οχι N και θα πάρουμε τις log2(N)


Title: Re: [ΨΕΣ] Λύσεις παλιών θεμάτων
Post by: Numb3rs on February 14, 2021, 00:09:26 am
Επίσης το 18 εμένα σωστό μου φαίνεται. Συμφωνεί κάποιος άλλος; Ουσιαστικά λέει πως η ακολουθία x είναι περιοδική με περίοδο Ν κάτι το οποίο από τις σημειώσεις προκύπτει για τον  DFS.
DFT λεει ομως, οχι DFS


Title: Re: [ΨΕΣ] Λύσεις παλιών θεμάτων
Post by: snek on February 14, 2021, 00:16:50 am
πως το εβγαλες; Εγω πηρα το αθροισμα, το εσπασα απο 0 εως απειρο και -απειρο έως 1, προσθαφαιρασα την μοναδα για να τα φερω στην μορφη απο 0 εως απειρο του b^n που ειναι γνωστο κλπ αλλα δεν εβγαλα κατι παρομοιο
Στην εικονα φαινονται οι πραξεις που εκανα, ενα βημα ακομα και βγαινει το αποτελεσμα


Title: Re: [ΨΕΣ] Λύσεις παλιών θεμάτων
Post by: achariso on February 14, 2021, 00:25:56 am
Γιατι να τις υπολογισει ολες και να παρει τις πρώτες log2(N)? Αφού λεει ότι απλά ο επιθυμητος αριθμός τιμών είναι log2(N) , οχι N και θα πάρουμε τις log2(N)

νομίζω γιατί ο fft δεν μπορείς να τον κόψεις εκεί που εκτελείται και να πάρεις πχ τις πρώτες 10 τιμές γιατί αυτές θέλεις... ενώ αν δεν κάνω λάθος στον DFT οι τιμές υπολογίζονται σειριακά..
sent from mTHMMY (https://play.google.com/store/apps/details?id=gr.thmmy.mthmmy) 


Title: Re: [ΨΕΣ] Λύσεις παλιών θεμάτων
Post by: achariso on February 14, 2021, 00:29:04 am
Κι εγω βγαζω οτι το φιλτρο ειναι all-pass (δηλαδη οτι το σωστο ειναι το b) με βαση σελ. 78-79 που εχουν ανεβει κι εδω στα downloads.

Ουσιαστικα η συναρτηση H(z) που μας δινεται ειναι 1/z^k, άρα έχει πολο/πολους στο 0. Αν κανεις το zero-pole plot και βαλεις τον/τους πολους σου στο μηδεν, θα δεις πως εχεις μονιμα αποσταση 1 απο τον μοναδιαιο κυκλο, από το 0 εως το π (μετα εχουμε συμμετρικοτητα). Αρα το μετρο της συναρτησης ειναι σταθερο και ετσι αποδεχεται ολες τις συχνοτητες (δεν εχει καμπυλη, οπως το BP φιλτρο των σημειωσεων).


Και εγώ συμφωνώ καθώς στην ουσία φαίνεται να είναι ένας απλός καθυστερητής.. Αλλά νομίζω πως έπρεπε να βάλουμε το c) δλδ. κανένα από τα υπόλοιπα, καθώς δεν ξέρουμε τη τιμή του k; Συμφωνείτε;;
sent from mTHMMY (https://play.google.com/store/apps/details?id=gr.thmmy.mthmmy)  


Title: Re: [ΨΕΣ] Λύσεις παλιών θεμάτων
Post by: snek on February 14, 2021, 00:36:11 am
νομίζω γιατί ο fft δεν μπορείς να τον κόψεις εκεί που εκτελείται και να πάρεις πχ τις πρώτες 10 τιμές γιατί αυτές θέλεις... ενώ αν δεν κάνω λάθος στον DFT οι τιμές υπολογίζονται σειριακά..
sent from mTHMMY (https://play.google.com/store/apps/details?id=gr.thmmy.mthmmy) 
Αμα το θεωρησεις ετσι ναι, αλλα εγω δεν το ελαβα οτι εχουμε N τιμες αρχικά και θελουμε τις log2(N) , το έλαβα ως οτι έχουμε  log2(N) τιμες και τελος.


Title: Re: [ΨΕΣ] Λύσεις παλιών θεμάτων
Post by: snek on February 14, 2021, 00:37:12 am
Ερώτηση 11 βαζουμε το κανενα απο τα υπολοιπα , γιατι μπορουμε να βρουμε καπως την ROC ? (μπορει να ειναι |z|>0.8 και |z|<0.8)


Title: Re: [ΨΕΣ] Λύσεις παλιών θεμάτων
Post by: achariso on February 14, 2021, 00:46:50 am
Αμα το θεωρησεις ετσι ναι, αλλα εγω δεν το ελαβα οτι εχουμε N τιμες αρχικά και θελουμε τις log2(N) , το έλαβα ως οτι έχουμε  log2(N) τιμες και τελος.

ναι και γω το σκέφτηκα αυτό αλλά αν ήταν έτσι το "Ν" που αναφέρεται;

στο 11 εγώ για αυτό έβαλα λαθος
sent from mTHMMY (https://play.google.com/store/apps/details?id=gr.thmmy.mthmmy) 


Title: Re: [ΨΕΣ] Λύσεις παλιών θεμάτων
Post by: snek on February 14, 2021, 03:55:46 am
DFT λεει ομως, οχι DFS
Τσεκαρε πανα σελιδα 36


Title: Re: [ΨΕΣ] Λύσεις παλιών θεμάτων
Post by: snek on February 14, 2021, 04:38:15 am
Αναλυτική λύση για ερωτηση 19 ?


Title: Re: [ΨΕΣ] Λύσεις παλιών θεμάτων
Post by: danaerose on February 14, 2021, 09:30:26 am
Αναλυτική λύση για ερωτηση 19 ?

Αν δεν κάνω λάθος αυτό ζητάει


Title: Re: [ΨΕΣ] Λύσεις παλιών θεμάτων
Post by: mermaid on February 14, 2021, 11:10:11 am
Στον Μ/Σ Fourier βάλε όπου ω το -ω. Μετά βάλε όπου x[n], το x[-n] (αφού η x είναι άρτια αυτά τα δύο είναι ίσα) και άλλαξε μεταβλητή στο άθροισμα σε u=-n. Τότε βγαίνει η ίδια ακριβώς σειρά με το X(ω), άρα Χ(ω)=Χ(-ω), οπότε είναι σωστό.

Αλλάζοντας μεταβλητή δεν θα αλλάξουν και τα όρια του αθροίσματος όμως; Ναι μεν θα έχεις την ίδια έκφραση στο εσωτερικό του αλλά τα όρια θα είναι από +άπειρο έως -άπειρο. Μπορείς να εξηγήσεις πως το αντιμετώπισες για να τα βγάλεις ίσα;


Title: Re: [ΨΕΣ] Λύσεις παλιών θεμάτων
Post by: danaerose on February 14, 2021, 11:45:02 am
Αλλάζοντας μεταβλητή δεν θα αλλάξουν και τα όρια του αθροίσματος όμως; Ναι μεν θα έχεις την ίδια έκφραση στο εσωτερικό του αλλά τα όρια θα είναι από +άπειρο έως -άπειρο. Μπορείς να εξηγήσεις πως το αντιμετώπισες για να τα βγάλεις ίσα;
Υπάρχει ένα πινακάκι στη σελίδα 97 σειράς shaum που σου λέει
Αν x(n) πραγματική και άρτια τότε Χ(ω) πραγματική και άρτια,αν x(n) φανταστική και άρτια τότε Χ(ω) φανταστική και άρτια.
Απο αυτό βγαίνει το συμπέρασμα οτι θα ισχύει αυτό για την ερώτηση 14 :)


Title: Re: [ΨΕΣ] Λύσεις παλιών θεμάτων
Post by: mermaid on February 14, 2021, 12:06:13 pm
Υπάρχει ένα πινακάκι στη σελίδα 97 σειράς shaum που σου λέει
Αν x(n) πραγματική και άρτια τότε Χ(ω) πραγματική και άρτια,αν x(n) φανταστική και άρτια τότε Χ(ω) φανταστική και άρτια.
Απο αυτό βγαίνει το συμπέρασμα οτι θα ισχύει αυτό για την ερώτηση 14 :)

θενξ! :) συμφωνώ με το αποτέλεσμα απλά δεν έπιασα την απόδειξη


Title: Re: [ΨΕΣ] Λύσεις παλιών θεμάτων
Post by: jim.jt on February 14, 2021, 12:22:15 pm
Αλλάζοντας μεταβλητή δεν θα αλλάξουν και τα όρια του αθροίσματος όμως; Ναι μεν θα έχεις την ίδια έκφραση στο εσωτερικό του αλλά τα όρια θα είναι από +άπειρο έως -άπειρο. Μπορείς να εξηγήσεις πως το αντιμετώπισες για να τα βγάλεις ίσα;

Οι όροι του αθροίσματος είναι ίδιοι. Δεν σε νοιάζει η διάταξη των ορίων όπως στο ολοκλήρωμα. Ουσιαστικά εδώ τα δύο αθροίσματα είναι ίσα επειδή είναι συμμετρικά ως προς το 0. Σκέψου να έχω το Σn από - 1 έως 1, τότε είναι - 1+0+1. Άμα άλλαζα μεταβλητή σε -n τότε οι αντίστοιχοι προσθεταίοι του αθροίσματος είναι οι 1,0 και - 1 άρα οι ίδιοι. Το ίδιο συμβαίνει και στο - άπειρο έως άπειρο. Μόνο άμα ήταν πχ από - 3 έως 1 θα είχαμε πρόβλημα.


Title: Re: [ΨΕΣ] Λύσεις παλιών θεμάτων
Post by: Thunderlord on February 14, 2021, 12:26:28 pm
Αλλάζοντας μεταβλητή δεν θα αλλάξουν και τα όρια του αθροίσματος όμως; Ναι μεν θα έχεις την ίδια έκφραση στο εσωτερικό του αλλά τα όρια θα είναι από +άπειρο έως -άπειρο. Μπορείς να εξηγήσεις πως το αντιμετώπισες για να τα βγάλεις ίσα;

Γενικά το άθροισμα δεν είναι σαν το ολοκλήρωμα. Δηλαδή μπορείς να αλλάξεις τα όρια και να τα βάλεις ανάποδα, χωρίς να χρειάζεται να προσθέσεις ένα - μπορστά, όπως θα γινόταν στο ολοκλήρωμα. Γιατί, τι διαφορά έχει αν προσθέσεις πρώτα τους όρους για θετικά n και μετά τους όρους για αρνητικά n ή το ανάποδο; Πάντα πρόσθεση όλων των όρων δεν κάνεις;

edit: Τώρα είδα ότι με πρόλαβε ο jim jt


Title: Re: [ΨΕΣ] Λύσεις παλιών θεμάτων
Post by: mermaid on February 14, 2021, 12:27:29 pm
Οι όροι του αθροίσματος είναι ίδιοι. Δεν σε νοιάζει η διάταξη των ορίων όπως στο ολοκλήρωμα. Ουσιαστικά εδώ τα δύο αθροίσματα είναι ίσα επειδή είναι συμμετρικά ως προς το 0. Σκέψου να έχω το Σn από - 1 έως 1, τότε είναι - 1+0+1. Άμα άλλαζα μεταβλητή σε -n τότε οι αντίστοιχοι προσθεταίοι του αθροίσματος είναι οι 1,0 και - 1 άρα οι ίδιοι. Το ίδιο συμβαίνει και στο - άπειρο έως άπειρο. Μόνο άμα ήταν πχ από - 3 έως 1 θα είχαμε πρόβλημα.

Γενικά το άθροισμα δεν είναι σαν το ολοκλήρωμα. Δηλαδή μπορείς να αλλάξεις τα όρια και να τα βάλεις ανάποδα, χωρίς να χρειάζεται να προσθέσεις ένα - μπορστά, όπως θα γινόταν στο ολοκλήρωμα. Γιατί, τι διαφορά έχει αν προσθέσεις πρώτα τους όρους για θετικά n και μετά τους όρους για αρνητικά n ή το ανάποδο; Πάντα πρόσθεση όλων των όρων δεν κάνεις;

έχετε δίκιο, ευχαριστώ παιδιά με ξεμπλοκάρατε ;D


Title: Re: [ΨΕΣ] Λύσεις παλιών θεμάτων
Post by: Thunderlord on February 14, 2021, 12:54:21 pm
Ερώτηση 24 τι βάλατε;


Title: Re: [ΨΕΣ] Λύσεις παλιών θεμάτων
Post by: πανωλεθρία on February 14, 2021, 13:44:42 pm
Ερώτηση 24 τι βάλατε;

d
μονο οταν εισαι πανω στον μοναδιαιο κυκλο μπορεις να πας απο τον ενα μετασχηματισμο στον αλλον


Title: Re: [ΨΕΣ] Λύσεις παλιών θεμάτων
Post by: πανωλεθρία on February 14, 2021, 13:47:06 pm
Και εγώ συμφωνώ καθώς στην ουσία φαίνεται να είναι ένας απλός καθυστερητής.. Αλλά νομίζω πως έπρεπε να βάλουμε το c) δλδ. κανένα από τα υπόλοιπα, καθώς δεν ξέρουμε τη τιμή του k; Συμφωνείτε;;
sent from mTHMMY (https://play.google.com/store/apps/details?id=gr.thmmy.mthmmy)  

Το k θα επηρεασει μονο την πολλαπλοτητα του πολου σου, οπως το σκεφτομαι εγω. Αλλα δεν επηρεαζει την αποσταση που θα εχει αυτος ο πολος z=0 (οποιας πολλαπλοτητας κι αν ειναι) από τον μονδιαιο κυκλο
Εσυ με ποια λογικη θεωρεις οτι επηρεαζει την απαντηση;


Title: Re: [ΨΕΣ] Λύσεις παλιών θεμάτων
Post by: Numb3rs on February 14, 2021, 13:55:45 pm
24 εβαλα a.  Στο |z|=1 υπαρχει ο DTFT αλλα οχι ο Z. Για |z|<1 ή >1 αδυνατο.

Στο 29 βασει των ερωτησεων του, οριζει διαφορετικα το DFS με Ck = (1/Ν)*Σ... και x[n]=Σ.... Αρα με καποιες προυποθεσεις μπορουμε να πουμε οτι είναι X(k)=Ck/N οπου Ck DFS και X(k) DFT;


Title: Re: [ΨΕΣ] Λύσεις παλιών θεμάτων
Post by: kanou_tom on February 14, 2021, 13:57:59 pm
Μια αναλυτική απάντηση για ερώτηση 21 με κυκλική συνέλιξη;


Title: Re: [ΨΕΣ] Λύσεις παλιών θεμάτων
Post by: Numb3rs on February 14, 2021, 14:08:53 pm
@kanou_tom

Για 18 αν το δουμε αλγοριθμικα χωρις να μας ενδιαφερουν τα ενδιαμεσα σταδια, X(k)=fft(x(n)) και x(n)=ifft(X(k)). Δεν ειναι περιοδικα, ειναι πεπερασμενα. Ασχετως αν κανουμε περιοδικες επεκτασεις για να τα ορισουμε μεσω DFS.
Παντως ειναι και θεμα ορισμου το πως θα εκλαβεις εν τελει την καθε ακολουθια.

Για την 21 το εκανα σε MATLAB


Title: Re: [ΨΕΣ] Λύσεις παλιών θεμάτων
Post by: Thunderlord on February 14, 2021, 14:09:02 pm
Μια αναλυτική απάντηση για ερώτηση 21 με κυκλική συνέλιξη;

Χρησιμοποιείς την εντολή cconv(x1,x2,4) στο matlab και βγαίνει :P


Title: Re: [ΨΕΣ] Λύσεις παλιών θεμάτων
Post by: kanou_tom on February 14, 2021, 14:18:37 pm
Α εντάξει στο MATLAB όντως προκύπτει  :P

Απλά επειδή δεν ξέρω αν επιτρέπεται η χρήση έλεγα μήπως το έβγαλε κανείς και με χέρι..


Title: Re: [ΨΕΣ] Λύσεις παλιών θεμάτων
Post by: danaerose on February 14, 2021, 14:43:01 pm
Ορίστε σορυ για τα γράμματα  :)


Title: Re: [ΨΕΣ] Λύσεις παλιών θεμάτων
Post by: lhmma_Jordan on February 14, 2021, 19:24:10 pm
Λοιπόν δεν είμαι σίγουρος για μερικά, γενικά στα περισσότερα συμφωνούμε. Δεν ξέρω τι παίζει στο (5) (είναι έτοιμος τύπος;) και στο (12) (μάλλον χαμός από πράξεις θα γίνει χωρίς matlab). Γενικά έτσι όπως τα βλέπω, με 75 λεπτά για αυτά τα θέματα που μερικά έχουν και λίγες πράξεις θα γίνει μακελειό. Εκτός αν επιτρέπεται η matlab.

Το αρχείο έχει και (πιθανές) λύσεις δεν είναι μόνο οι επιλογές που έκανα.

Edit: το (6) λάθος.


Title: Re: [ΨΕΣ] Λύσεις παλιών θεμάτων
Post by: Thunderlord on February 14, 2021, 19:32:03 pm
Λοιπόν δεν είμαι σίγουρος για μερικά, γενικά στα περισσότερα συμφωνούμε. Δεν ξέρω τι παίζει στο (5) (είναι έτοιμος τύπος;) και στο (12) (μάλλον χαμός από πράξεις θα γίνει χωρίς matlab). Γενικά έτσι όπως τα βλέπω, με 75 λεπτά για αυτά τα θέματα που μερικά έχουν και λίγες πράξεις θα γίνει μακελειό. Εκτός αν επιτρέπεται η matlab.

Το αρχείο έχει και (πιθανές) λύσεις δεν είναι μόνο οι επιλογές που έκανα.

Στο 6 πώς κατέληξες στο κανένα;


Title: Re: [ΨΕΣ] Λύσεις παλιών θεμάτων
Post by: lhmma_Jordan on February 14, 2021, 19:36:08 pm
Στο 6 πώς κατέληξες στο κανένα;

Χαχαχα δεν διάβασα καλά την εκφώνηση, βιάστικα και πήρα απλά την H(z) . Όντως και αυτό δεν ξέρω πως βγαίνει.


Title: Re: [ΨΕΣ] Λύσεις παλιών θεμάτων
Post by: drak on February 14, 2021, 19:58:51 pm
Μια αναλυτική απάντηση για ερώτηση 21 με κυκλική συνέλιξη;
https://www.youtube.com/watch?v=A6b3UkraTgw
Νομίζω βοηθάει το βίντεο.


Title: Re: [ΨΕΣ] Λύσεις παλιών θεμάτων
Post by: snek on February 14, 2021, 20:05:45 pm
Στο 6 πώς κατέληξες στο κανένα;
Εχεις αναλυτικη λύση για το 6 ? Δεν δινει δεδομενα για συχνοτητα αποκοπης κτλ


Title: Re: [ΨΕΣ] Λύσεις παλιών θεμάτων
Post by: lhmma_Jordan on February 14, 2021, 20:55:57 pm
Εχεις αναλυτικη λύση για το 6 ? Δεν δινει δεδομενα για συχνοτητα αποκοπης κτλ

Πιστεύω μόνο "ποιοτική" απάντηση μπορεί να δωθεί. Κάπου (σελ.80, κάπου εκεί στις σημειώσεις που έχουν ανέβει) αναφέρει πως περνάνε οι συχνότητες κοντά σε πόλους και κόβονται κοντά σε μηδενικά. Άρα εδώ θα παίρναμε μάλλον το ίδιο φίλτρο, αλλά με πόλο από την άλλη. Εκεί δηλαδή που είχαμε πόλο στο z=0.9, να έχουμε στο z= -0.9. Αυτό μας το δίνει το (c), Που εκτός ότι δίνει πόλο στον αρνητικό πραγματικό άξονα, αυτός έχει και ίδιο μέγεθος με τον πόλο που είχε το Lowpass.


Title: Re: [ΨΕΣ] Λύσεις παλιών θεμάτων
Post by: snek on February 14, 2021, 21:18:01 pm
To 26 πως προκυπτει ? τις πραξεις δεν παιζει να τις κανεις ειναι απειρες.


Title: Re: [ΨΕΣ] Λύσεις παλιών θεμάτων
Post by: lhmma_Jordan on February 14, 2021, 21:21:07 pm
To 26 πως προκυπτει ? τις πραξεις δεν παιζει να τις κανεις ειναι απειρες.

Δεν χρειάζεται, έχει π/(2n) και στις τρεις επιλογές που είναι λάθος. Αν γινόντουσαν, έχουμε κάνει την ανάλυση για είσοδο ημίτονα/συνημίτονα και θα έβγαινε σχετικά εύκολα.


Title: Re: [ΨΕΣ] Λύσεις παλιών θεμάτων
Post by: snek on February 14, 2021, 21:23:11 pm
Δεν χρειάζεται, έχει π/(2n) και στις τρεις επιλογές που είναι λάθος. Αν γινόντουσαν, έχουμε κάνει την ανάλυση για είσοδο ημίτονα/συνημίτονα και θα έβγαινε σχετικά εύκολα.
Ναι , το προβλημα με το π/(2n) δεν κτλβ ποιο ειναι


Title: Re: [ΨΕΣ] Λύσεις παλιών θεμάτων
Post by: Thunderlord on February 14, 2021, 21:25:08 pm
To 26 πως προκυπτει ? τις πραξεις δεν παιζει να τις κανεις ειναι απειρες.

Έχει ένα n στον παρονομοαστή που δεν βγάζει νόημα. Κατά τα άλλα, υπάρχουν κι άλλα τυπογραφικά αλλά αν διορθωθουν, σωστό είναι το b

Οι πράξεις είναι εύκολες, απλά όταν έχεις είσοδο cosω0n η έξοδος θα είναι |H(ω0)cos(ω0n+arg(H(ω0))

Για το σταθερό όρο το κάνεις 10cos0n


Title: Re: [ΨΕΣ] Λύσεις παλιών θεμάτων
Post by: lhmma_Jordan on February 14, 2021, 21:59:30 pm
Για το 5 υπάρχει πουθενά η λύση; Δεν θυμάμαι να πήρε κάπου το μάτι μου sin(n)/n.


Title: Re: [ΨΕΣ] Λύσεις παλιών θεμάτων
Post by: snek on February 14, 2021, 22:34:29 pm
Για το 5 υπάρχει πουθενά η λύση; Δεν θυμάμαι να πήρε κάπου το μάτι μου sin(n)/n.
Σημειωσεις καναβουρα σελ20


Title: Re: [ΨΕΣ] Λύσεις παλιών θεμάτων
Post by: cgeorgoud on February 06, 2022, 22:08:26 pm
Έχει λύσει κανεις φεβρουαριο 2020?


Title: Re: [ΨΕΣ] Λύσεις παλιών θεμάτων
Post by: illuv4tar on February 14, 2022, 22:22:02 pm
Σεπτέμβρη του 16 θέμα 4 στο β ερωτημα ζητάει να πουμε αν η h[n] ειναι πραγματική χωρίς να την υπολογίσουμε. έχει κανεις καμιά ιδέα για το πότε είναι πραγματική εμπειρικά ? σκέφτηκα οτι άμα  αναπτύξουμε τις εξισώσεις διαφορών βγάζουμε όρους αναδρομικούς οπότε είναι IIR το σύστημα, αλλά δεν ξέρω αν αυτό ειναι αρκετο


Title: Re: [ΨΕΣ] Λύσεις παλιών θεμάτων
Post by: Πατερ Ημμυων on February 14, 2022, 22:30:06 pm
Γενικά μου φαίνεται περίεργο το οτι έχει μόνο έναν πόλο μιγαδικό, δηλαδή σε πραγματικά συστήματα θα είχαμε και το συζυγές του, οπότε θα έλεγα οτι δεν είναι πραγματικό


Title: Re: [ΨΕΣ] Λύσεις παλιών θεμάτων
Post by: illuv4tar on February 14, 2022, 22:59:51 pm
Φλεβάρης του 20, θέμα πεμπτο, η Ha(s) έχει δύο πόλους s1=-1 και s2= -2 στο αριστερό ημιεπίπεδο, άρα είναι ευσταθης. Στις σημειώσεις του ρέκανου λεεί ότι άμα είναι η αναλογική ευσταθής θα είναι και η ψηφιακή. Εγώ στο ψηφιακό βρήκα 2 πόλους |e^Τ| και |e^2T|. h[n]=T( e^nT * u[n]  - e^2nT * u[n] ), άρα είναι αιτιατή , σωστα ? η Η(z) θα έχει ROC |z|>e^2T , το οποίο όμως για κάθε Τ θετικό  βγαίνει >1, άρα ο μοναδιαίος δεν ανοικει στο ROC, άρα δεν είναι ευσταθες. Κάνω κάπου λάθος στον συλλογισμό μου ?


Title: Re: [ΨΕΣ] Λύσεις παλιών θεμάτων
Post by: Πατερ Ημμυων on February 14, 2022, 23:17:52 pm
Νομίζω βγαίνει e^(-nT)  και e^(-2nT) οπότε για αιτιατό θα έχουν μέτρα μικρότερα του 1.


Title: Re: [ΨΕΣ] Λύσεις παλιών θεμάτων
Post by: illuv4tar on February 14, 2022, 23:20:53 pm
ναι σωστα, ξεχασα τον λαπλας με τα πολλα  διακριτα  :D


Title: Re: [ΨΕΣ] Λύσεις παλιών θεμάτων
Post by: nilatos on February 09, 2023, 18:50:31 pm
Θεμα 2, Σεπτεμβρης 2019 πως ειναι ευσταθες το συστημα με αυτους τους πολους που εχει η H(z) ? Δεν θα πρεπει οι πολοι να ναι εντος του μοναδιαιου κυκλου λογω σχεσης Laplace-Z ? Εγω βγαζω ζ=2 και ζ=1/2... χάνω κατι?


Title: Re: [ΨΕΣ] Λύσεις παλιών θεμάτων
Post by: odysseass on February 12, 2023, 22:56:48 pm
Θεμα 2, Σεπτεμβρης 2019 πως ειναι ευσταθες το συστημα με αυτους τους πολους που εχει η H(z) ? Δεν θα πρεπει οι πολοι να ναι εντος του μοναδιαιου κυκλου λογω σχεσης Laplace-Z ? Εγω βγαζω ζ=2 και ζ=1/2... χάνω κατι?

Πρέπει η περιοχή σύγκλισης να περιέχει τον μοναδιαίο κύκλο για να μπορείς να βάλεις e^jω για να υπάρχει ο Fourier. Τους πόλους τους βγάζεις σωστά, αλλα αυτόι οι 2 μας δίνουν τρεις πιθανές περιοχές σύγκλισης: |z| < 1/2 ή 1/2 < |z| < 2 ή |z| > 2. Άρα η εκφώνηση σου λέει εμμεσα ότι ROC είναι η δεύτερη περιπτωση, οπότε οταν κάνεις inverse zeta για να βρεις h[n] πρέπει να προσέξεις στο 2 να βάλεις τον τύπο με το u[-n-1]


Title: Re: [ΨΕΣ] Λύσεις παλιών θεμάτων
Post by: odysseass on February 12, 2023, 23:03:57 pm
Υπάρχουν λύσεις για Φεβρουαριο του 22; Και μήπως έχει κανείς και του Σεπτεμβρίου 22, να δούμε αν επέμεινε σε πάρομοιας φύσης θέματα, γιατί αν είναι να βάλει σαν τα περσινά να το αφήσουμε το μάθημα απο τώρα  :P


Title: Re: [ΨΕΣ] Λύσεις παλιών θεμάτων
Post by: gpapadimi on February 13, 2023, 00:38:08 am
Αν κάποιος έχει τα  θέματα του Σεπτεμβρίου του 22 θα ήταν πράγματι πολύ βοηθητικό να τα ανεβάσει, αν και μάλλον θα κινήθηκαν στα ίδια επίπεδα με του Φλεβάρη υποθέτω.
sent from mTHMMY (https://play.google.com/store/apps/details?id=gr.thmmy.mthmmy) 


Title: Re: [ΨΕΣ] Λύσεις παλιών θεμάτων
Post by: odysseass on February 13, 2023, 13:38:14 pm
Τα βρήκα από έναν φίλο τελικά, τα έστειλα και για upload


Title: Re: [ΨΕΣ] Λύσεις παλιών θεμάτων
Post by: nilatos on February 15, 2023, 03:33:25 am
Αν μας ζηταει να βρουμε απο το H(z) το H(f) και οχι το H(ω) υπάρχει διαφορετική μεθοδολογια? Γιατι λεει στο Β θεμα του σεπτεμβριου, να βρειτε αποκριση συχνοτητας |H(f)| και οχι |H(ω)|.


Title: Re: [ΨΕΣ] Λύσεις παλιών θεμάτων
Post by: SilentLightning on February 15, 2023, 12:14:12 pm
Αν μας ζηταει να βρουμε απο το H(z) το H(f) και οχι το H(ω) υπάρχει διαφορετική μεθοδολογια? Γιατι λεει στο Β θεμα του σεπτεμβριου, να βρειτε αποκριση συχνοτητας |H(f)| και οχι |H(ω)|.
Νομίζω αντικαθιστάς στην Η(z) όπου z = e^(j * 2π * f / fs)


Title: Re: [ΨΕΣ] Λύσεις παλιών θεμάτων
Post by: nilatos on February 15, 2023, 15:15:14 pm
exei lusei kaneis oloklhro to 2o thema tou septemvrh?


έχει λύσει κανείς ολόκληρο το 2ο θέμα του Σεπτέμβρη;


edit: no greeklish


Title: Re: [ΨΕΣ] Λύσεις παλιών θεμάτων
Post by: nilatos on February 15, 2023, 17:12:16 pm
Αν εχω x[n]=sin(π/2n) για να βρω το y[n] στη μονιμη κατασταση μπορώ να το κάνω απευθέιας με αποκριση συχνοτητας μετρο φαση κτλ η πρεπει να το μετασχηματισω σε μορφη cos[n]?


Title: Re: [ΨΕΣ] Λύσεις παλιών θεμάτων
Post by: manek on February 15, 2023, 17:31:39 pm
Αν εχω x[n]=sin(π/2n) για να βρω το y[n] στη μονιμη κατασταση μπορώ να το κάνω απευθέιας με αποκριση συχνοτητας μετρο φαση κτλ η πρεπει να το μετασχηματισω σε μορφη cos[n]?
Νομίζω απευθείας γίνεται,δε χρειάζεται να το κάνεις συνημίτονο.


Title: Re: [ΨΕΣ] Λύσεις παλιών θεμάτων
Post by: SilentLightning on February 15, 2023, 17:43:46 pm
έχει λύσει κανείς ολόκληρο το 2ο θέμα του Σεπτέμβρη;
Επισυνάπτω, με επιφύλαξη για λάθη.


Title: Re: [ΨΕΣ] Λύσεις παλιών θεμάτων
Post by: nilatos on February 15, 2023, 18:57:13 pm
Και γω τοσο το βγαλα..


Title: Re: [ΨΕΣ] Λύσεις παλιών θεμάτων
Post by: nilatos on February 15, 2023, 19:00:57 pm
Θεμα 1 Φεβ 22 με κυκλικη συνελιξη κανεις;


Title: Re: [ΨΕΣ] Λύσεις παλιών θεμάτων
Post by: SilentLightning on February 15, 2023, 19:27:55 pm
Θεμα 1 Φεβ 22 με κυκλικη συνελιξη κανεις;
Σορρυ, είναι λίγο κακογραμμένη κι αυτή, η λογική είναι ότι η γραμμική συνέλιξη θα έχει ακριβώς 8 όρους, δηλ μη μηδενικοί όροι της y[n] μπορούν να είναι μόνο oι y[0],...,y[7]
Γενικά ισχύει ότι αν το χ αρχίζει από κάποιο n1 και τελειώνει σε κάποιο n2
και το h αρχίζει από κάποιο m1 και τελειώνει σε κάποιο m2
η γραμμική συνέλιξη των x,h θα αρχίζει από το n = n1+m1 και θα τελειώνει στο n = n2+m2
αν δεν κάνω λάθος.
Εδωπέρα έχεις n1 = 0 , n2 = 3, m1 = 0 , m2 = 4.
Λόγω αυτών των μηδενισμών της y[n] έξω από το σύνολο {0,1,...,7}, στο απειρο αθροισμα που σου δίνει επιβιώνουν λίγοι όροι και έτσι λύνεις το απλό σύστημα.


Title: Re: [ΨΕΣ] Λύσεις παλιών θεμάτων
Post by: kathrin_p on February 15, 2023, 19:41:37 pm
Θεμα 2 Φεβρ 2022 καμια ιδεα?


Title: Re: [ΨΕΣ] Λύσεις παλιών θεμάτων
Post by: manek on February 15, 2023, 19:55:09 pm
Στο Θεμα 1 Σεπτεμβριου 2022 σας βγηκε x[n]=0 για n>0?


Title: Re: [ΨΕΣ] Λύσεις παλιών θεμάτων
Post by: manek on February 15, 2023, 20:03:09 pm
Θεμα 2 Φεβρ 2022 καμια ιδεα?

Σε αυτό πρώτα βρίσκεις το x[n] από το τύπο που δίνει και μετά για τον DTFT λες ότι το επιμέρους γινόμενο των 2 sinc στον χρόνο που έχεις γίνεται συνέλιξη των επιμέρους DTFT τους,που ο ο DTFT της κάθε μιας προκύπτει ένας ορθογώνιος παλμός με διαφορετική διάρκεια.Μετά κάνεις συνέλιξη αυτών των 2 παλμών και προκύπτει ένα σχήμα σαν υπερυψωμένο τραπέζιο.


Title: Re: [ΨΕΣ] Λύσεις παλιών θεμάτων
Post by: SilentLightning on February 15, 2023, 20:07:39 pm
Θεμα 2 Φεβρ 2022 καμια ιδεα?
Ναι, από το βιβλίο μπερμπερίδη στα σήματα η σχέση 5.54. Είχε κάνει κι ο Κίτσας μια άσκηση μέσα στο εξάμηνο που την χρησιμοποίησε.
Η λογική είναι όπως είπε και o manek
Στο Θεμα 1 Σεπτεμβριου 2022 σας βγηκε x(n)=0 για n>0?
για ν<0 ξέρεις ότι είναι 0 από εκφώνηση.
Εμένα μου βγήκε x[n] = δ(n) - 3δ(n-2) + 2δ(n-3)
(μπορεί να ναι λάθος)
από σχέσεις kramers-kronig σημειώσεις σελ 62
https://helit.org/ece-notes/dsp.pdf
Edit: βάζω και την λύση στο θ1 σεπτ 22


Title: Re: [ΨΕΣ] Λύσεις παλιών θεμάτων
Post by: manek on February 15, 2023, 20:15:23 pm
Kαι εγω σχεσεις Kramer Kronig πηρα,απλα μετα στο ολοκληρωμα που βγαινει μετα απο πραξεις μου εβγαιναν κατι sin((3-n)π) και sin((3+n )π) οπου επειδη το n ειναι ακεραιο θα βγαινουν μηδεν.Μπορει να εκανα λαθος στις πραξεις.


Title: Re: [ΨΕΣ] Λύσεις παλιών θεμάτων
Post by: odysseass on February 15, 2023, 21:31:49 pm
Και μένα 0 βγήκε για τους ίδιους λόγους


Title: Re: [ΨΕΣ] Λύσεις παλιών θεμάτων
Post by: manek on February 15, 2023, 21:50:59 pm
Και μένα 0 βγήκε για τους ίδιους λόγους
Η λυση του silent ligthning πρεπει να ειναι η σωστη παντως.
Καταλαβα τι εκανα λαθος.
Για τα ημιτονο που ειπα βγαινει συγκεκριμενα (1/(n+3))*sin[(n+3)π] οπου σε ολες τις τιμες εκτος n=-3 μηδενιζεται.Στο n=-3 ομως γινεται 0/0 και μετα πρεπει να παρεις το οριο στο n=-3,κανεις delopital και βγαινει το οριο 1,οποτε ολο αυτο παλι καταληγει σε δ(n+3).Το ιδιο και με τα αλλα παρομοια ημιτονα για να βγουν οι αλλες δ.Απιστευτο πως μια λεπτομερεια αλλαζει ολο το αποτελεσμα.


Title: Re: [ΨΕΣ] Λύσεις παλιών θεμάτων
Post by: nilatos on February 15, 2023, 23:00:16 pm
το αποτελεσμα της συνελιξης στο θεμα 2 φεβ 22 γιατι βγαινει τοσο?


Title: Re: [ΨΕΣ] Λύσεις παλιών θεμάτων
Post by: odysseass on February 15, 2023, 23:41:24 pm
Η λυση του silent ligthning πρεπει να ειναι η σωστη παντως.
Καταλαβα τι εκανα λαθος.
Για τα ημιτονο που ειπα βγαινει συγκεκριμενα (1/(n+3))*sin[(n+3)π] οπου σε ολες τις τιμες εκτος n=-3 μηδενιζεται.Στο n=-3 ομως γινεται 0/0 και μετα πρεπει να παρεις το οριο στο n=-3,κανεις delopital και βγαινει το οριο 1,οποτε ολο αυτο παλι καταληγει σε δ(n+3).Το ιδιο και με τα αλλα παρομοια ημιτονα για να βγουν οι αλλες δ.Απιστευτο πως μια λεπτομερεια αλλαζει ολο το αποτελεσμα.

Δεν το σκέφτηκα έτσι, πολυ καλη παρατήρηση γιατί το είχε και σε άλλη χρόνια παρόμοιο θεμα


Title: Re: [ΨΕΣ] Λύσεις παλιών θεμάτων
Post by: odysseass on February 15, 2023, 23:42:59 pm
το αποτελεσμα της συνελιξης στο θεμα 2 φεβ 22 γιατι βγαινει τοσο?

Από ποια άποψη;


Title: Re: [ΨΕΣ] Λύσεις παλιών θεμάτων
Post by: nilatos on February 16, 2023, 01:30:41 am
Aυτα δεν τα παιρνουμε σε ορθογωνιους παλμους?Κατι δεν θυμαμαι πανω σε αυτο μην λεω και μπαρουφες αλλα αφου ειναι Π δεν θα παρουμε απλα ενα "κουτι" με την μικροτερη ω απο τις δυο που υπαρχουν στα Π που γινονται συνελιξη?Αυτη η ευθεια με κλιση απο το π/2 εως π πως υπολογιζεται?Ουσιαστικα δηλαδη δεν θυμαμαι ακριβως γιατι βγαινει ετσι απο π/2 ως π


Title: Re: [ΨΕΣ] Λύσεις παλιών θεμάτων
Post by: nilatos on February 16, 2023, 02:39:21 am
Θεμα 2 σεπτεμβριος 2019 πως βρισκουμε y[n] αφου το x[n] δεν εχει u[n] και ειναι απειρης διαρκειας?


Title: Re: [ΨΕΣ] Λύσεις παλιών θεμάτων
Post by: SilentLightning on February 16, 2023, 13:05:45 pm
Θεμα 2 σεπτεμβριος 2019 πως βρισκουμε y[n] αφου το x[n] δεν εχει u[n] και ειναι απειρης διαρκειας?
πολλαπλασιάζεις με το πλάτος της αποκρισης συχν και προσθετεις την φάση της στο όρισμα του συνημιτόνου.
Αυτός ο τύπος κανονικά ισχύει μόνο όταν έχεις άπειρη διάρκεια δηλ cos(ω0n + φ) (ή με ημίτονο)
Απλά σε κάτι άλλες ασκήσεις έλεγε βρείτε την έξοδο στη μόνιμη κατάσταση όπου επειδή έχει περάσει άπειρος χρόνος είναι σαν να υπήρχε από πάντα η ημιτονική είσοδος χωρίς το u[n].


Title: Re: [ΨΕΣ] Λύσεις παλιών θεμάτων
Post by: nilatos on February 16, 2023, 13:08:57 pm
Σωστος


Title: Re: [ΨΕΣ] Λύσεις παλιών θεμάτων
Post by: SilentLightning on February 16, 2023, 13:47:34 pm
το αποτελεσμα της συνελιξης στο θεμα 2 φεβ 22 γιατι βγαινει τοσο?
υπάρχει ένας αναλυτικός τρόπος για να μην μπλέκεις με την γεωμετρία τόσο,
αν εκφράσεις τους τετραγων. παλμούς σαν διαφορά 2 μετατοπισμένων βηματικών,
δηλαδή Α*(u(t+B) - u(t-B)). Επισυνάπτω ένα πδφ από εποχή covid στα σήματα